Está en la página 1de 184

REPÚBLICA BOLIVARIANA DE VENEZUELA

MINISTERIO DEL PODER POPULAR PARA LA EDUCACIÓN


U.E. COLEGIO “SAN AGUSTÍN” EL MARQUÉS
DEPARTAMENTO DE MATEMÁTICAS Y FÍSICA

MATEMÁTICA 4TO AÑO EDUCACIÓN MEDIA


Índice
Funciones algebraicas 2
Función exponencial y logaritmica 66
1 Función Logarítmica 69
Funciones trigonométricas 83
Números Complejos A 148
Progresiones aritméticas y geométricas 170
República Bolivariana de Venezuela
Ministerio del Poder Popular para la Educación
Unidad Educativa: Colegio San Agustín – El Marqués
Profesor: Clemente Moreno P

Funciones
Consideraciones previas

El plano cartesiano es un sistema de referencia compuesto de


dos rectas numéricas, una dispuesta horizontalmente y la otra
verticalmente, llamadas ejes, que se intersecan en el punto (0,0)
u origen del sistema de coordenadas, generando 4 regiones o
cuadrantes, identificadas con números romanos, dispuestas en
sentido contrario a las manecillas del reloj.
A la recta horizontal se llama eje «𝑥𝑥» o eje de las abscisas y a

la recta vertical, eje «𝑦𝑦» o eje de las ordenadas, los ejes no pertenecen a ningún cuadrante.

Los puntos en el plano se identifican con letras mayúsculas


A, B, C, etc., y se denotan en la forma A(a, b), donde A es el
nombre del punto, mientras que «a» y «b» corresponden a sus
coordenadas respecto al eje «𝑥𝑥» y al eje «𝑦𝑦», respectivamente.
La abscisa «a» es la distancia a la que está el punto del eje «𝑦𝑦»,
medida paralelamente al eje «𝑥𝑥»; la ordenada «b» es la distancia
del punto al eje «𝑥𝑥», medida paralelamente al eje «𝑦𝑦». Así, al

ubicar el punto A de coordenadas (2, 3) se debe mover dos unidades hacia la derecha sobre
el eje «𝑥𝑥» con respecto al origen y luego desde 2 ubicado sobre el eje «𝑥𝑥» se debe subir 3
unidades en forma paralela al eje «𝑦𝑦».

Todo punto en el eje de las abscisas tiene ordenada cero «A(a, 0)» y todo punto en el eje
de las ordenadas tiene abscisa cero «B(0, b)». Así E(1,0) y G(−4,0) se ubican en eje de las
abscisas, mientras que F(0,4) y H(0, −4) se ubican en el eje de las ordenadas.

CLEMENTE MORENO (MATERIAL DIDÁCTICO EN VALIDACIÓN 1


En el plano coordenado se definen las llamadas relaciones binarias, entendidas como
cualquier propiedad que relaciona las variables «𝑥𝑥» e «𝑦𝑦» conformando pares ordenados (𝑥𝑥, 𝑦𝑦)
que dan pie a figuras planas, cuyas propiedades matemáticas pueden estudiarse en el plano
cartesiano. Una de estas relaciones binarias es el objeto matemático función que se define a
continuación.

La Función en matemática
Una función es un conjunto de pares ordenados (𝑥𝑥, 𝑦𝑦), donde no existen dos parejas que
tengan igual la primera componente, que es «𝑥𝑥», en el par ordenado (𝑥𝑥, 𝑦𝑦)

Ejemplo. Considere los conjuntos A = {𝑎𝑎, 𝑏𝑏, 𝑐𝑐, 𝑑𝑑} y B = {1, 2, 3, 4, 5} y las relaciones definidas
en los diagramas como se indica, en ellos se tiene que:

𝑓𝑓1 𝑓𝑓2 𝑓𝑓3


A B B B
A A

a ° °1 ° 1 1
a° a° °
° 2 2 2
b °
b° b
°
°
° 3 3
3 °
c °
4 c °
°
° 4 c° ° 4
d° ° 5 d° d° 5
° 5 °

En el conjunto 𝑓𝑓1 = {(a, 1), (b, 3), (c, 2)} el elemento d ∈ A no se relaciona con ningún
elemento del conjunto B; en 𝑓𝑓2 = {(a, 1), (a, 4), (b, 3), (c, 2), (d, 5)} todos los elementos de A están
relacionados con los elementos B, pero las dos primeras parejas del conjunto 𝑓𝑓2 tienen igual la
primera componente; en 𝑓𝑓3 = {(a, 1), (b, 3), (c, 2), (d, 5) } todos los elementos de A están
relacionados con los elementos de B y en el conjunto no existen dos pares ordenados que
tengan igual la primera componente. Por tal razón, sólo 𝑓𝑓3 califica como función

Definición. Una función 𝑓𝑓 es un conjunto de pares ordenados que cumple la siguiente


propiedad:
Si (𝑥𝑥, 𝑦𝑦) ∈ 𝑓𝑓 y (𝑥𝑥, 𝑧𝑧) ∈ 𝑓𝑓 entonces 𝑦𝑦 = 𝑧𝑧 (∗)

CLEMENTE MORENO (MATERIAL DIDÁCTICO EN VALIDACIÓN 2


Lo expresado en (∗) es la forma matemática de decir que en el conjunto de definición de
la función, no existen dos parejas ordenadas distintas que tengan igual la primera componente.

Si 𝑓𝑓 es una función y (𝑥𝑥, 𝑦𝑦) ∈ 𝑓𝑓, a «𝑦𝑦» se denomina valor de 𝑓𝑓 en «𝑥𝑥» y se designa por
𝑓𝑓(𝑥𝑥), hecho que se denota escribiendo 𝑦𝑦 = 𝑓𝑓(𝑥𝑥).

Al conjunto de todos los valores posibles de «𝑥𝑥» se llama dominio, es decir, el conjunto
de los puntos donde 𝑓𝑓 está definida, mientras que al conjunto de todos los valores posibles de
«𝑦𝑦», esto es, el conjunto de los valores que toma 𝑓𝑓 es denominado rango de la función.

COMENTARIOS

1. La definición de función incluye tres aspectos: el dominio de definición de la función, el rango


de la función y la relación o regla de correspondencia 𝑓𝑓, entendida ésta, como la proposición
que relaciona cada elemento «𝑥𝑥» del dominio con un único elemento «𝑦𝑦» del rango. Tanto
«𝑥𝑥» como «𝑦𝑦» son variables, «𝑥𝑥» es la variable independiente y «𝑦𝑦» la variable dependiente,
equivalente a decir que «𝑦𝑦 depende de 𝑥𝑥» o también que «𝑦𝑦» es función de «𝑥𝑥».
En particular, el tiempo «𝑡𝑡» requerido para realizar una obra depende del número de
obreros empleados, luego el tiempo es función del número de obreros, lo cual se escribe
𝑡𝑡 = 𝑓𝑓(𝑜𝑜𝑜𝑜𝑜𝑜𝑜𝑜𝑜𝑜𝑜𝑜𝑜𝑜); el interés «𝐼𝐼» mensual que produce un capital depende del tanto por ciento
«𝑟𝑟» a que esté colocado, luego el interés es función del tanto por ciento y se escribe I = 𝑓𝑓(𝑟𝑟);
el salario «𝑆𝑆» de una persona depende entre otras cosas del tiempo que haya trabajado,
luego se escribe S = 𝑓𝑓(𝑡𝑡).

2. Debido a que toda función es una relación «especial», es lícito hablar de su gráfica como el
conjunto de puntos correspondientes a los pares ordenados del grafo de la función, la
característica de estos pares impide que una recta vertical pueda cortar el gráfico en más de
un punto. Este hecho permite identificar una relación funcional con sólo avistar su gráfica, así

CLEMENTE MORENO (MATERIAL DIDÁCTICO EN VALIDACIÓN 3


de las cuatro curvas adjuntas, las gráficas «a» y «c» corresponden a funciones, mientras que
«b» y «d» no son relaciones funcionales.

Y Y Y
Y

°
X
X X
X

(a) (b) (c) (d)

3. La notación 𝑓𝑓: X → Y indica que 𝑓𝑓 es una función con dominio el conjunto X y rango contenido
en el conjunto Y. Si tanto X como Y son subconjuntos del conjunto de los números reales ℝ,
entonces la función 𝑓𝑓 es una función real de variable real.

Ejercicio N° 𝟏𝟏
En los casos siguientes, indique si la relación es una función. Justifique su respuesta atendiendo
a la definición de función
1) Considere la relación «se sienta en» entre el conjunto de trece estudiantes que asisten a una
aula de clase que cuenta con doce pupitres, suponga que cada alumno ocupa un solo pupitre.
9
2) ℛ1 = �(1, 3), (2, 4), �√4, 6�, � , 7� , (6, 8)�
3

3) ℛ2 = �(𝑥𝑥, 𝑦𝑦)⁄𝑦𝑦 = √4 − 𝑥𝑥 2 �
4) ℛ3 = {(𝑥𝑥, 𝑦𝑦)⁄𝑦𝑦 > 𝑥𝑥 + 1}
5) ℛ4 = �(𝑥𝑥, 𝑦𝑦)⁄𝑥𝑥 = �4 − 𝑦𝑦 2 �
6) ℛ5 = {(𝑥𝑥, 𝑦𝑦)⁄𝑥𝑥 = 𝑦𝑦 2 }
7) ℛ6 = {(𝑥𝑥, 𝑦𝑦)⁄𝑥𝑥 = −2}

Las siguientes gráficas corresponden a relaciones, indique cuales de ellas son funciones.
Razone su respuesta en conexión con la definición de función.

CLEMENTE MORENO (MATERIAL DIDÁCTICO EN VALIDACIÓN 4


Y Y Y Y Y

X X
X X
x=k
(8) (9) (10) (11) (12)

Decida si los puntos que se indican pertenecen o no al grafo «conjunto de pares ordenados» de
𝑦𝑦 = 𝑓𝑓(𝑥𝑥).
13) 𝑓𝑓(𝑥𝑥) = 2𝑥𝑥 + 3; (0, 3), (−2, 0), (−1, 1), (1, 5), (2, 3);
14) 𝑓𝑓(𝑥𝑥) = 2 − 3𝑥𝑥 + 𝑥𝑥 2 ; (−2, 12); (−1, 6); (0, −2); (1, 1) (2, 0);
𝑥𝑥 2 − 𝑥𝑥 − 6 1
15) 𝑓𝑓(𝑥𝑥) = ; (−3, 0); (2, 0); (0, −1); (1, − ); (3, 2);
𝑥𝑥 2 + 5𝑥𝑥 + 6 2

�√𝑥𝑥 + 1 3
16) 𝑓𝑓(𝑥𝑥) = 3 ; (−1, 0); (0, 1); (1, √2); (3, −6); (4, √5);
√𝑥𝑥

COMENTARIO

para resolver la situación anterior, sustituya a «𝑥𝑥» por la primera componente del par,
ejecute las operaciones indicadas y vea si el resultado coincide con la segunda componente;
en este caso el par pertenece al grafo de 𝑓𝑓, en otro caso el par no es del grafo de la función 𝑓𝑓

Funciones reales elementales


Función constante 𝑓𝑓(𝑥𝑥) = k; con k ∈ ℝ «cualquier número real «𝑥𝑥» es transformado en el número «k»»

Ejemplo. Considere la función 𝑓𝑓(𝑥𝑥) = 3 «aquí k vale 3; la función transforma todos los números en 3»
3 1 1
Así, … 𝑓𝑓(−2) = 3; 𝑓𝑓 �− � = 3; 𝑓𝑓(−1) = 3; 𝑓𝑓 �− � = 3; 𝑓𝑓(0) = 3; 𝑓𝑓 � � = 3; 𝑓𝑓(1) = 3; 𝑓𝑓(1) = 2..
2 2 2

es un cálculo que genera algunos pares ordenados del grafo de la función, es decir
3 1 1 3
𝑓𝑓 = �… (−2,3), … �− , 3� , . . (−1,3), … �− , 3� , … (0,3), … � , 3� , … (1,3), … � , 3� , … (2,3), … �
2 2 2 2

Los puntos suspensivos antes del primer par ordenado indican que se pudo iniciar en un
número real más pequeño, los puntos entre par ordenado y par ordenado señalan que entre

CLEMENTE MORENO (MATERIAL DIDÁCTICO EN VALIDACIÓN 5


las primeras componentes de los pares hay infinitos números reales que pueden considerase y
los puntos al final del último par ordenado dicen que se pueden tomar muchos otros pares. Este
hecho indica que el dominio de la función es ℝ y se escribe 𝐷𝐷𝐷𝐷𝐷𝐷(𝑓𝑓) = ℝ

Del grafo de la función se suele tomar un subconjunto finito y representarse en una tabla,
donde «𝑥𝑥» es la variable independiente y «𝑦𝑦» la variable dependiente, tal como se indica:
«en matemática, la expresión de la función a través de
3 1 1 3
tablas, se conoce como modo de representación tabular de
𝑥𝑥 −2 −
2 −1 − 2 0 2 1 2 2
la función»
𝑦𝑦 3 3 3 3 3 3 3 3 3

Cuando los pares ordenados de la tabla o del grafo de la función 𝑓𝑓, se ubican en el
plano y se unen mediante un trazo continuo, previa confirmación que el dominio de 𝑓𝑓 es ℝ
«conjunto de los números reales», se obtiene la gráfica de 𝑓𝑓, en este caso se tiene:
(i) «para representar los pares ordenados contenidos en la
𝑦𝑦 tabla se siguen las instrucciones dadas para ubicar puntos
y=k en el plano, en este caso están coloreados de rojo»
∙ ∙ ∙ ∙ 3∙ ∙ ∙ ∙ ∙
2 (ii) «los puntos se unen mediante un trazo continuo, sabiendo
1 que el dominio de la función es el conjunto de los reales ℝ»
𝑥𝑥
3 1 (iii) «la representación gráfica de la función es una recta que
0 1 1 3 2
−2 − −1 − pasa por 3 y es paralela al eje «𝑥𝑥»»
2 2 2 2
(iv) «este es el modo de representación grafico de la función»

En general, para 𝑓𝑓(𝑥𝑥) = k; con k ∈ ℝ se tiene que:


Y
f ( x) = k
𝐺𝐺𝐺𝐺𝐺𝐺(𝑓𝑓) = {(x, k): 𝑥𝑥 ∈ ℝ}; 𝐷𝐷𝐷𝐷𝐷𝐷(𝑓𝑓) = ℝ; 𝑅𝑅𝑅𝑅𝑅𝑅(𝑓𝑓) = {k};
La representación gráfica de 𝑓𝑓 es recta la horizontal de la figura
X

Función identidad 𝑔𝑔(𝑥𝑥) = 𝑥𝑥 «cualquier número real «𝑥𝑥» es transformado en el mismo número «𝑥𝑥»»

1 1 1 1
Así, … 𝑔𝑔(−2) = −2; 𝑔𝑔(−1) = −1; 𝑔𝑔 �− � = − ; 𝑔𝑔(0) = 0; 𝑔𝑔 � � = ; 𝑔𝑔(1) = 1; 𝑔𝑔(2) = 2.. luego
2 2 2 2
1 1 1 1
𝑔𝑔 = �… (−2, −2), … (−1, −1), … �− , − � , … (0,0), … � , � , … (1,1), … (2,2), … � se corresponde
2 2 2 2

con el grafo de la función, mientras que


(i) «la tabla contiene un subconjunto finito del grafo de
3 1 1 3 la función identidad 𝑔𝑔»
𝑥𝑥 −2 − −1 − 0 2 1 2
2 2 2
3 1 3 (ii) «la tabla corresponde al modo de representación
0 1
𝑦𝑦 −2 − −1 − 1 2 tabular de la función identidad»
2 2 2 2

CLEMENTE MORENO (MATERIAL DIDÁCTICO EN VALIDACIÓN 6


Por su parte, la recta de la figura, corresponde al modo de representación gráfico de la
función idéntica 𝑔𝑔
(i) «la representación gráfica de la función 𝑔𝑔 es una recta
oblicua, cuya inclinación «pendiente» en grados es 45° y en
𝑦𝑦 radianes «número» es 1, que coincide con el coeficiente de
2 𝑥𝑥»
3
(ii) «la recta oblicua se prolonga indefinidamente, de modo que
2
al considerar un número real muy pequeño, tendiendo a −∞
1 «menos infinito» sobre el eje 𝑥𝑥, desde él se puede imaginar
1 un segmento punteado que se encuentra con la recta; igual
2 ocurre si se considera un número muy grande, tendiendo a
+∞ «más infinito» sobre el eje 𝑥𝑥, el segmento punteado se
3 1 1 3 encontrará con la recta oblicua»
−2 − −1 − 1 2 𝑥𝑥
2 2 1 2 2
− (iii) «lo señalado en (ii) indica que el dominio de la función es el
2
conjunto de los números reales y se escribe 𝐷𝐷𝐷𝐷𝐷𝐷(𝑔𝑔) = ℝ»
−1
3 (iv) «si el número real pequeño, tendiendo a −∞ ahora se

2 imagina en el eje 𝑦𝑦, el segmento punteado también se
encuentra con la recta oblicua, lo mismo sucede si el
−2 número real grande, tendiendo a +∞ se ubica en el eje 𝑦𝑦,
el segmento punteado se encuentra con la recta oblicua»

(v) «(iv) indica que el rango es ℝ y se escribe 𝑅𝑅𝑅𝑅𝑅𝑅(𝑔𝑔) = ℝ»

En general, para 𝑔𝑔(𝑥𝑥) = 𝑥𝑥 que es la función idéntica se Y


tiene: g ( x) = x

𝐺𝐺𝐺𝐺𝐺𝐺(𝑔𝑔) = {(x, x): 𝑥𝑥 ∈ ℝ}; 𝐷𝐷𝐷𝐷𝐷𝐷(𝑔𝑔) = ℝ; 𝑅𝑅𝑅𝑅𝑅𝑅(𝑔𝑔) = ℝ


X
La representación gráfica de 𝑔𝑔 es una línea recta que
pasa por el origen de coordenadas como ilustra la figura.

Función raíz cuadrada ℎ(𝑥𝑥) = √𝑥𝑥 «a número real positivo «𝑥𝑥» le extrae una de las raíces, digamos la positiva»

1 1 √2 3 3 √6
Así ℎ(0) = √0 = 0,… ℎ � � = � = ,… ℎ(1) = √1 = 1,…ℎ � � = � = ,… ℎ(2) = √2,..luego
2 2 2 2 2 2

1 √2 3 √6 5 √10
ℎ = �(0,0), … � , � , … (1,1), … � , � , … �2, √2�, … � , � , … (4, 2), … � corresponde al grafo de
2 2 2 2 2 2

la función, mientras que el modo de representación tabular dado en la tabla, se tiene que
(i) «la tabla contiene un subconjunto finito del grafo de
la función identidad ℎ»
1 3 5
𝑥𝑥 0 1 2 4
2 2 2 (ii) «la tabla como se indicó corresponde al modo de
representación tabular de la función raíz cuadrada»
𝑦𝑦 0 √2 1 √6 √10
√2 2
2 2 2

CLEMENTE MORENO (MATERIAL DIDÁCTICO EN VALIDACIÓN 7


Por su parte, la curva de la figura, corresponde al modo de representación gráfico de la
función idéntica ℎ
(i) «la curva indica que √𝑥𝑥 es un número real, en el caso y solo en el caso
𝑦𝑦 de que 𝑥𝑥 ≥ 0, en otro caso la raíz no tiene sentido en el conjunto ℝ»
2 (ii)
1,414
«lo señalado en (i) indica que el dominio de la función 𝑦𝑦 = ℎ(𝑥𝑥) = √𝑥𝑥 es
1,225 el conjunto de los reales positivos y se escribe 𝐷𝐷𝐷𝐷𝐷𝐷(ℎ) = [0, +∞)»
1
0,707 (iii) «la curva inicia en 0 con alta tasa de crecimiento que luego disminuye
para crecer sin fin de manera moderada»
1 1 3 2 5 3 4 𝑥𝑥
2 2 2 (iv) «lo señalado en (iii) indica que el rango de la función 𝑦𝑦 = ℎ(𝑥𝑥) = √𝑥𝑥 es
el conjunto de los reales positivos y se escribe 𝑅𝑅𝑅𝑅𝑅𝑅(ℎ) = [0, +∞) »»

En general, para ℎ(𝑥𝑥) = √𝑥𝑥 que es la función raíz cuadrada Y

se tiene que: 𝐺𝐺𝐺𝐺𝐺𝐺(ℎ) = �(𝑥𝑥, 𝑦𝑦) ∈ ℝ 2⁄


𝑦𝑦 = √𝑥𝑥�
f ( x) = x
Aquí, ℎ(𝑥𝑥) es un número real si 𝑥𝑥 ∈ ℝ+ ∪ {0} o bien 𝑥𝑥 ≥ 0.
Así, el 𝐷𝐷𝐷𝐷𝐷𝐷(ℎ) = ℝ+ ∪ {0} o su equivalente 𝐷𝐷𝐷𝐷𝐷𝐷(ℎ) = [0, +∞ ), X

mientras que su recorrido o 𝑅𝑅𝑅𝑅𝑅𝑅(ℎ) = [0, +∞).

Algebra de funciones
El álgebra de funciones «operaciones», expresada en la suma, sustracción, producto y
cociente con funciones elementales, unida a la composición «que se estudiará más adelante»
permite generar nuevas funciones.

Definición
Sean 𝑓𝑓 y 𝑔𝑔 dos funciones, suponga que 𝐷𝐷𝐷𝐷𝐷𝐷(𝑓𝑓) y 𝐷𝐷𝐷𝐷𝐷𝐷(𝑔𝑔) son los dominios de 𝑓𝑓 y 𝑔𝑔
𝒇𝒇
en ese orden, entonces se definen las funciones «𝑓𝑓 + 𝑔𝑔», «𝑓𝑓 − 𝑔𝑔», «𝑓𝑓 ∙ 𝑔𝑔» y « »,
𝒈𝒈
respectivamente suma, diferencia, producto y cociente, como se indica a continuación:
(i) (𝑓𝑓 + 𝑔𝑔)(𝑥𝑥) = 𝑓𝑓(𝑥𝑥) + 𝑔𝑔(𝑥𝑥) para todo 𝑥𝑥 ∈ 𝐷𝐷𝐷𝐷𝐷𝐷(𝑓𝑓) ∩ 𝐷𝐷𝐷𝐷𝐷𝐷(𝑔𝑔) «la suma de funciones es otra función»

(ii) (𝑓𝑓 − 𝑔𝑔)(𝑥𝑥) = 𝑓𝑓(𝑥𝑥) − 𝑔𝑔(𝑥𝑥) para todo 𝑥𝑥 ∈ 𝐷𝐷𝐷𝐷𝐷𝐷(𝑓𝑓) ∩ 𝐷𝐷𝐷𝐷𝐷𝐷(𝑔𝑔) «la resta de funciones es otra función»

(iii) (𝑓𝑓 ∙ 𝑔𝑔)(𝑥𝑥) = 𝑓𝑓(𝑥𝑥) ∙ 𝑔𝑔(𝑥𝑥) para todo 𝑥𝑥 ∈ 𝐷𝐷𝐷𝐷𝐷𝐷(𝑓𝑓) ∩ 𝐷𝐷𝐷𝐷𝐷𝐷(𝑔𝑔) «el producto de funciones es otra función»

𝑓𝑓 𝑓𝑓(𝑥𝑥)
(iv) � � (𝑥𝑥) = para todo 𝑥𝑥 ∈ 𝐷𝐷𝐷𝐷𝐷𝐷(𝑓𝑓) ∩ 𝐷𝐷𝐷𝐷𝐷𝐷(𝑔𝑔)~{𝑔𝑔(𝑥𝑥) = 0} «el cociente de funciones es otra función»
𝑔𝑔 𝑔𝑔(𝑥𝑥)

CLEMENTE MORENO (MATERIAL DIDÁCTICO EN VALIDACIÓN 8


COMENTARIO

Antes de realizar cualquiera de las cuatro operaciones entre funciones, se debe verificar
si la intersección de los dominios de las funciones que se van a operar, da como resultado un
conjunto distinto al vacío «conjunto que carece de elementos», pues si la intersección es vacía
la operación resultante no sería función, debido a que no tiene conjunto de definición «dominio».
Además, en el cociente se debe quitar todos aquellos números que anulen el denominador de
la fórmula que resulta de dividir las funciones ya que no es posible dividir entre cero.

Ejemplo 1. Si 𝑓𝑓(𝑥𝑥) = 𝑘𝑘 es la función constante y 𝑔𝑔(𝑥𝑥) = 𝑥𝑥 es la función idéntica, entonces la


función ℎ(𝑥𝑥) = 𝑓𝑓(𝑥𝑥) ∙ 𝑔𝑔(𝑥𝑥) es:
𝐷𝐷𝐷𝐷𝐷𝐷(𝑓𝑓) ∩ 𝐷𝐷𝐷𝐷𝐷𝐷(𝑔𝑔) implica ℝ ∩ ℝ «arriba se demostró que 𝐷𝐷𝐷𝐷𝐷𝐷(𝑓𝑓) = ℝ y 𝐷𝐷𝐷𝐷𝐷𝐷(𝑔𝑔) = ℝ»

Y, ℝ ∩ ℝ = ℝ ℝ
«la recta que representa a ℝ, superpuesta sobre ella misma es ℝ»
0 «la recta ℝ coloreada en rojo se superpone en la coloreada en negro»

Dado que la intersección de los dominios de 𝑓𝑓 y 𝑔𝑔 es ℝ ≠ ∅ « ℝ distinto de vacío», el


producto entre 𝑓𝑓 y 𝑔𝑔 se puede realizar, luego
ℎ(𝑥𝑥) = 𝑓𝑓(𝑥𝑥) ∙ 𝑔𝑔(𝑥𝑥) implica ℎ(𝑥𝑥) = 𝑘𝑘𝑘𝑘 (∗) «k es la pendiente de la idéntica e indica la inclinación de la recta»

En relación con la constante «k» puede ocurrir que:


𝑘𝑘 ∈ (−∞, −1) «k un número real comprendido entre −∞ y −1, la pendiente es negativa y la recta se acerca al eje «𝑦𝑦»»

𝑘𝑘 = −1 «la pendiente es −1 y en grados 135° que llega al 2do cuadrante e indicando que la recta es decreciente»

𝑘𝑘 ∈ (−1,0) «k un número real comprendido entre −1 y 0, la pendiente es negativa y la recta se acerca al eje «𝑥𝑥»»

𝑘𝑘 ∈ (0,1) «k un número real comprendido entre 0 y 1, la pendiente es positiva y la recta se acerca al eje «𝑥𝑥»»

𝑘𝑘 = 1 «la pendiente es 1 y en grados 45° biseca al 1er cuadrante e indicando que la recta es creciente»

𝑘𝑘 ∈ (1, +∞) «k un número real comprendido entre 1 y +∞, la pendiente es positiva y la recta se acerca al eje «𝑦𝑦»»

Las expresiones (−∞, −1), (−1,0), (0,1) y (1, +∞) corresponden a intervalos de números
reales, en este caso en la recta real ℝ se marcaron los puntos −1, 0, 1, lo cual la dividió en
estos cinco intervalos. Además, decir por ejemplo que 𝑘𝑘 ∈ (−∞, −1) significa que «k» puede
597
tomar cualquier valor real comprendido en ese intervalo; así k = −10.000; k = − , k = −1,001
25

son números que pertenecen a ese intervalo. Igual consideración vale para k en los otros
intervalos considerados arriba

CLEMENTE MORENO (MATERIAL DIDÁCTICO EN VALIDACIÓN 9


Ejemplo 2. En la figura adjunta se representa un ejemplo particular de los casos señalados
«en la identificación de las rectas se sustituyó 𝑓𝑓(𝑥𝑥) por 𝑦𝑦, que es su equivalente (ver pag. 3)»
(i) 1 1
«en 𝑦𝑦 = − 2 𝑥𝑥 la constante k = − 2 está en (−1,0) la recta decrece
𝑦𝑦
más lentamente que la recta 𝑦𝑦 = −𝑥𝑥 donde k = −1 y la recta
𝑦𝑦 = −2𝑥𝑥 donde k = −2 está en (−∞, −1) decrece más rápidamente
que la recta 𝑦𝑦 = −𝑥𝑥 en la que k = −1»

«lo apuntado en (i) se puede generalizar para cualquier número


(ii)
que pertenezca a los dos primeros intervalos»

𝑥𝑥 «la recta 𝑦𝑦 = 2𝑥𝑥 donde k = 2 está en (1, +∞) crece más rápido que
(iii) 1 1
la recta 𝑦𝑦 = 𝑥𝑥 en la que k = 1 y la recta 𝑦𝑦 = 𝑥𝑥 donde k = crece
2 2
más lentamente que la recta 𝑦𝑦 = 𝑥𝑥»

«lo apuntado en (iii) se puede generalizar para cualquier número


(iv) que pertenezca a los dos últimos intervalos»

Como ejercicio, el lector debe elaborar el estudio tabular «hacer la tabla dando a «x» los
valores tomados en los ejemplos anteriores» y el estudio grafico «ubicar los pares (x, y) de la
tabla en el plano cartesiano y trazar la recta que contiene a esos puntos», para cada uno de los
casos indicados en la anterior figura, a fin de constatar lo indicado en (i) y (iii)

Ejemplo 3. Siendo 𝑓𝑓1 (𝑥𝑥) = 𝑚𝑚; 𝑓𝑓2 (𝑥𝑥) = 𝑥𝑥 y 𝑓𝑓3 (𝑥𝑥) = 𝑛𝑛, la operación combinada (𝑓𝑓1 ∙ 𝑓𝑓2 + 𝑓𝑓3 )(𝑥𝑥)
está definida, debido a que 𝐷𝐷𝐷𝐷𝐷𝐷(𝑓𝑓1 ) ∩ 𝐷𝐷𝐷𝐷𝐷𝐷(𝑓𝑓2 ) ∩ 𝐷𝐷𝐷𝐷𝐷𝐷(𝑓𝑓3 ) = ℝ ∩ ℝ ∩ ℝ = ℝ. En consecuencia
(𝑓𝑓1 ∙ 𝑓𝑓2 + 𝑓𝑓3 )(𝑥𝑥) = 𝑓𝑓1 (𝑥𝑥) ∙ 𝑓𝑓2 (𝑥𝑥) + 𝑓𝑓3 (𝑥𝑥) = m𝑥𝑥 + n; además si 𝑓𝑓(𝑥𝑥) = 𝑓𝑓1 (𝑥𝑥) ∙ 𝑓𝑓2 (𝑥𝑥) + 𝑓𝑓3 (𝑥𝑥) se tiene
«𝒇𝒇(𝒙𝒙) = 𝐦𝐦𝒙𝒙 + 𝐧𝐧» que es la función afín, se genera como un álgebra de funciones elementales

Función afín 𝑓𝑓(𝑥𝑥) = 𝑚𝑚𝑚𝑚 + 𝑛𝑛 «a la cantidad variable «𝑥𝑥» la multiplica por «m» y le suma «n»; m y n constantes»

Ejemplo 4. Al realizar el estudio completo de la función afín 𝑓𝑓(𝑥𝑥) = 2𝑥𝑥 + 1 se tiene:

Dado que el 𝐷𝐷𝐷𝐷𝐷𝐷(𝑓𝑓) = ℝ, tal como se demostró en el ejemplo 3, se puede tomar una
1 1
muestra de números reales, digamos −1, − , 0, , 1 para construir un subconjunto finito de su
2 2

grafo, elaborar el modo de representación tabular de la función y desde este realizar su dibujo
o modo de representación grafico, tal como se indica a continuación:

CLEMENTE MORENO (MATERIAL DIDÁCTICO EN VALIDACIÓN 10


La evaluación de la función «cualquiera que ella sea», es su valor numérico y consiste en
sustituir la variable «𝑥𝑥» por el número que se considere y luego efectuar las operaciones en ella
indicadas, lo cual genera el par (𝑥𝑥, 𝑦𝑦) del grafo y punto en plano para su dibujo. Aquí se tiene:

(i) «se evalúo la función en «−1», efectuando las operaciones»


𝑓𝑓(−1) = 2(−1) + 1 implica 𝑓𝑓(−1) = −1
1 1 1
𝑓𝑓 �− � = 2 �− � + 1 implica 𝑓𝑓 �− � = 0 (ii) 1
«se evalúo la función en «− 2», efectuando las operaciones»
2 2 2

𝑓𝑓(0) = 2(0) + 1 implica 𝑓𝑓(0) = 1 (iii)


«se evalúo la función en «0», efectuando las operaciones»
1
1 1 1 «se evalúo la función en «2», efectuando las operaciones»
𝑓𝑓 � � = 2 � � + 1 implica 𝑓𝑓 � � = 2 (iv)
2 2 2
«se evalúo la función en «1», efectuando las operaciones»
𝑓𝑓(1) = 2(1) + 1 implica 𝑓𝑓(1) = 3 (v)

1 1
Los cálculos se reúnen en el 𝑔𝑔𝑔𝑔𝑔𝑔(𝑓𝑓) = �… (−1, −1), . . �− , 0� , . . (0,1), . . � , 2� , . . (1,3) … �,
2 2

en adelante se colocará directamente en la tabla correspondiente al estudio tabular, es decir:

1 1 «la tabla es la forma usual de escribir «en los libros» el


𝑥𝑥 −1 − 0 1
2 2 (vi) subconjunto finito del grafo de la función. Esta tabla es la
𝑦𝑦 −1 0 1 2 3 síntesis del modo de representación tabular de la función»

La ubicación de estos pares como puntos del plano, unidos con un trazo continuó «debido
a que el 𝐷𝐷𝐷𝐷𝐷𝐷(𝑓𝑓) = ℝ», corresponde a la gráfica de la función o modo su representación gráfico,
tal como se indica a continuación:

(vii) «observe que la gráfica de 𝑓𝑓(𝑥𝑥) = 2𝑥𝑥 + 1 es como la de 𝑦𝑦 = 2𝑥𝑥, del


+∞ 𝑦𝑦 1
ejemplo 2, pero en este caso se movilizó 2 a la izquierda de 0, es decir
3 ∙ 1
− 2 y 1 hacia el norte, las cuales corresponden a los cortes de la recta
con los ejes rectangulares»
2 ∙ (viii) «para calcular los cortes con los ejes se cambia «𝑦𝑦» por «𝑓𝑓(𝑥𝑥)», es
1 ∙ decir y = 2x + 1; así si x = 0 se tiene y = 2 ∙ 0 + 1 o bien y = 1,
resultando el punto P1 = (0,1); si 𝑦𝑦 = 0 resulta 2x + 1 = 0 de cuya
𝑥𝑥
∙1
1 1
solución se tiene x = − 2 dando el punto P2 = �− 2 , 0�»
−1 1 +∞
−∞ − 1
2 2
∙ −1 (ix) «los símbolos −∞ y +∞ no son números, en matemática se utiliza para
indicar que el conjunto de los números reales ℝ, viene desde un
−∞ número muy pequeño y llega hasta un número muy grande, es decir
que el intervalo (−∞, +∞) = ℝ»

CLEMENTE MORENO (MATERIAL DIDÁCTICO EN VALIDACIÓN 11


COMENTARIOS

y−1 1 1
(i) Si en 𝑦𝑦 = 2𝑥𝑥 + 1 se despeja «x» se tiene y − 1 = 2x o bien x = es decir x = y − y al
2 2 2
1 1
intercambiar los papeles de «x» e «y» resulta y = x − que es la función inversa y se
2 2
1 1
escribe 𝑓𝑓 −1 (𝑥𝑥) = 𝑥𝑥 − que es otra función afín, luego 𝐷𝐷𝐷𝐷𝐷𝐷(𝑓𝑓 −1 ) = ℝ, el cual coincide con
2 2

el 𝑅𝑅𝑅𝑅𝑅𝑅(𝑓𝑓). En consecuencia, el 𝑅𝑅𝑅𝑅𝑅𝑅(𝑓𝑓) = ℝ


(ii) Dado que el dominio y el rango de la función afín, es el conjunto de los números reales ℝ y
que la gráfica de la función afín es una recta y la recta se puede dibujar con sólo dos puntos,
digamos por ejemplo los puntos de corte con los ejes, la tabla adjunta genera esos puntos
n
x 0 −
m
𝑦𝑦 n 0

(iii) La ecuación 𝑦𝑦 = f(x) = mx + n es la forma explícita de la recta, en este caso la pendiente o


inclinación de la recta es igual al coeficiente «m» de «x». En el ejemplo 4, la pendiente de la
recta es m = 2

Ejercicio N° 𝟐𝟐
2 3
Dadas 𝑓𝑓(𝑥𝑥) = ; 𝑔𝑔(𝑥𝑥) = 𝑥𝑥; ℎ(𝑥𝑥) = − ; 𝑙𝑙(𝑥𝑥) = 2; 𝑚𝑚(𝑥𝑥) = −3 encuentre, previa determinación de
5 4

los dominios, las siguientes funciones. Luego realice su estudio tabular y dibuje su gráfica

1) (𝑓𝑓 ∙ 𝑔𝑔)(𝑥𝑥); 2) (𝑚𝑚 ∙ 𝑔𝑔)(𝑥𝑥); 3) (ℎ ∙ 𝑔𝑔)(𝑥𝑥); 4) (𝑙𝑙 ∙ 𝑔𝑔 + 𝑚𝑚)(𝑥𝑥); 5) (𝑓𝑓 ∙ 𝑔𝑔 − ℎ)(𝑥𝑥);

Nota. (𝑓𝑓 ∙ 𝑔𝑔)(𝑥𝑥) = 𝑓𝑓(𝑥𝑥) ∙ 𝑔𝑔(𝑥𝑥) y (𝑙𝑙 ∙ 𝑔𝑔 + 𝑚𝑚)(𝑥𝑥) = 𝑙𝑙(𝑥𝑥) ∙ 𝑔𝑔(𝑥𝑥) + 𝑚𝑚(𝑥𝑥)

Siendo 𝑓𝑓(𝑥𝑥) = 1 − 3𝑥𝑥, indique ¿cuáles de los pares siguientes están en el grafo de la función?

1 1
6) (4, 1); 7) (−1, 4); 8) �− , 2�; 9) (3, 2); 10) (1, −2); 11) � , 0�; 12) (0, 2)
3 3

CLEMENTE MORENO (MATERIAL DIDÁCTICO EN VALIDACIÓN 12


1
Siendo 𝑔𝑔(𝑥𝑥) = 𝑥𝑥 + 2 determine el valor de «𝑔𝑔» en cada uno de los casos siguientes
3

13) 𝑔𝑔(−2); 14) 𝑔𝑔(ℎ); 15) 𝑔𝑔(ℎ + 1); 16) 𝑔𝑔(ℎ) + 1; 17) 𝑔𝑔(ℎ + 2); 18) 𝑔𝑔(ℎ − 1) + 2𝑔𝑔(ℎ);

Realice el estudio tabular y gráfico de las siguientes funciones afines

19) 𝑓𝑓: ℝ → ℝ⁄𝑓𝑓(𝑥𝑥) = 3𝑥𝑥 − 2; 20) 𝑓𝑓: (−1,2) → 𝑅𝑅 ⁄ 𝑓𝑓(𝑥𝑥) = 3𝑥𝑥 − 2;

22) 𝑔𝑔: ℝ → ℝ⁄𝑔𝑔(𝑥𝑥) = −2𝑥𝑥 + 3; 23) 𝑔𝑔: (−1, 2) → (−7,7)⁄𝑔𝑔(𝑥𝑥) = −2𝑥𝑥 + 3;


1 3 𝑥𝑥
24) ℎ: ℝ → ℝ⁄ℎ(𝑥𝑥) = 𝑥𝑥 + ; 24) ℎ: ℝ+ → ℝ+ ⁄ℎ(𝑥𝑥) = − + 3
3 4 2

25) El espacio recorrido por una partícula en movimiento rectilíneo uniforme se da por la
ecuación 𝑆𝑆 = 𝑠𝑠0 + 𝑣𝑣. 𝑡𝑡, dibuje la gráfica de la trayectoria de la partícula para 𝑠𝑠0 = 2 y 𝑣𝑣 = 2.

26) Siendo 𝑣𝑣𝑓𝑓 = 𝑣𝑣0 + 𝑎𝑎. 𝑡𝑡, la velocidad final de una partícula en movimiento, dibuje la gráfica
1
de 𝑣𝑣𝑓𝑓 para 𝑣𝑣0 = 2 y 𝑎𝑎 = .
3

Ecuación de la recta

Si 𝑦𝑦 = 𝑓𝑓(𝑥𝑥) con 𝑓𝑓(𝑥𝑥) = 𝑚𝑚𝑚𝑚 + 𝑛𝑛 entonces 𝑦𝑦 = 𝑚𝑚𝑚𝑚 + 𝑛𝑛 (1) es


𝑦𝑦
la ecuación de la recta con pendiente «𝑚𝑚» y ordenada inicial 𝑙𝑙
P2 = (x2 , y2 )
«𝑛𝑛». La pendiente «𝑚𝑚» es el cociente de diferencias entre las
y2 − y1
ordenadas y las abscisas de los puntos P1 = (𝑥𝑥1 , 𝑦𝑦1 ) y
P1 = (x1 , y1 )
𝑦𝑦2 − 𝑦𝑦1
P2 = (𝑥𝑥2 , 𝑦𝑦2 ) de la recta; así m = (∗) con 𝑥𝑥1 ≠ 𝑥𝑥2 x 2 − x1
𝑥𝑥2 − 𝑥𝑥1
𝑥𝑥
Además, si P = (𝑥𝑥, 𝑦𝑦) es un punto cualquiera de «𝑙𝑙», la igualdad
𝑦𝑦2 − 𝑦𝑦1
𝑦𝑦 − 𝑦𝑦1 = (𝑥𝑥 − 𝑥𝑥1 ) (2) equivale a la identidad (1)
𝑥𝑥2 − 𝑥𝑥1

Al sustituir (∗) en la identidad (2) se tiene:


𝑦𝑦 − 𝑦𝑦1 = 𝑚𝑚(𝑥𝑥 − 𝑥𝑥1 ) (3)

CLEMENTE MORENO (MATERIAL DIDÁCTICO EN VALIDACIÓN 13


Tipos de pendientes

Pendiente negativa Pendiente nula Pendiente positiva

𝑦𝑦 𝑦𝑦 𝑦𝑦
𝑙𝑙 𝑙𝑙
𝑙𝑙

𝑥𝑥 𝛼𝛼
𝛼𝛼
𝑥𝑥
𝑥𝑥

𝑚𝑚 < 0 𝑚𝑚 = 0 𝑚𝑚 > 0

COMENTARIO

Tanto la identidad (2) como la identidad (3) se conocen como la fórmula punto pendiente
para hallar la ecuación de la recta en el plano. Para su aplicación, con la identidad (∗) se
determina la pendiente de la recta si se conocen dos de sus puntos, luego a partir de la identidad
(2) se puede hallar la ecuación que la define; con la identidad (3), se halla la ecuación de la
recta, si se conoce su pendiente y uno de sus puntos

Ejemplo 1. Hallar la ecuación de la recta que pasa por los puntos P1 = (−2, 5) y P2 = (3, −4)
−4−5 9 (i) «en la identidad (∗) se sustituyeron los valores y ejecutaron operaciones»
𝑚𝑚 = implica 𝑚𝑚 = −
3−(−2) 5

5−(−4) 9 (ii) «en la identidad (∗) se sustituyeron los valores y ejecutaron operaciones»
𝑚𝑚 = implica 𝑚𝑚 = −
−2−3 5

Lo apuntado en (i) y (ii) indica que no importa el punto que se elija de primero para hallar
la pendiente «𝑚𝑚», sino la resta entre ordenadas divida entre la resta de sus abscisas.
Determinada la pendiente, se puede aplicar la identidad (3), como se indica:
9 9
𝑦𝑦 − 5 = − �𝑥𝑥 − (−2)� implica 𝑦𝑦 − 5 = − (𝑥𝑥 + 2) «se eligió P1 , sustituyo en (3) y se operó signos»
5 5
9 18 9 18
Así, 𝑦𝑦 − 5 = − 𝑥𝑥 − o bien 𝑦𝑦 = − 𝑥𝑥 − +5 «se distribuyó en 2do miembro y se despejo a 𝑦𝑦»
5 5 5 5
9 7 −9𝑥𝑥 + 7
Luego, 𝑦𝑦 = − 𝑥𝑥 + o bien 𝑦𝑦 = «es la ecuación de la recta que pasa por los puntos dados»
5 5 5

CLEMENTE MORENO (MATERIAL DIDÁCTICO EN VALIDACIÓN 14


9
A la misma ecuación se llega, si luego de determinada la pendiente 𝑚𝑚 = − , en lugar
5

de haber elegido a P1 = (−2, 5) se hubiese seleccionado a P2 = (3, −4). Este caso, se deja de
ejercicio al lector

Ejemplo 2. Halle la ecuación de la recta que pasa por (5, 15) y tiene una pendiente de −3

𝑦𝑦 − 𝑦𝑦1 = 𝑚𝑚(𝑥𝑥 − 𝑥𝑥1 ) implica 𝑦𝑦 − 15 = −3(𝑥𝑥 − 5) «en (3) se sustituyó el punto y la pendiente dados»

Y, 𝑦𝑦 − 15 = −3𝑥𝑥 + 15 «en 2do lado de la igualdad se aplicó propiedad distributiva»

Luego, 𝑦𝑦 = −3𝑥𝑥 + 15 + 15 o bien 𝑦𝑦 = −3𝑥𝑥 + 30 «se despejó a «𝑦𝑦» y se efectúo la suma»

Forma particular de la ecuación de la recta


De la ecuación punto pendiente dada en (3) que se deriva de (2), se puede escribir la
ecuación de manera principal, es decir

𝑦𝑦 − 𝑦𝑦1 = 𝑚𝑚(𝑥𝑥 − 𝑥𝑥1 ) implica 𝑦𝑦 − 𝑦𝑦1 = 𝑚𝑚𝑚𝑚 − 𝑚𝑚𝑥𝑥1 «en 2do miembro se aplicó la propiedad distributiva»

Y, 𝑦𝑦 = 𝑚𝑚𝑚𝑚 − 𝑚𝑚𝑥𝑥1 + 𝑦𝑦1 «se despejo a 𝑦𝑦»

Luego 𝑦𝑦 = 𝑚𝑚𝑚𝑚 + 𝑛𝑛 donde 𝑛𝑛 = −𝑚𝑚𝑥𝑥1 + 𝑦𝑦1 «la operación entre las constantes 𝑚𝑚, 𝑥𝑥1 y 𝑦𝑦1 es la constante 𝑛𝑛»

Esta forma explícita « 𝑦𝑦 = 𝑚𝑚𝑚𝑚 + 𝑛𝑛 » es una de las más representativas de la gráfica, pues
al tenerla se puede apreciar si la pendiente de la recta, es negativa, nula o positiva y con ella
el tipo de recta a representar en el plano, mientras que el coeficiente de posición «𝑛𝑛», indica el
punto donde la recta intersecta al eje «𝑦𝑦»

Además, si 𝑚𝑚 = 0 entonces y = mx + n se reduce a «y = n»



que es la función constante estudiada arriba y cuya representación
y=n
gráfica es una recta paralela al eje de las abscisas, indicando que es
una recta de pendiente nula o «0» ℝ
En efecto, P1 = (−2, n) y P2 = (3, n) son puntos distintos de la
𝑛𝑛 − 𝑛𝑛 0
recta «y = f(x) = n», luego 𝑚𝑚 = así 𝑚𝑚 = o bien 𝑚𝑚 = 0
3−(−2) 5

CLEMENTE MORENO (MATERIAL DIDÁCTICO EN VALIDACIÓN 15


Si 𝑦𝑦 = f(x) = 0 entonces la ecuación 0 = mx + n se escribe
𝑛𝑛 m ℝ
como x = − , pero si con k = − se tiene «x = k» que es una
𝑚𝑚 n

recta paralela al eje de las ordenadas, la cual no representa a una


función, pues existirían infinitas parejas en el grafo que tienen igual ℝ

la primera componente, en particular P1 = (k, −3) y P2 = (k, 3) son del x=k


grafo y sus primeras componentes son iguales

Por otra parte, si los dos miembros de la forma principal o explicita «𝑦𝑦 = 𝑚𝑚𝑚𝑚 + 𝑛𝑛» de la
recta se reúnen en un lado dejando «0» en el otro lado, se llega a la forma general de la recta
en el plano, es decir:
Ax + By + C = 0 (4) con A, B y C números reales

Así, al tener la ecuación en forma principal, basta con igualarla a «0» para llegar a la
ecuación general. De manera inversa, al tener la ecuación en forma general, basta despejar a
«𝑦𝑦» para llegar a la forma particular

3
Ejemplo 1. Para llevar a la forma general, la ecuación 𝑦𝑦 = − 𝑥𝑥 + 1 se tiene:
4

3 −3𝑥𝑥 + 4
𝑦𝑦 = − 𝑥𝑥 + 1 implica 𝑦𝑦 = «se efectúo la suma de fracciones del 2do lado de la igualdad»
4 4

Así, 4𝑦𝑦 = −3𝑥𝑥 + 4 o bien 3𝑥𝑥 + 4𝑦𝑦 − 4 = 0 «se suprimió denominador y reunió en 1er lado los términos»

Inversamente, para llevar la forma general 3𝑥𝑥 + 4𝑦𝑦 − 4 = 0 a la forma particular se tiene:
3𝑥𝑥 + 4𝑦𝑦 − 4 = 0 implica 4𝑦𝑦 = −3𝑥𝑥 + 4 «se dejó a «y» con su coeficiente en 1er lado de la igualdad»

−3𝑥𝑥 + 4 3
Luego, 𝑦𝑦 = o bien 𝑦𝑦 = − 𝑥𝑥 + 1 «se dividió entre 4, se separaron las fracciones y se simplificó»
4 4

Finalmente, se dice que dos rectas son paralelas si tienen la misma pendiente, es decir
𝐦𝐦𝟏𝟏 = 𝐦𝐦𝟐𝟐 ; serán perpendiculares si el producto entre las pendientes de las recta es igual a −1,
1
es decir, m1 ∙ m2 = −1 o bien m1 = − ; y, en caso de no ser paralelas, ni perpendiculares
m2

son rectas secantes

CLEMENTE MORENO (MATERIAL DIDÁCTICO EN VALIDACIÓN 16


Rectas paralelas Rectas perpendiculares Rectas secantes

𝑙𝑙 𝑙𝑙
𝑙𝑙

𝑝𝑝
𝑝𝑝
𝑝𝑝

Ejemplo 2. Las rectas 2𝑥𝑥 − 𝑦𝑦 + 3 = 0 y 2𝑦𝑦 + 𝑥𝑥 − 2 = 0 son perpendiculares, debido a que:

2𝑥𝑥 − 𝑦𝑦 + 3 = 0 implica 𝑦𝑦 = 2𝑥𝑥 − 3. Así 𝑚𝑚1 = 2 «la ecuación se llevó a forma principal y vio su pendiente»

𝑥𝑥 1
2𝑦𝑦 + 𝑥𝑥 − 2 = 0 implica 𝑦𝑦 = − + 1. Así 𝑚𝑚2 = − «la ecuación se llevó a forma principal y vio su pendiente»
2 2
1
𝑚𝑚1 ∙ 𝑚𝑚2 = 2 ∙ (− ) implica 𝑚𝑚1 ∙ 𝑚𝑚2 = −1 «las rectas son perpendiculares, pues m1 ∙ m2 = −1»
2

Ejercicio N° 𝟑𝟑

En los ejercicios del 1 al 6, halle la ecuación de la recta que pasa por los puntos dados. Grafique
la recta a través de los puntos

1) (3, −7) y (1, 0); 2) (−4, −1) y (1, −1); 3) (5,2) y (4, −3);
2 3 3 2 2 3 2 3
4) �− , 4� y �−1, − �; 5) � , − � y �− , �; 6) �−3, � y � , −4�;
3 5 4 5 3 7 7 5

7) ¿Cuál es la pendiente de la recta que pasa por los puntos A(−1, −2) y B(5,2)?
8) ¿Cuál es la pendiente entre los puntos (−2,5) y (3, −2)?

En los problemas 9 al 15, calcula la pendiente y las intersecciones con los ejes «𝑥𝑥» e
«𝑦𝑦» de la recta dada en cada caso

9) 3𝑥𝑥 − 4𝑦𝑦 + 12 = 0; 1 11) 2𝑥𝑥 − 3𝑦𝑦 = 9;


10) 𝑥𝑥 − 3𝑦𝑦 = 3;
2

12) −4𝑥𝑥 − 2𝑦𝑦 + 6 = 0; 13) 2𝑥𝑥 + 5𝑦𝑦 − 8 = 0; 2


14) 𝑦𝑦 + 𝑥𝑥 = 1;
3

CLEMENTE MORENO (MATERIAL DIDÁCTICO EN VALIDACIÓN 17


Transforme a la forma general de la recta, las ecuaciones explicitas de las rectas siguientes:

15) 𝑦𝑦 = 10𝑥𝑥 − 7; 16) 𝑦𝑦 = −3𝑥𝑥 + 19 2 3 2


17) 𝑦𝑦 = − 𝑥𝑥 + 2; 18) 𝑦𝑦 = 𝑥𝑥 − ;
3 4 3

Escriba en forma principal, las ecuaciones generales de las rectas dadas en cada caso:

19) 20𝑥𝑥 + 4𝑦𝑦 + 18 = 0; 20) 3𝑥𝑥 + 10𝑦𝑦 + 10 = 0; 21) 11𝑥𝑥 + 3𝑦𝑦 − 39 = 0;


22) −8𝑥𝑥 − 6𝑦𝑦 − 12 = 0; 23) −16𝑥𝑥 − 4𝑦𝑦 + 22 = 0; 24) −2x + 8y − 10 = 0;

En cada caso, diga si los puntos indicados, pertenecen a la recta dada

25) 6𝑥𝑥 + 2𝑦𝑦 + 2 = 0 y P( 2 , −6 ); 26) 5𝑥𝑥 − 2𝑦𝑦 + 2 = 0 y P(0, 1);

27) 𝑥𝑥 + 3𝑦𝑦 + 6 = 0 y P(3, −3); 28) −2𝑥𝑥 + 3𝑦𝑦 − 1 = 0 y P(1,1)

29) y = 3x + 9 y P(3, 2); 1


30) 𝑦𝑦 = − 𝑥𝑥 − 7 y P(2, −1)
3

Halle la ecuación de la recta que pasa por los puntos indicados en cada caso
31) P(3, 6) y Q( 1, 4); 32) Q(3, 6) y R ( 1, 2); 33) P(0, 2) y M(4, 1);
2 2 3 2 1 3 3 2 2
34) A � , −4� y B �−2, �; 35) C � , � y D �− , − � 36) E �−2, � y F � , − �;
5 3 4 3 2 5 5 3 7

¿Qué relación «paralela, perpendicular o secante» tiene la recta 3𝑥𝑥 + 4𝑦𝑦 − 2 = 0 con cada una
de las rectas siguientes?

37) 8𝑥𝑥 − 6𝑦𝑦 + 5 = 0; 38) 9𝑥𝑥 + 12𝑦𝑦 + 7 = 0; 39) 3𝑥𝑥 + 𝑦𝑦 − 4 = 0;

40) 12𝑥𝑥 − 9𝑦𝑦 + 2 = 0; 41) 2𝑥𝑥 + 𝑦𝑦 − 6 = 0; 42) 2𝑥𝑥 + 5𝑦𝑦 − 3 = 0;

En lo que sigue, encuentre la ecuación de la recta indicada en cada caso:

43) Que pasa por (5, 15) y tiene una pendiente de −3

44) Que pasa por el punto (6, 4) y es paralela al eje de las «𝑥𝑥»

45) Que pasa por el punto (−1, 2) y es paralela a la recta que une los puntos (20, 50) y
(100, 400)

46) Que pasa por el punto (4, −3) y es paralela a la recta de ecuación 4𝑥𝑥 − 5𝑦𝑦 + 1 = 0

CLEMENTE MORENO (MATERIAL DIDÁCTICO EN VALIDACIÓN 18


47) Que pasa por el punto (3, −2) y es perpendicular a la que pasa por los puntos (−1, −2) y
(3, 7)

48) Que pasa por el punto (4, 3) y es paralela a la que pasa por los puntos (0, −3) y (6,1)

49) Que pasa por el punto (5, 15) y es paralela a la recta de ecuación 𝑦𝑦 = 𝑥𝑥 + 25
4 − 𝑘𝑘
50) Dadas las rectas 𝑦𝑦 = 3𝑥𝑥 + 2 y 𝑦𝑦 = � � 𝑥𝑥 − 5, halle el valor de k para que las rectas
𝑘𝑘
sean perpendiculares

La función cuadrática

Sean las funciones 𝑓𝑓1 (𝑥𝑥) = 𝑎𝑎, 𝑓𝑓2 (𝑥𝑥) = 𝑥𝑥, 𝑓𝑓3 (𝑥𝑥) = 𝑏𝑏 y 𝑓𝑓4 (𝑥𝑥) = 𝑐𝑐 «funciones constantes y la
función idéntica estudiadas arriba» con 𝐷𝐷𝐷𝐷𝐷𝐷(𝑓𝑓1 ) = ℝ, 𝐷𝐷𝐷𝐷𝐷𝐷(𝑓𝑓2 ) = ℝ, 𝐷𝐷𝐷𝐷𝐷𝐷(𝑓𝑓3 ) = ℝ y
𝐷𝐷𝐷𝐷𝐷𝐷(𝑓𝑓4 ) = ℝ, respectivamente. En estas condiciones, la operación (𝑓𝑓1 ∙ 𝑓𝑓2 ∙ 𝑓𝑓2 + 𝑓𝑓3 ∙ 𝑓𝑓2 + 𝑓𝑓4 )(𝑥𝑥)
está definida ya que 𝐷𝐷𝐷𝐷𝐷𝐷(𝑓𝑓1 ) ∩ 𝐷𝐷𝐷𝐷𝐷𝐷(𝑓𝑓2 ) ∩ 𝐷𝐷𝐷𝐷𝐷𝐷(𝑓𝑓3 ) ∩ 𝐷𝐷𝐷𝐷𝐷𝐷(𝑓𝑓4 ) = ℝ ∩ ℝ ∩ ℝ ∩ ℝ = ℝ, decir que:

(𝑓𝑓1 ∙ 𝑓𝑓2 ∙ 𝑓𝑓2 + 𝑓𝑓3 ∙ 𝑓𝑓2 + 𝑓𝑓4 )(𝑥𝑥) = 𝑓𝑓1 (𝑥𝑥) ∙ 𝑓𝑓2 (𝑥𝑥) ∙ 𝑓𝑓2 (𝑥𝑥) + 𝑓𝑓3 (𝑥𝑥) ∙ 𝑓𝑓2 (𝑥𝑥) + 𝑓𝑓4 (𝑥𝑥) «definición de operaciones»

= 𝑎𝑎 ∙ 𝑥𝑥 ∙ 𝑥𝑥 + 𝑏𝑏 ∙ 𝑥𝑥 + 𝑐𝑐 «sustituyendo el valor de las funciones»

= 𝑎𝑎𝑥𝑥 2 + 𝑏𝑏𝑏𝑏 + 𝑐𝑐 «resolviendo la potencia y operando»

Luego, 𝑓𝑓(𝑥𝑥) = 𝑎𝑎𝑥𝑥 2 + 𝑏𝑏𝑏𝑏 + 𝑐𝑐 «siendo 𝑓𝑓(𝑥𝑥) = (𝑓𝑓1 ∙ 𝑓𝑓2 ∙ 𝑓𝑓2 + 𝑓𝑓3 ∙ 𝑓𝑓2 + 𝑓𝑓4 )(𝑥𝑥), el álgebra de funciones es otra función»

Definición
Desde el punto de vista formal, una función «𝑓𝑓» de un conjunto A en otro conjunto B,
denotada 𝑓𝑓: A ⟶ B, con A y B subconjuntos del conjunto de los números reales ℝ «A, B ⊆ ℝ»
es una función cuadrática, si para cualquier 𝑥𝑥 ∈ A, 𝑓𝑓(𝑥𝑥) = 𝑎𝑎𝑥𝑥 2 + 𝑏𝑏𝑏𝑏 + 𝑐𝑐 (∗) con 𝑎𝑎, 𝑏𝑏, 𝑐𝑐 ∈ ℝ y
𝑎𝑎 ≠ 0 «si 𝑎𝑎 = 0, la identidad dada en (∗) es la función afín»

CLEMENTE MORENO (MATERIAL DIDÁCTICO EN VALIDACIÓN 19


COMENTARIO

Algunas veces, en lugar «𝑓𝑓(𝑥𝑥)» se escribe «𝑦𝑦». Así, (∗) también se escribe 𝑦𝑦 = 𝑎𝑎𝑥𝑥 2 𝑏𝑏𝑏𝑏 + 𝑐𝑐
El dominio de la función es el conjunto de los números reales «𝐷𝐷𝐷𝐷𝐷𝐷(𝑓𝑓) = ℝ» y el recorrido
o rango de la función es un subconjunto propio de ℝ «Rgo(𝑓𝑓) ⊂ ℝ»
El grafo de la función cuadrática es el conjunto 𝐺𝐺𝐺𝐺𝐺𝐺(𝑓𝑓) = {(𝑥𝑥, 𝑦𝑦) ∈ ℝ2 ⁄ 𝑦𝑦 = 𝑎𝑎𝑥𝑥 2 + 𝑏𝑏𝑏𝑏 + 𝑐𝑐},
mientras que su gráfica corresponde a una parábola

Representación gráfica de la parábola

(i) Análisis de los parámetros 𝒂𝒂, 𝒃𝒃 y 𝒄𝒄 en la función cuadrática


El estudio de la función 𝑦𝑦 = 𝑓𝑓(𝑥𝑥) = 𝑎𝑎𝑥𝑥 2 + 𝑏𝑏𝑏𝑏 + 𝑐𝑐, depende de los parámetros 𝑎𝑎, 𝑏𝑏, 𝑐𝑐 y
𝑎𝑎 ≠ 0, que son los coeficientes de «𝑥𝑥 2 », «𝑥𝑥» y término independiente, respectivamente
Así, si 𝑎𝑎 > 0 y 𝑏𝑏 = 𝑐𝑐 = 0, la parábola 𝑦𝑦 = 𝑓𝑓(𝑥𝑥) = 𝑎𝑎𝑥𝑥 2 es cóncava hacia arriba, en este
caso el vértice es el origen de coordenadas (0,0) y corresponde al punto más bajo o mínimo
absoluto; si a < 0 con 𝑏𝑏 = 𝑐𝑐 = 0 entonces la parábola es cóncava hacia abajo y el vértice, (0,0)
corresponde al punto más alto o máximo absoluto, tal como se ilustra en las figuras

𝑦𝑦 = 𝑎𝑎𝑥𝑥 2 con 𝑎𝑎 > 0 𝑦𝑦 = 𝑎𝑎𝑥𝑥 2 con 𝑎𝑎 < 0

CLEMENTE MORENO (MATERIAL DIDÁCTICO EN VALIDACIÓN 20


COMENTARIO

Si miras con atención el dibujo «gráficas» de las parábolas 𝑦𝑦 = 𝑎𝑎𝑥𝑥 2 o bien 𝑓𝑓(𝑥𝑥) = 𝑎𝑎𝑥𝑥 2 ,
mostradas arriba, puedes observar que las parábolas se van cerrando a medida que el
coeficiente «𝑎𝑎» crece en valor absoluto. Así, la parábola coloreada de rojo que representa a la
función 𝑦𝑦 = 𝑥𝑥 2 o bien 𝑓𝑓(𝑥𝑥) = 𝑥𝑥 2 donde 𝑎𝑎 = 1 y también a la función 𝑦𝑦 = −𝑥𝑥 2 o 𝑓𝑓(𝑥𝑥) = −𝑥𝑥 2
donde 𝑎𝑎 = −1, es la más ancha de todas las curvas y «se aproxima al eje 𝑥𝑥»
La parábola coloreada de verde que representa a la función 𝑦𝑦 = 2𝑥𝑥 2 o bien 𝑓𝑓(𝑥𝑥) = 2𝑥𝑥 2
donde 𝑎𝑎 = 2 y también a la función 𝑦𝑦 = −2𝑥𝑥 2 o bien 𝑓𝑓(𝑥𝑥) = −2𝑥𝑥 2 donde 𝑎𝑎 = −2, se cierra un
poco más que la parábola coloreada de rojo, acercándose al eje «𝑦𝑦»
Asimismo, la parábola de color azul intenso que representa a la función 𝑦𝑦 = 5𝑥𝑥 2 o bien
𝑓𝑓(𝑥𝑥) = 5𝑥𝑥 2 donde 𝑎𝑎 = 5 y también a la función 𝑦𝑦 = −5𝑥𝑥 2 o bien 𝑓𝑓(𝑥𝑥) = −5𝑥𝑥 2 donde 𝑎𝑎 = −5,
se cierra más que la parábola coloreada de verde, acercándose al eje «𝑦𝑦»
Igualmente, la parábola de color violeta que representa a la función 𝑦𝑦 = 10𝑥𝑥 2 o también
𝑓𝑓(𝑥𝑥) = 10𝑥𝑥 2 donde 𝑎𝑎 = 10 y también a la función 𝑦𝑦 = −10𝑥𝑥 2 o bien 𝑓𝑓(𝑥𝑥) = −10𝑥𝑥 2 donde
𝑎𝑎 = −10, se cierra más que la parábola de color azul intenso, acercándose al eje «𝑦𝑦»
En conclusión, el coeficiente de «𝑥𝑥 2 » indica la rapidez con que la curva crece o decrece
con respecto al eje «𝑦𝑦», que en este caso es el eje de simetría de la parábola

Nota. Para trabajar un caso particular de la parábola se procede como se indica en los ejemplos
siguientes

Ejemplo 1. Al dibujar la curva de 𝑦𝑦 = 𝑓𝑓(𝑥𝑥) = 𝑥𝑥 2 , donde 𝑎𝑎 = 1 > 0 se toma un subconjunto del


dominio de f, digamos … −2, −1, 0, 1, 2… que sustituidos en «𝑥𝑥» de 𝑓𝑓(𝑥𝑥) = 𝑥𝑥 2 resulta:

Si 𝑥𝑥 = −2 se tiene 𝑓𝑓(−2) = (−2)2 entonces 𝑦𝑦 = 𝑓𝑓(−2) = 4 «sustituyendo y operando»

Si 𝑥𝑥 = −1 se tiene 𝑓𝑓(−1) = (−1)2 entonces 𝑦𝑦 = 𝑓𝑓(−1) = 1 «sustituyendo y operando»

Si 𝑥𝑥 = 0 se tiene 𝑓𝑓(0) = (0)2 entonces 𝑦𝑦 = 𝑓𝑓(0) = 0 «sustituyendo y operando»

Si 𝑥𝑥 = 1 se tiene 𝑓𝑓(1) = (1)2 entonces 𝑦𝑦 = 𝑓𝑓(1) = 1 «sustituyendo y operando»

Si 𝑥𝑥 = 2 se tiene 𝑓𝑓(2) = (2)2 entonces 𝑦𝑦 = 𝑓𝑓(2) = 4 «sustituyendo y operando»

CLEMENTE MORENO (MATERIAL DIDÁCTICO EN VALIDACIÓN 21


Estos valores pueden apuntarse en una tabla como la siguiente:

𝑥𝑥 −2 −1 0 1 2
𝑦𝑦 4 1 0 1 4
(𝑥𝑥, 𝑦𝑦) (−2,4) (−1,1) (0,0) (1,1) (2,4)

La ubicación de estos puntos en el plano y el


hecho de que el Dom(𝑓𝑓) = ℝ y su Rgo(𝑓𝑓) = [0, +∞),
permite dibujar la curva que representa gráficamente
a la función 𝑦𝑦 = 𝑓𝑓(𝑥𝑥) = 𝑥𝑥 2 , como ilustra la figura

Ejemplo 2. Al dibujar la curva de 𝑦𝑦 = 𝑓𝑓(𝑥𝑥) = −𝑥𝑥 2 , donde 𝑎𝑎 = −1 < 0 se toma un subconjunto


del dominio de 𝑓𝑓, digamos …−2, −1, 0, 1, 2… que sustituidos en «𝑥𝑥» de 𝑓𝑓(𝑥𝑥) = −𝑥𝑥 2 resulta:
Si 𝑥𝑥 = −2 se tiene 𝑓𝑓(−2) = −(−2)2 entonces 𝑦𝑦 = 𝑓𝑓(−2) = −4 «sustituyendo y operando»

Si 𝑥𝑥 = −1 se tiene 𝑓𝑓(−1) = −(−1)2 entonces 𝑦𝑦 = 𝑓𝑓(−1) = −1 «sustituyendo y operando»

Si 𝑥𝑥 = 0 se tiene 𝑓𝑓(0) = −(0)2 entonces 𝑦𝑦 = 𝑓𝑓(0) = 0 «sustituyendo y operando»

Si 𝑥𝑥 = 1 se tiene 𝑓𝑓(1) = −(1)2 entonces 𝑦𝑦 = 𝑓𝑓(1) = −1 «sustituyendo y operando»

Si 𝑥𝑥 = 2 se tiene 𝑓𝑓(2) = −(2)2 entonces 𝑦𝑦 = 𝑓𝑓(2) = −4 «sustituyendo y operando»

Estos valores pueden apuntarse en una tabla como la siguiente:

x −2 −1 0 1 2
y 4 1 0 1 4
(x, y) (−2, −4) (−1, −1) (0,0) (1, −1) (2, −4)

De nuevo, la ubicación de estos puntos en el plano y


el hecho de que el Dom(𝑓𝑓) = ℝ y su Rgo(𝑓𝑓) = (−∞, 0],
permite dibujar la curva que representa gráficamente
a la función 𝑦𝑦 = 𝑓𝑓(𝑥𝑥) = −𝑥𝑥 2 , como ilustra la figura

El parámetro «𝑐𝑐» determina cuanto se desplaza la gráfica de 𝑦𝑦 = 𝑓𝑓(𝑥𝑥) = 𝑎𝑎𝑥𝑥 2 en el eje


vertical «traslación en el rango». Si 𝑐𝑐 > 0, se desplaza «𝑐𝑐» unidades hacia arriba y si 𝑐𝑐 < 0, se

CLEMENTE MORENO (MATERIAL DIDÁCTICO EN VALIDACIÓN 22


desplaza |𝑐𝑐| unidades hacia abajo. Las coordenadas del vértice siempre se encuentran en
V(0, c) como se ilustra en las figuras

𝑦𝑦 = 𝑓𝑓(𝑥𝑥) = 𝑎𝑎𝑥𝑥 2 + 𝑐𝑐 con 𝑐𝑐 > 0 𝑦𝑦 = 𝑓𝑓(𝑥𝑥) = 𝑎𝑎𝑥𝑥 2 + 𝑐𝑐 con 𝑐𝑐 < 0

COMENTARIO

De nuevo, si miras las gráficas de las parábolas 𝑦𝑦 = 𝑎𝑎𝑥𝑥 2 + 𝑐𝑐 o bien 𝑓𝑓(𝑥𝑥) = 𝑎𝑎𝑥𝑥 2 + 𝑐𝑐,
mostradas arriba, puedes observar que las parábolas se trasladan sobre el eje «𝑦𝑦», tantas
unidades como indique el parámetro «𝑐𝑐». Así, la parábola de color rojo que representa a la
función 𝑦𝑦 = 𝑥𝑥 2 + 4 o bien 𝑓𝑓(𝑥𝑥) = 𝑥𝑥 2 + 4 se trasladó 4 unidades hacia arriba con respecto a
la parábola de color azul que representa a la función 𝑦𝑦 = 𝑥𝑥 2 o bien 𝑓𝑓(𝑥𝑥) = 𝑥𝑥 2 ; de igual modo,
la parábola de color rojo que representa a la función 𝑦𝑦 = 𝑥𝑥 2 − 4 o bien 𝑓𝑓(𝑥𝑥) = 𝑥𝑥 2 − 4 se
trasladó 4 unidades hacia abajo con respecto a la parábola de color azul que representa a la
función 𝑦𝑦 = −𝑥𝑥 2 o bien 𝑓𝑓(𝑥𝑥) = −𝑥𝑥 2
En conclusión, el parámetro «𝑐𝑐» es una traslación vertical de la parábola que representa
a la función 𝑦𝑦 = 𝑎𝑎𝑥𝑥 2 o bien 𝑓𝑓(𝑥𝑥) = 𝑎𝑎𝑥𝑥 2

Ejemplo 3 Al dibujar la curva de 𝑦𝑦 = 𝑓𝑓(𝑥𝑥) = −𝑥𝑥 2 + 3 donde 𝑎𝑎 = −1 < 0 y 𝑐𝑐 = 3 se toma un


subconjunto del dominio de 𝑓𝑓, … −2, −1, 0, 1, 2… que sustituidos en 𝑓𝑓(𝑥𝑥) = −𝑥𝑥 2 + 3 resulta:

CLEMENTE MORENO (MATERIAL DIDÁCTICO EN VALIDACIÓN 23


Si 𝑥𝑥 = −2 se tiene 𝑓𝑓(−2) = −(−2)2 + 3 luego 𝑓𝑓(−2) = −1 «sustituyendo y operando»

Si 𝑥𝑥 = −1 se tiene 𝑓𝑓(−1) = −(−1)2 + 3 luego 𝑓𝑓(−1) = 2 «sustituyendo y operando»

Si 𝑥𝑥 = 0 se tiene 𝑓𝑓(0) = −(0)2 + 3 luego 𝑓𝑓(0) = 3 «sustituyendo y operando»

Si 𝑥𝑥 = 1 se tiene 𝑓𝑓(1) = −(1)2 + 3 luego 𝑓𝑓(1) = 2 «sustituyendo y operando»

Si 𝑥𝑥 = 2 se tiene 𝑓𝑓(2) = −(2)2 + 3 luego 𝑓𝑓(1) = −1 «sustituyendo y operando»

Estos valores pueden apuntarse en una tabla como la siguiente:

x −2 −1 0 1 2
y −1 2 3 2 −1
(x, y) (−2, − 1) (−1, 2) (0,3) (1,2) (2, − 1)

De nuevo, la ubicación de estos puntos en el plano


y el hecho de que el Dom(𝑓𝑓) = ℝ y Rgo(𝑓𝑓) = (−∞, 3],
permite dibujar la curva que representa gráficamente a la
función 𝑦𝑦 = 𝑓𝑓(𝑥𝑥) = −𝑥𝑥 2 + 3, como ilustra la figura

Finalmente, para el estudio del comportamiento de la función 𝑦𝑦 = 𝑓𝑓(𝑥𝑥) = 𝑎𝑎𝑥𝑥 2 + 𝑏𝑏𝑏𝑏 + 𝑐𝑐 (1)
se reescribe (1) utilizado la estrategia de completar cuadrados:
𝑏𝑏 2 4𝑎𝑎𝑎𝑎 − 𝑏𝑏2
Así, (1) se escribe 𝑦𝑦 = 𝑓𝑓(𝑥𝑥) = 𝑎𝑎 �𝑥𝑥 + � + (2)
2𝑎𝑎 4𝑎𝑎
𝑏𝑏 4𝑎𝑎𝑎𝑎 − 𝑏𝑏2
Siendo ℎ = − y 𝑘𝑘 = entonces (2) se reescribe como 𝑦𝑦 = 𝑓𝑓(𝑥𝑥) = 𝑎𝑎(𝑥𝑥 − ℎ)2 + 𝑘𝑘
2𝑎𝑎 4𝑎𝑎

o su equivalente 𝑦𝑦 − 𝑘𝑘 = 𝑎𝑎(𝑥𝑥 − ℎ)2 (3)

En (3), el punto (ℎ, 𝑘𝑘) corresponde al vértice de la parábola y la recta vertical «𝑥𝑥 = ℎ» se
denomina eje de simetría. Por ello, el parámetro «𝑏𝑏», determina un desplazamiento horizontal
de |ℎ| unidades, como ilustran las figuras

CLEMENTE MORENO (MATERIAL DIDÁCTICO EN VALIDACIÓN 24


y = f(x) = ax 2 + bx + c con a > 0 y = f(x) = ax 2 + bx + c con a < 0

COMENTARIO

La parábola de color rojo que representa a la curva y = f(x) = a(x − h)2 + k o también
y − k = a(x − h)2 es una traslación compuesta de la parábola de color azul y que representa
a la función y = ax 2 o bien f(x) = ax 2 . Es compuesta debido a que realizó una traslación
horizontal de «h» unidades junto a una traslación vertical de «k» unidades
Con esta traslación el vértice se ubica en el punto (h, k) y la recta vertical x = h, que en
el dibujo se identifica con «y ′ » corresponde al eje de simetría
Nota. Si la hoja donde se dibuja la parábola se dobla por el «eje de simetría», las ramas de
la parábola se superpone una sobre la otra, una es reflexión de la otra; es decir, al dibujar
una de las ramas, la otra se dibuja por reflexión

Cuando k = 0, la traslación ocurre solo en el eje horizontal y el trinomio que corresponde


a la función cuadrática, es un trinomio cuadrado perfecto. Las figuras ilustran el caso
y = f(x) = a(x − p)2 con a > 0 y = f(x) = a(x − p)2 con a < 0

𝑦𝑦
𝑦𝑦

(0,0) (0, p)
𝑥𝑥

(0, −p) (0,0) 𝑥𝑥

CLEMENTE MORENO (MATERIAL DIDÁCTICO EN VALIDACIÓN 25


(ii) Intersección con los ejes y vértice de la parábola

Para hallar la intersección de la parábola con el eje «𝑦𝑦» se hace 𝑥𝑥 = 0, luego este valor se
sustituye en la función, es decir:
𝑦𝑦 = 𝑓𝑓(0) = 𝑎𝑎(0)2 + 𝑏𝑏(0) + 𝑐𝑐, por ello 𝑦𝑦 = 𝑐𝑐
Entonces, la intersección de la parábola con el eje «𝑦𝑦», ocurre en el punto P = (0, 𝑐𝑐)

Para hallar la intersección con el eje «𝑥𝑥» se hace 𝑦𝑦 = 0, luego este valor se sustituye en la
función, es decir:
0 = 𝑎𝑎𝑥𝑥 2 + 𝑏𝑏𝑏𝑏 + 𝑐𝑐 o bien 𝑎𝑎𝑥𝑥 2 + 𝑏𝑏𝑏𝑏 + 𝑐𝑐 = 0, ecuación cuadrática asociada a la parábola, aquí:
Cuando 𝑏𝑏 2 − 4𝑎𝑎𝑎𝑎 > 0, la ecuación tiene dos soluciones reales distintas, luego la parábola
corta al eje «𝑥𝑥» en dos puntos
Cuando 𝑏𝑏 2 − 4𝑎𝑎𝑎𝑎 = 0, la ecuación tiene una solución real, entonces la parábola toca al
eje «x» en un punto
Cuando 𝑏𝑏 2 − 4𝑎𝑎𝑎𝑎 < 0, la ecuación no tiene raíces reales, entonces la parábola no toca
ni cruza el eje «x», la figura ilustra algunos de los casos

𝑦𝑦 𝑦𝑦

(ℎ, 𝑘𝑘) (h, k) 𝑥𝑥


(ℎ, 𝑘𝑘)

(ℎ, 𝑘𝑘)
(ℎ, 𝑘𝑘) (ℎ, 𝑘𝑘)
𝑥𝑥

−𝑏𝑏 − √𝑏𝑏2 − 4𝑎𝑎𝑎𝑎 −𝑏𝑏 + √𝑏𝑏2 − 4𝑎𝑎𝑎𝑎


Por otra parte, dado que , son las raíces de la ecuación
2𝑎𝑎 2𝑎𝑎

−𝑏𝑏 − �𝑏𝑏2 −4𝑎𝑎𝑎𝑎 −𝑏𝑏 + �𝑏𝑏2 − 4𝑎𝑎𝑎𝑎


+ −2𝑏𝑏
cuadrática, su promedio 2𝑎𝑎 2𝑎𝑎
= genera la ordenada del vértice, es decir
2 4𝑎𝑎
b 𝑏𝑏
𝑥𝑥𝑣𝑣 = − , mientras que la abscisa 𝑦𝑦𝑣𝑣 = 𝑓𝑓 �− � corresponde a la evaluación de la función en
2a 2𝑎𝑎
𝑏𝑏
𝑥𝑥𝑣𝑣 = − , luego el vértice de la parábola es:
2𝑎𝑎

b b
𝑣𝑣 = �− , 𝑓𝑓 �− ��
2a 2a

CLEMENTE MORENO (MATERIAL DIDÁCTICO EN VALIDACIÓN 26


Ejemplo 1. Al dibujar la parábola asociada a la función y = f(x) = x 2 + 3x − 5 se tiene:

COMENTARIO

Como el Dominio de la función cuadrática es el conjunto de los números reales ℝ, la


parábola asociada a la función es una curva continua, su dibujo puede realizarse a partir del
vértice y los cortes con los ejes rectangulares «en caso de que existan»

Con respecto al ejemplo que se está considerando, se tiene que:

𝑏𝑏 3 3
𝑥𝑥𝑣𝑣 = − implica 𝑥𝑥𝑣𝑣 = − =− «sustituyendo y operando» 𝑦𝑦
2𝑎𝑎 2∙1 2
8
3 3 2 3 6
𝑦𝑦𝑣𝑣 = 𝑓𝑓 �− � = �− � + 3 �− � − 5 «sustituyendo xv en f(x)»
2 2 2 4
9 9 2
= − −5 «resolviendo potencias y productos»
−8 −6 −4 −2
𝑥𝑥
4 2 2 4 6
29 −2
=− «resolviendo suma de fracciones»
−4
4
3 29 −6
Luego, 𝑉𝑉 = �− , − � «vértice de la parábola» −8
2 4

Si 𝑥𝑥 = 0 se tiene 𝑦𝑦 = 𝑓𝑓(0) = 02 + 3 ∙ 0 − 5 «sustituyendo «0» en f(x)»

Así, 𝑦𝑦 = −5 luego P1 = (0, −5) «punto de intersección de la parábola con el eje «y»»

Si 𝑦𝑦 = 0 se tiene 𝑥𝑥 2 + 3𝑥𝑥 − 5 = 0 «ecuación cuadrática asociada a la parábola»

−3 − √29 −3 + √29
Y, 𝑥𝑥1 = y 𝑥𝑥2 = «soluciones de la ecuación»
2 2
−3 − √29 −3+ √29
Así, P2 = � , 0� y P3 = � , 0� «puntos de intersección de la parábola con el eje «x»»
2 2

El dibujo de la parábola se muestra en la figura ubicada arriba y a la derecha

Ejemplo 2. Inversamente, dados tres puntos, digamos (0,3); (1,4) y (3,0) que pertenecen a la
parábola, se puede determinar la ecuación de la función asociada a dicha parábola, así:

CLEMENTE MORENO (MATERIAL DIDÁCTICO EN VALIDACIÓN 27


Si (0,3) pertenece a 𝑦𝑦 = 𝑎𝑎𝑥𝑥 2 + 𝑏𝑏𝑏𝑏 + 𝑐𝑐 entonces 3 = a ∙ 02 + b ∙ 0 + c «sustituyendo el punto (0, 3)»

Aquí, se genera la ecuación 𝑐𝑐 = 3 (1) «operando»

Si (1,4) pertenece a 𝑦𝑦 = 𝑎𝑎𝑥𝑥 2 + 𝑏𝑏𝑏𝑏 + 𝑐𝑐 entonces 4 = 𝑎𝑎 ∙ 12 + 𝑏𝑏 ∙ 1 + 𝑐𝑐 «sustituyendo el punto (1, 4)»

Aquí, se genera la ecuación 𝑎𝑎 + 𝑏𝑏 + 𝑐𝑐 = 4 (2) «operando»

Si (3,0) pertenece a 𝑦𝑦 = 𝑎𝑎𝑥𝑥 2 + 𝑏𝑏𝑏𝑏 + 𝑐𝑐 entonces 0 = 𝑎𝑎 ∙ 32 + 𝑏𝑏 ∙ 3 + 𝑐𝑐 «sustituyendo el punto (3, 0)»

Aquí, se genera la ecuación 9𝑎𝑎 + 3𝑏𝑏 + 𝑐𝑐 = 0 (3) «operando»

𝑐𝑐 = 3 (1)
Así, � 𝑎𝑎 + 𝑏𝑏 + 𝑐𝑐 = 4 (2) «sistema formado por las 3 ecuaciones»
9𝑎𝑎 + 3𝑏𝑏 + 𝑐𝑐 = 0 (3)
𝑎𝑎 + 𝑏𝑏 = 1 (4)
Y, � «sustituyendo (1) en (2) y (3) y operando»
9𝑎𝑎 + 3𝑏𝑏 = −3 (5)
𝑎𝑎 + 𝑏𝑏 = 1
Es decir, � «dividiendo ecuación (5) entre 3»
3𝑎𝑎 + 𝑏𝑏 = −1
−𝑎𝑎 − 𝑏𝑏 = −1
O bien � «multiplicando por (−1) la 1ra ecuación»
3𝑎𝑎 + 𝑏𝑏 = −1
Así, 2𝑎𝑎 = −2 o bien 𝑎𝑎 = −1 «sumando miembro a miembro y despejando»

Y, −1 + 𝑏𝑏 = 1 implica 𝑏𝑏 = 2 «sustituyendo el valor de « a = −1» en 1ra ecuación y operando»

Por último, 𝑦𝑦 = 𝑓𝑓(𝑥𝑥) = −𝑥𝑥 2 + 2𝑥𝑥 + 3 «sustituyendo « a = −1, b = 2 y c = 3 » en y = ax 2 + bx + c»

Resta dibujar la parábola asociada a la función encontrada y = f(x) = −x 2 + 2x + 3, para ello:

2
𝑥𝑥𝑣𝑣 = − o bien 𝑥𝑥𝑣𝑣 = 1 b
«sustituyendo en xv = − 2a y operando»
2 ∙ (−1)

𝑦𝑦𝑣𝑣 = −(1)2 + 2(1) + 3 «sustituyendo en xv = 1 en la función»

Así, 𝑦𝑦𝑣𝑣 = −1 + 2 + 3 o bien 𝑦𝑦𝑣𝑣 = 4 «operando»

Luego, 𝑉𝑉 = (1, 4) «es el vértice de la parábola»

Si, 𝑥𝑥 = 0 entonces 𝑦𝑦 = −02 + 2(0) + 3 «sustituyendo»

Así, 𝑦𝑦 = 3 luego P1 = (0, 3) «intersección de la parábola con el eje «y»»

Si 𝑦𝑦 = 0 se tiene −x 2 + 2x + 3 = 0 «ecuación de la parábola»

Mientras que, 𝑥𝑥 = −1 y 𝑥𝑥 = 3 «son las soluciones de la ecuación de la parábola»

Y, P2 = (−1, 0) y P3 = (3, 0) «puntos de intersección de la parábola con el eje «x»»

El dibujo de la parábola asociada a 𝑦𝑦 = 𝑓𝑓(𝑥𝑥) = −𝑥𝑥 2 + 2𝑥𝑥 + 3 se muestra en la figura

CLEMENTE MORENO (MATERIAL DIDÁCTICO EN VALIDACIÓN 28


Ejercicio N° 𝟒𝟒

1. Observa la gráfica y asóciala a las funciones dadas en los incisos


(a) (b) (c) (d)

A) 𝑓𝑓(𝑥𝑥) = −𝑥𝑥 2 ______


B) 𝑔𝑔(𝑥𝑥) = 3 − 𝑥𝑥 2 ______
C) ℎ(𝑥𝑥) = −𝑥𝑥 2 − 3 ______
D) 𝐿𝐿(𝑥𝑥) = −2𝑥𝑥 2 ______

2. ¿Cuál de los siguientes gráficos, es el que mejor representa a la función f(x) = x 2 − 5x + 6?

1
3. Considere la parábola 𝑦𝑦 = (𝑥𝑥 − 1)2 . ¿Cuál(es) de las siguientes afirmaciones es (son)
2

verdadera(s)
I) La parábola se abre hacia arriba
II) Su vértice se encuentra en (1,0)
III) Su eje de simetría es 𝑥𝑥 = 1
A) Solo I
B) Solo I y II
C) Solo I y III
D) Solo II y III
E) I, II y III

CLEMENTE MORENO (MATERIAL DIDÁCTICO EN VALIDACIÓN 29


4. El punto que no pertenece a la función 𝑦𝑦 = 𝑓𝑓(𝑥𝑥) = 𝑥𝑥 2 + 2𝑥𝑥 + 1 es:
A) (1, 4)
B) (−1, 0)
C) (0, 1)
D) (2, 9)
E) (1, 1)

5. En la función 𝑦𝑦 = 𝑓𝑓(𝑥𝑥) = 4𝑥𝑥 2 − 4𝑥𝑥 − 3 las coordenadas de su vértice son:


1
A) � , −4�
2
1
B) �− , −4�
2

C) (2, −4)
D) (2, 4)
1
E �− , 4�
2

6. El recorrido o rango de 𝑦𝑦 = 𝑓𝑓(𝑥𝑥) = 4𝑥𝑥 2 − 4𝑥𝑥 − 3 se corresponde con el conjunto


A) [4, +∞[
B) ] − ∞, −4]
C) ] − ∞, 4]
D) ] − 4, +∞[
E) Ninguno de estos

7. En la función 𝑓𝑓(𝑥𝑥) = 𝑥𝑥 2 + 6𝑥𝑥 + 13, el menor valor perteneciente al recorrido es:


A) −2
B) 3
C) −3
D) 4
E) −4

CLEMENTE MORENO (MATERIAL DIDÁCTICO EN VALIDACIÓN 30


8. Dada la parábola de ecuación 𝑦𝑦 = 𝑥𝑥 2 − 2𝑥𝑥 + a, ¿cuál(es) de las siguientes afirmaciones es
(son) correcta(s)?
I) Si 𝑎𝑎 > 1, la parábola intersecta en dos puntos al eje 𝑥𝑥
II) Si 𝑎𝑎 = 1, la parábola intersecta en un solo punto al eje 𝑥𝑥
III) Si 𝑎𝑎 < 1, la parábola no intersecta al eje 𝑥𝑥
A) Solo I
B) Solo II
C) Solo I y II
D) Solo I y III
E) Solo II y III
9. La gráfica de 𝑓𝑓(𝑥𝑥) = 𝑥𝑥 2 − 𝑥𝑥 − 6 intersecta al eje «𝑥𝑥» en:
A) 3 y 2
B) −3 y 2
C) 3 y −2
D) −3 y −2
E) −1 y −6
10. ¿Cuál de las siguientes figuras representa mejor al gráfico de la función f(x) = x 2 − 1?
(A) (B) (C) (D) (E)

11. ¿Cuáles de los siguientes puntos pertenece(n) a la función 𝑦𝑦 = −𝑥𝑥 2 − 3𝑥𝑥 + 2?


I) (0, 2)
II) (1, −2)
III) (−1, 4)
A) Solo I
B) Solo II
C) Solo I y II
D) Solo II y III
E) I, II y III

CLEMENTE MORENO (MATERIAL DIDÁCTICO EN VALIDACIÓN 31


𝑡𝑡 2
12. La distancia que recorre un móvil viene dada por la función 𝑥𝑥 = 2𝑡𝑡 − con 𝑥𝑥 expresado
2

en metros y el tiempo 𝑡𝑡 en segundos. Luego, la distancia máxima que recorre el móvil es:
A) 0 m
B) 1 m
D) 2 m
C) 4 m
E) 6 m

13. Dada la función cuadrática 𝑓𝑓(𝑥𝑥) = 4𝑥𝑥 2 + 3𝑥𝑥 + 1, se puede afirmar que:
A) La parábola es cóncava hacia abajo
B) La parábola corta al eje y en el punto (0, −1)
C) El vértice de la parábola es el punto (−4, −3)
3
D) El eje de simetría de la parábola es la recta de ecuación 𝑥𝑥 =
8

E) La parábola no corta al eje x


14. La función cuadrática que representa la parábola del gráfico adjunto es:
A) 𝑓𝑓(𝑥𝑥) = 𝑥𝑥 2 + 5𝑥𝑥 + 4
B) 𝑓𝑓(𝑥𝑥) = −𝑥𝑥 2 − 5𝑥𝑥 − 4
C) 𝑓𝑓(𝑥𝑥) = 𝑥𝑥 2 + 5𝑥𝑥 − 4
D) 𝑓𝑓(𝑥𝑥) = −𝑥𝑥 2 + 5𝑥𝑥 − 4
E) 𝑓𝑓(𝑥𝑥) = −𝑥𝑥 2 + 5𝑥𝑥 + 4

1
15. ¿Cuál(es) de los siguientes puntos pertenece(n) a la función 𝑓𝑓(𝑥𝑥) = 𝑥𝑥 2 − 3𝑥𝑥 + 6?
4

I) (4, −2)
II) (−2, 13)
37
III) �−1, �
4

A) Solo I
B) Solo I y II
C) Solo I y III
D) Solo II y III
E) I, II y III

CLEMENTE MORENO (MATERIAL DIDÁCTICO EN VALIDACIÓN 32


16. ¿Cuál(es) de las siguientes funciones cuadráticas tiene(n) el mismo mínimo?
I) 𝑓𝑓(𝑥𝑥) = 𝑥𝑥 2 + 2𝑥𝑥 + 1
II) 𝑓𝑓(𝑥𝑥) = −𝑥𝑥 2 − 2𝑥𝑥 − 1
1 1 1
III) 𝑓𝑓(𝑥𝑥) = 𝑥𝑥 2 + 𝑥𝑥 +
4 2 4

A) I y II
B) I y III
C) II y III
D) I, II y III
E) Ninguna de ellas

17. La parábola asociada a la función 𝑓𝑓(𝑥𝑥) = 𝑥𝑥 2 − 10𝑥𝑥 + 25 corta al eje «x» en el(los) punto(s)
A) (−5, 0)
B) (5, 0)
C) (10, 0)
D) (−10, 0)
E) A y B
2
18. La imagen de −3 bajo la función 𝑓𝑓(𝑥𝑥) = 𝑥𝑥 2 − 3𝑥𝑥 + 4 es:
5
43
A) −
5
47
B)
5
53
C)
5
77
D)
5
83
E)
5

19. La pre-imagen de 6 bajo la función 𝑓𝑓(𝑥𝑥) = −3𝑥𝑥 2 + 11𝑥𝑥 es:


2
A)
3
2
B) −
3

C) 3
D) A y B
E) A y C

CLEMENTE MORENO (MATERIAL DIDÁCTICO EN VALIDACIÓN 33


20. Encuentre la función cuadrática que pasa por los puntos (0, 1), (1, 0) y (−2, 9)

21. Represente a las funciones 𝑓𝑓(𝑥𝑥) = 𝑥𝑥 2 ; 𝑔𝑔(𝑥𝑥) = 𝑥𝑥 2 − 2; ℎ(𝑥𝑥) = 𝑥𝑥 2 − 4 y 𝑚𝑚(𝑥𝑥) = 𝑥𝑥 2 + 4, en el


mismo eje de coordenadas. ¿En qué se parecen y en qué se diferencian estas funciones?

22. Dibuje las funciones 𝑓𝑓(𝑥𝑥) = −2𝑥𝑥 2 , 𝑔𝑔(𝑥𝑥) = −2𝑥𝑥 2 + 2, ℎ(𝑥𝑥) = −2𝑥𝑥 2 − 2 y 𝑚𝑚(𝑥𝑥) = −2𝑥𝑥 2 + 8
en el mismo eje de coordenadas. ¿En qué se parecen y qué se diferencian las funciones?

23. Dibuje las funciones 𝑓𝑓(𝑥𝑥) = −2𝑥𝑥 2 , 𝑔𝑔(𝑥𝑥) = −2(𝑥𝑥 + 2)2 , ℎ(𝑥𝑥) = −2(𝑥𝑥 − 3)2 y 𝑚𝑚(𝑥𝑥) = −2(𝑥𝑥 + 4)2
en el mismo eje de coordenadas. ¿En qué se parecen y diferencian las funciones?

24. Represente las siguientes funciones 𝑓𝑓(𝑥𝑥) = 𝑥𝑥 2 , 𝑔𝑔(𝑥𝑥) = (𝑥𝑥 + 2)2 + 1, ℎ(𝑥𝑥) = (𝑥𝑥 − 3)2 − 4 y
𝑚𝑚(𝑥𝑥) = (𝑥𝑥 + 4)2 + 2 en el mismo sistema de coordenadas. ¿En qué se parecen y diferencian
las funciones?

25. Si el producto de dos números que se diferencian en 45 unidades es 364, ¿Cuáles son los
números? S: 7 y 52

26. Una empresa se capitaliza bajo la siguiente función 𝐶𝐶(𝑡𝑡) = −𝑡𝑡 2 + 6𝑡𝑡 + 5, donde C es el
capital (en decenas de millones) y t es el tiempo (en décadas). Determine el capital máximo
de la empresa y en qué año de funcionamiento lo logra?
S: 𝐶𝐶 = 14 decenas de millones, 𝑡𝑡 = 3 años

27. Jesús y sus amigos están jugando paintball, la altura alcanzada por una de sus bolas de
pintura está dada por ℎ(𝑡𝑡) = 12𝑡𝑡 − 3𝑡𝑡 2 donde 𝑡𝑡 corresponde al tiempo que la bola está en el
aire. ¿A qué altura está a los 3 segundos de vuelo? S: 9 m

28. Encontrar dos números que sumados den 120 y cuyo producto sea el máximo posible
S: ambos son 60

CLEMENTE MORENO (MATERIAL DIDÁCTICO EN VALIDACIÓN 34


29. La trayectoria de un proyectil está dada por la ecuación 𝑠𝑠(𝑡𝑡) = 100𝑡𝑡 − 5𝑡𝑡 2 , donde t se mide
en segundos y la altura, 𝑠𝑠(𝑡𝑡) en metros. Calcular el valor de 𝑡𝑡 para que el proyectil se
encuentre a 255 metros sobre el nivel del suelo. S: 𝑡𝑡 = 3 seg

30. Si la ganancia de una empresa se puede determinar por la función f(x) = −x 2 + 14x + 13,
donde x es el número de unidades diarias producidas y f(x) las ganacias, ¿cuál será el
número de unidades diarias que se han producir para obtener la máxima ganancia?
S: 7 unidades

Función raíz cuadrada generalizada

Si te fijas, cuando se presentó a la función raíz cuadrada ℎ(𝑥𝑥) = √𝑥𝑥 como función
elemental «final página 7 e inicio de la página 8», puedes observar que a la función identidad
en la cantidad subradical le tomo, digamos la raíz cuadrada positiva, pero en este proceso, la
recta que representa a la idéntica fue deformada por la raíz cuadrada en una curva creciente
limitada a un cuadrante del plano, en ese caso al primero. Hecho debido a que la cantidad
subradical debe ser un número real positivo o cero para que la raíz cuadrada exista, es decir
𝑥𝑥 ≥ 0, lo cual indica que el dominio de la función es la solución a dicha inecuación

En tal sentido, el estudio de la función raíz cuadrada generalizada 𝑓𝑓(𝑥𝑥) = �𝑃𝑃(𝑥𝑥), donde
𝑃𝑃(𝑥𝑥) es una función polinomica o racional «limitado en este curso a polinomios de primero y
segundo grado junto a sus cocientes», inicia que el cálculo del dominio de la función es la
solución a la inecuación 𝑃𝑃(𝑥𝑥) ≥ 0, a partir del cual se construye el grafo de la función, dando
pie a su modo de representación tabular y modo de representación gráfico

Ejemplo 1. Realizar el estudio completo de la función 𝑓𝑓(𝑥𝑥) = √2𝑥𝑥 − 3 se tiene que:

𝑓𝑓(𝑥𝑥) es un número real si 2𝑥𝑥 − 3 ≥ 0 «planteamiento de la inecuación que conduce al dominio de 𝑓𝑓»

3
Y, 2𝑥𝑥 − 3 ≥ 0 implica 2𝑥𝑥 ≥ 3 o bien 𝑥𝑥 ≥ «se procede como en las ecuaciones, pero con el signo ≥»
2

3 3
Pero 𝑥𝑥 ≥ implica 𝑥𝑥 ∈ � , +∞� 3
«𝑥𝑥 pertenece al intervalo cerrado que inicia e incluye a 2 y se acerca a +∞»
2 2

CLEMENTE MORENO (MATERIAL DIDÁCTICO EN VALIDACIÓN 35


«representación gráfica del intervalo solución a la inecuación»
[
0 1 3 2 +∞ 3
«la flecha en rojo indica que inicia en 2 y toma todos los reales que siguen»
2
3
Luego 𝐷𝐷𝐷𝐷𝐷𝐷(𝑓𝑓) = � , +∞� 3
«del intervalo se toman algunos números iniciando en 2 para llenar la tabla»
2

3 3 5
Dado que 𝐷𝐷𝐷𝐷𝐷𝐷(𝑓𝑓) = � , +∞� tomemos como muestra a , 2, , 3, 4 que son números
2 2 2

del intervalo y evaluemos la función en dichos para ver su comportamiento, tal como se indica:

3
«se sustituyó 𝑥𝑥 por 2, se operó y calculó la raíz»
3 3 3
𝑓𝑓 � � = �2 � � − 3 = √0, así 𝑓𝑓 � � = 0
2 2 2
«se sustituyó 𝑥𝑥 por 2 se operó y calculó la raíz»
𝑓𝑓(2) = √2 ∙ 2 − 3 = √1, así 𝑓𝑓(2) = 1
5
5 5 5 «se sustituyó 𝑥𝑥 por 2, se operó y se dejó indicada la raíz»
𝑓𝑓 � � = �2 � � − 3 = √2, así 𝑓𝑓 � � = √2
2 2 2

«se sustituyó 𝑥𝑥 por 3, se operó y se dejó indicada la raíz»


𝑓𝑓(3) = √2 ∙ 3 − 3 = √3, así 𝑓𝑓(3) = √3

𝑓𝑓(4) = √2 ∙ 4 − 3 = √5, así 𝑓𝑓(4) = √5 «se sustituyó 𝑥𝑥 por 5, se operó y se dejó indicada la raíz»

Con los cálculos efectuados se realiza el estudio tabular y gráfico de la función, es decir:

+∞
3 5
𝑥𝑥 2 2 2 3 4
√5
√3
𝑦𝑦 0 1 √2 √3 √5 √2
1
[
0 1 3 2 5 3 4 +∞
2 2

Al comparar la representación gráfica de 𝑓𝑓(𝑥𝑥) = √2𝑥𝑥 − 3 con la de ℎ(𝑥𝑥) = √𝑥𝑥 dada al


3
inicio de este capítulo, se puede observar que «𝑓𝑓» con respecto a «ℎ» se traslado « » unidades
2

a la derecha de «0» y que crece con mayor rapidez debido a que el coeficiente de «𝑥𝑥» es «2»
que es mayor a «1». De haber sido un número comprendido entre «0» y «1» habría crecido con
menor rapidez que la curva de «ℎ», debido a que se comporta como la función del ejemplo 𝟐𝟐
de la página 𝟏𝟏𝟏𝟏. Por otra parte, al mirar la gráfica de «𝑓𝑓» se observa que el 𝑅𝑅𝑅𝑅𝑅𝑅(𝑓𝑓) = [0, +∞)

CLEMENTE MORENO (MATERIAL DIDÁCTICO EN VALIDACIÓN 36


Ejemplo 2. Realizar el estudio completo de la función 𝑔𝑔(𝑥𝑥) = √3 − 2𝑥𝑥 se tiene que:

𝑔𝑔(𝑥𝑥) es un número real si 3 − 2𝑥𝑥 ≥ 0 «planteamiento de la inecuación que conduce al dominio de 𝑓𝑓»

Y, 3 − 2𝑥𝑥 ≥ 0 implica −3 + 2𝑥𝑥 ≤ 0 «se multiplico por «−1» pues «𝑥𝑥» era negativa, la desigual cambia el sentido»

3
Así, 2𝑥𝑥 ≤ 3 o bien 𝑥𝑥 ≤ «se resuelve como ecuación pero en lugar de «=» se tiene a desigualdad «≤»»
2

3
Luego 𝐷𝐷𝐷𝐷𝐷𝐷(𝑔𝑔) = �−∞, � «el dominio se explicitó como intervalo para facilitar la toma de la muestra»
2

3 3
Dado que el 𝐷𝐷𝐷𝐷𝐷𝐷(𝑔𝑔) = �−∞, � se puede tomar a −1, 0, 1 y que son números del
2 2

intervalo para realizar el estudio tabular, es decir:

«se cambió 𝑥𝑥 por «−1» se operó y se dejó indicada la raíz»


𝑔𝑔(−1) = �3 − 2(−1) = √5, así 𝑔𝑔(−1) = √5

𝑔𝑔(0) = �3 − 2(0) = √3, así 𝑔𝑔(0) = √3 «se sustituyó 𝑥𝑥 por 0 se operó y se dejó indicada la raíz»

𝑔𝑔(1) = √3 − 2 ∙ 1 = √1, así 𝑔𝑔(1) = 1 «se sustituyó 𝑥𝑥 por 1, se operó y calculó la raíz»

3 3 3
𝑔𝑔 � � = �3 − 2 � � = √0, así 𝑔𝑔 � � = 0 3
«se sustituyó 𝑥𝑥 por 2, se operó y calculó la raíz»
2 2 2

Con los cálculos efectuados se realiza el estudio tabular y gráfico de la función, es decir:

+∞
3 √5
𝑥𝑥 −2 −1 0 1 √3
2 1
𝑦𝑦 √7 √5 √3 1 0 [
−∞ −2 −1 0 1 3
2

Ejemplo 3. Realizar el estudio completo de la función ℎ(𝑥𝑥) = √𝑥𝑥 2 − 4


Encontrar el dominio de este tipo de funciones implica resolver inecuaciones de grado
mayor que «1», para tal propósito se aplican los pasos descriptos a continuación

CLEMENTE MORENO (MATERIAL DIDÁCTICO EN VALIDACIÓN 37


COMENTARIOS

La estrategia de solución de inecuaciones de grado mayor a 1, consisten en lo siguiente:


1. Se dibuja la recta que representa a ℝ y se ubican los valores que anulan los factores en
que se descompone el miembro no nulo de la inecuación. A partir de estos se construye:
a. Rectas verticales que indican el número donde el factor que ellas anulan cambian de
signo, de «+» a «−» o de «–» a «+», formando columnas, a las cuales se le agrega
una columna para ubicar los factores en que se descompuso el miembro no nulo
b. La cantidad de factores en que se descompone el miembro no nulo de la inecuación,
indica el número de filas que compone la matriz, a estas también se agrega otra fila
para ubicar el producto de los signos contenidos en la columna
2. En la intersección de las filas con la última columna se colocan uno a uno los factores
lineales en que se descompuso el miembro no nulo de la inecuación. A partir de estos se
estudian los signos del factor, en la forma siguiente:
a. La «𝑥𝑥» del factor se sustituye por un número mayor al valor que anula el factor, si el
resultado es positivo se coloca «+» en las demás casillas ubicadas a la derecha de la
vertical y «−» en las demás casillas de la izquierda
b. La «𝑥𝑥» del factor se sustituye por un número mayor al valor que anula el factor, si el
resultado es negativo se coloca «−» en las demás casillas ubicadas a la derecha de
la vertical y «+» en las demás casillas de la izquierda
3. La última fila recoge el producto de los signos ubicados en cada columna y de los factores
en que se descompuso el miembro no nulo, incluido el signo de la inecuación. Con respecto
a estos se tiene:
a. Si al miembro no nulo lo precede el signo «> o ≥», se toma el intervalo o la unión de
los intervalos que tengan signo «+» como solución, estos serán abiertos en el caso de
«>» y cerrados cuando tengan «≥»
b. Si al miembro no nulo lo precede el signo «< o ≤», se toma el intervalo o la unión de
los intervalos que tengan signo «−» como solución, estos serán abiertos en el caso de
«<» y cerrados cuando tengan «≤»

CLEMENTE MORENO (MATERIAL DIDÁCTICO EN VALIDACIÓN 38


Aplicando los comentarios al cálculo del dominio de la función ℎ(𝑥𝑥) = √𝑥𝑥 2 − 4, se tiene:

ℎ(𝑥𝑥) es un número real si x 2 − 4 ≥ 0 «planteamiento de la inecuación que conduce al dominio de ℎ»

x 2 − 4 ≥ 0 implica (x − 2)(x + 2) ≥ 0 «se factoriza el binomio del 1er miembro»

Con, x = 2 y x = −2 «ceros de los factores de la inecuación, números que anulan los factores»

Resta estudiar el cambio de signos siguiendo los comentarios arriba descriptos


«las verticales por −2 y 2 indican el cambio de signo del factor»
−2 2 ℝ
− + + 𝑥𝑥 + 2 «al sustituir x < −2 en x + 2 da (−); y al sustituir las 𝑥𝑥 > −2 da (+)»

− − + 𝑥𝑥 − 2 «al sustituir x < 2 en x − 2 da (−); y al sustituir las 𝑥𝑥 > 2 da (+)»


+ − + 𝑥𝑥 2 − 4 ≥ 0 «los signos de la última fila resultan de multiplicar los signos de las otras»

Luego, 𝐷𝐷𝐷𝐷𝐷𝐷(ℎ) = (−∞, −2] ∪ [2, +∞) «el dominio es la unión de los intervalos «+», la inecuación es «≥ 0»»

Y, −4,−3,−2 junto a 2, 3, 4 «son del 𝐷𝐷𝐷𝐷𝐷𝐷(ℎ), la función puede evaluares en ellos para hacer la tabla»

«se sustituyó 𝑥𝑥 por −4 se operó y se dejó indicada la raíz»


ℎ(−4) = �(−4)2 − 4 = √12, así ℎ(−4) = 2√3

ℎ(−3) = �(−3)2 − 4 = √5, así ℎ(−3) = √5 «se sustituyó 𝑥𝑥 por −3 se operó y se dejó indicada la raíz»

«se sustituyó 𝑥𝑥 por −2 se operó y se calculó la raíz»


ℎ(−2) = �(−2)2 − 4 = √0, así ℎ(−2) = 0
«se sustituyó 𝑥𝑥 por 2 se operó y se calculó la raíz»
ℎ(2) = √22 − 4 = √0, así ℎ(2) = 0
«se sustituyó 𝑥𝑥 por 3 se operó y se dejó indicada la raíz»
ℎ(3) = √32 − 4 = √5, así ℎ(3) = √5

ℎ(4) = √42 − 4 = √12, así ℎ(4) = 2√3 «se sustituyó 𝑥𝑥 por 4 se operó y se dejó indicada la raíz»

Con los valores calculados y teniendo en cuenta el dominio de la función, se construye


su tabla y su gráfica, tal como se indica a continuación:

+∞
4
𝑥𝑥 −4 −3 −2 2 3 4 3
2
𝑦𝑦 2√3 √5 0 0 √5 2√3
1

−∞ −4 −3 −2 −1 1 2 3 4 +∞

CLEMENTE MORENO (MATERIAL DIDÁCTICO EN VALIDACIÓN 39


Ejemplo 4. Realizar el estudio completo de la función 𝑓𝑓(𝑥𝑥) = √6 − 5𝑥𝑥 − 6𝑥𝑥 2 se tiene que:

𝑓𝑓(𝑥𝑥) es un número real si 6 − 5𝑥𝑥 − 6𝑥𝑥 2 ≥ 0 «planteamiento de la inecuación que lleva al dominio de ℎ»

6 − 5x − 6x 2 ≥ 0 equivale a (2x + 3)(2 − 3x) ≥ 0 «factorizando el 1er miembro de la desigualdad»

3 2 «elaboración de la tabla según pasos 1 y 2 de los comentarios»



2 3 ℝ «estudio de signos según paso 2 de los comentarios»
− + + 2𝑥𝑥 + 3
+ + − 2 − 3𝑥𝑥 «multiplicación de signos según paso 3 de los comentarios»
− + −

3 2
Luego 𝐷𝐷𝐷𝐷𝐷𝐷(𝑓𝑓) = �− , � 3
«el dominio es intervalo cerrado de extremos − 2 y 3»
2
2 3
3 2
Y, − , −1, 0, «son del 𝐷𝐷𝐷𝐷𝐷𝐷(𝑓𝑓), la función puede evaluares en ellos para hacer la tabla»
2 3

3 3 3 2 15 18 3 3√2
𝑓𝑓 �− � = �6 − 5 �− � − 6 �− � = �6 + − así 𝑓𝑓 �− � = «operando y racionalizando»
2 2 2 2 2 2 2

𝑓𝑓(−1) = �6 − 5(−1) − 6(−1)2 = √5, así 𝑓𝑓(−1) = √5 «sustituyendo y operando»

𝑓𝑓(0) = �6 − 5(0) − 6(0)2 = √6, así 𝑓𝑓(0) = √6 «sustituyendo y operando»

2 2 2 2 10 8 2
𝑓𝑓 � � = �6 − 5 � � − 6 � � = �6 − − , así 𝑓𝑓 � � = 0 «sustituyendo y operando»
3 3 3 3 3 3

Con los valores calculados y teniendo en cuenta el dominio de la función, se construye


su tabla y su gráfica, tal como se indica en lo que sigue

3 2 3
𝑥𝑥 − −1 0
2 3 2
3√2
𝑦𝑦
2
√5 √6 0 1

3 2
−∞ − −1 1 +∞
2 3

CLEMENTE MORENO (MATERIAL DIDÁCTICO EN VALIDACIÓN 40


3
Función raíz cúbica 𝑓𝑓(𝑥𝑥) = √𝑥𝑥 «a cualquier número real «𝑥𝑥» le extrae su raíz cúbica»

3 3 3 3
En ella 𝑓𝑓(−8) = √−8 = −2,… 𝑓𝑓(−4) = √−4,… 𝑓𝑓(−1) = √−1 = −1,… 𝑓𝑓(0) = √0 = 0,…
3 3 3
𝑓𝑓(1) = √1 = 1,… 𝑓𝑓(4) = √4,… 𝑓𝑓(8) = √8 = 2,… De este modo, el grafo de la función sería:
3 3
𝐺𝐺𝐺𝐺𝐺𝐺(𝑓𝑓) = �… (−8, −2), … �−4, √−4�, … (−1, −1), … (0,0), … (1, 1), … �4, √4�, … (8, 2), … � y a
partir de estos valores se puede hacer la tabal «subconjunto finito del grafo» y la representación
gráfica de la función, tal como se indica

𝑥𝑥 −8 −4 −1 0 1 4 8

𝑦𝑦 −2 √−4 −1 0 1 √4 2

COMENTARIOS

(i) Si te fijas, la cantidad subradical de la raíz cúbica es la función identidad, cuya gráfica era
una recta oblicua que indicaba, que tanto su dominio como su rango era ℝ, pues bien, la
función raíz cúbica deforma esa recta en la curva de la figura anterior, pero no limita ni su
dominio ni su rango. En tal sentido, para hallar el dominio ℎ(𝑥𝑥) = 3�𝑓𝑓(𝑥𝑥) basta determinar el
dominio de 𝑓𝑓(𝑥𝑥), es decir 𝐷𝐷𝐷𝐷𝐷𝐷(ℎ) = 𝐷𝐷𝐷𝐷𝐷𝐷(𝑓𝑓); de igual modo, 𝑅𝑅𝑅𝑅𝑅𝑅(ℎ) = 𝑅𝑅𝑅𝑅𝑅𝑅(𝑓𝑓)
(ii) El estudio tabular se limita a evaluar a ℎ(𝑥𝑥) = 3�𝑓𝑓(𝑥𝑥) en una muestra conveniente de
números reales y confeccionar la tabla, mientras que el dibujo de su gráfica es un trazo
3
continuo similar al de 𝑓𝑓(𝑥𝑥) = √𝑥𝑥

3 3
Ejemplo 1. Calcular el dominio de 𝑓𝑓(𝑥𝑥) = √2𝑥𝑥 − 3 y 𝑔𝑔(𝑥𝑥) = √𝑥𝑥 2 − 5𝑥𝑥 + 6

𝐷𝐷𝐷𝐷𝐷𝐷(𝑓𝑓) = 𝐷𝐷𝐷𝐷𝐷𝐷(𝑓𝑓1 ) donde 𝑓𝑓1 (𝑥𝑥) = 2𝑥𝑥 − 3 «definición de la función ubicada como expresión subradical»

Y, 𝐷𝐷𝐷𝐷𝐷𝐷(𝑓𝑓1 ) = ℝ «𝑓𝑓1 es una función afín y su dominio es el conjunto de los números reales»

Luego, 𝐷𝐷𝐷𝐷𝐷𝐷(𝑓𝑓) = ℝ «𝐷𝐷𝐷𝐷𝐷𝐷(𝑓𝑓) = 𝐷𝐷𝐷𝐷𝐷𝐷(𝑓𝑓1 ) de acuerdo al comentario (i)»

𝐷𝐷𝐷𝐷𝐷𝐷(𝑔𝑔) = 𝐷𝐷𝐷𝐷𝐷𝐷(𝑔𝑔1 ) donde 𝑔𝑔1 (𝑥𝑥) = 𝑥𝑥 2 − 5𝑥𝑥 + 6 «definición de la función ubicada como expresión subradical»

Y, 𝐷𝐷𝐷𝐷𝐷𝐷(𝑔𝑔1 ) = ℝ «𝑔𝑔1 es una función cuadrática y su dominio es el conjunto de los números reales»

CLEMENTE MORENO (MATERIAL DIDÁCTICO EN VALIDACIÓN 41


Luego, 𝐷𝐷𝐷𝐷𝐷𝐷(𝑔𝑔) = ℝ «𝐷𝐷𝐷𝐷𝐷𝐷(𝑔𝑔) = 𝐷𝐷𝐷𝐷𝐷𝐷(𝑔𝑔1 ) de acuerdo al comentario (i)»

Como actividad para el lector, resta realizar el estudio tabular y gráfico de cada una de
estas funciones y a partir de sus gráficas verificar que sus rangos corresponden al conjunto de
los números reales ℝ

Ejercicio N° 𝟓𝟓
En las expresiones dadas a continuación, determine, en caso de ser posible, el conjunto de
definición «dominio» para que las expresiones sean consideradas funciones
3
1) 𝑓𝑓(𝑥𝑥) = √3𝑥𝑥 + 5; 2) 𝑔𝑔(𝑥𝑥) = √2𝑥𝑥 − 7; 2 1
3) ℎ(𝑥𝑥) = � − 𝑥𝑥;
3 9

3 3 5 3 2𝑥𝑥
4) 𝑙𝑙(𝑥𝑥) = � 𝑥𝑥 − ; 5) 𝑝𝑝(𝑥𝑥) = �2𝑥𝑥 − ; 6) 𝑞𝑞(𝑥𝑥) = �1 − ;
4 16 4 25

3
7) 𝑚𝑚(𝑥𝑥) = √2𝑥𝑥 2 − 5𝑥𝑥 + 7; 8) 𝑓𝑓(𝑥𝑥) = √𝑥𝑥 2 + 3𝑥𝑥 − 4; 9) 𝑛𝑛(𝑥𝑥) = √2 + 3𝑥𝑥 − 2𝑥𝑥 2 ;
3
10) 𝑔𝑔(𝑥𝑥) = √9𝑥𝑥 2 − 30𝑥𝑥 + 25; 11) ℎ(𝑥𝑥) = √4𝑥𝑥 2 − 12𝑥𝑥 + 9; 12) 𝑝𝑝(𝑥𝑥) = √5𝑥𝑥 2 − 3𝑥𝑥 + 4:

Realice el estudio completo «cálculo del dominio, estudio tabular y gráfico e indicar el rango»
en cada una de las siguientes funciones
3
13) 𝑓𝑓(𝑥𝑥) = √3𝑥𝑥 − 5; 14) 𝑔𝑔(𝑥𝑥) = √3 − 4𝑥𝑥; 15) ℎ(𝑥𝑥) = √2𝑥𝑥 − 3;
3
16) 𝑙𝑙(𝑥𝑥) = √4𝑥𝑥 2 − 9; 17) 𝑝𝑝(𝑥𝑥) = √4𝑥𝑥 2 − 9; 18) 𝑞𝑞(𝑥𝑥) = √4 − 𝑥𝑥 2 ;
3
19) 𝑚𝑚(𝑥𝑥) = √4 − 𝑥𝑥 2 ; 20) 𝑓𝑓(𝑥𝑥) = √9𝑥𝑥 2 − 6𝑥𝑥 + 1; 21) 𝑛𝑛(𝑥𝑥) = √2𝑥𝑥 2 − 3𝑥𝑥 − 2;

3
Dadas las funciones 𝑓𝑓(𝑥𝑥) = √𝑥𝑥 − 1; 𝑔𝑔(𝑥𝑥) = 𝑥𝑥 2 − 3; ℎ(𝑥𝑥) = √2𝑥𝑥 − 3 y 𝑙𝑙(𝑥𝑥) = √𝑥𝑥 2 − 4, halle en
caso de ser posible, las imágenes «evaluación de las funciones en esos puntos» de:

22) (𝑓𝑓 + 𝑔𝑔)(1); 23) (𝑔𝑔 − 𝑓𝑓)(2); 24) (𝑓𝑓 − 𝑔𝑔)(5)


𝑓𝑓 𝑔𝑔 𝑓𝑓
25) � � (2) 26) � � (2); 27) � � (5);
𝑔𝑔 𝑓𝑓 ℎ

ℎ 𝑙𝑙 𝑔𝑔
28) � � (1); 29) � � (0); 30) � � (2);
𝑙𝑙 𝑔𝑔 𝑙𝑙

CLEMENTE MORENO (MATERIAL DIDÁCTICO EN VALIDACIÓN 42


Función valor absoluto 𝑓𝑓(𝑥𝑥) = |𝑥𝑥| «a todo número real le toma su valor absoluto»

𝑥𝑥 si 𝑥𝑥 ≥ 0
En atención a la definición de valor absoluto, la función 𝑓𝑓(𝑥𝑥) = |𝑥𝑥| = �
−𝑥𝑥 si 𝑥𝑥 < 0
indica que cuando el número real a evaluar en la función es cero o mayor que cero, la evaluación
debe hacerse en la primera rama de la función y cuando el número real es menor que cero
«negativo» la evaluación debe hacerse en la segunda rama. Así, para evaluar «hallar la imagen»
1 1
de por ejemplo −2, −1, − , 0, , 1, 2 se procede como se indica a continuación
2 2

𝑓𝑓(−2) = |−2| = −(−2), así 𝑓𝑓(−2) = 2 «−2 es menor que 0 «negativo» se evalúa en la segunda rama»

𝑓𝑓(−1) = |−1| = −(−1), así 𝑓𝑓(−1) = 1 «−1 es menor que 0 «negativo» se evalúa en la segunda rama»

1 1 1 1 1
𝑓𝑓 �− � = �− � = − �− �, así 𝑓𝑓 �− � = 1
«− 2 es menor que 0 «negativo» se evalúa en la segunda rama»
2 2 2 2 2

𝑓𝑓(0) = |0| = 0 «0 según la definición se evalúa en la primera rama»

1 1 1 1 1
𝑓𝑓 � � = � � = , así 𝑓𝑓 � � = 1
«2 es mayor que 0 «positivo» se evalúa en la primer rama»
2 2 2 2 2

𝑓𝑓(1) = |1| = 1, así 𝑓𝑓(1) = 1 «1 es mayor que 0 «positivo» se evalúa en la primera rama»

𝑓𝑓(2) = |2| = 2, así 𝑓𝑓(2) = 2 «2 es mayor que 0 «positivo» se evalúa en la primera rama»

Con estos cálculos se elabora el grafo de la función a fin de construir su tabla y


representación gráfica, tal como se ha venido haciendo con las funciones anteriores, es decir:
1 1 1 1
𝐺𝐺𝐺𝐺𝐺𝐺(𝑓𝑓) = �… (−2, 2), … (−1, 1), … �− , � , … (0,0), … � , � , … (1,1), … (2, 2), … �, mientras que
2 2 2 2

su tabla y representación gráfica son:

+∞
2
1 1
𝑥𝑥 −2 −1 − 0 1 2
2 2 1
1 1
𝑦𝑦 2 1 0 1 2
2 2
−∞ −2 −1 − 1 1 1 2 +∞
2 2

CLEMENTE MORENO (MATERIAL DIDÁCTICO EN VALIDACIÓN 43


COMENTARIOS

(i) Si observas con atención, veras que la función valor absoluto provocó una reflexión del
segmento de recta del tercer cuadrante «coloreada en azul» con respecto al eje «𝑥𝑥»
ubicándola en el segundo cuadrante «la gráfica de 𝑦𝑦 = 𝑥𝑥, doblada por el eje 𝑥𝑥, luego
remarcada desde el dorso del papel doblado genera la gráfica de 𝑦𝑦 = |𝑥𝑥|»
(ii) La función valor absoluto no restringe el dominio de la función identidad, de modo general
𝐷𝐷𝑜𝑜𝑜𝑜(𝑓𝑓) = |𝑔𝑔(𝑥𝑥)| = 𝐷𝐷𝐷𝐷𝐷𝐷(𝑔𝑔), pero el rango se restringe a los reales positivos unidos con el
conjunto cuyo único elemento es cero, es decir 𝑅𝑅𝑅𝑅𝑅𝑅(𝑓𝑓) = ℝ+ ∪ {0} o bien 𝑅𝑅𝑅𝑅𝑅𝑅(𝑓𝑓) = [0, +∞)

Ejemplo 1. Realizar el estudio completo de la función 𝑓𝑓(𝑥𝑥) = |2𝑥𝑥 − 3| se tiene que:


2𝑥𝑥 − 3 si 2𝑥𝑥 − 3 ≥ 0
𝑓𝑓(𝑥𝑥) = |2𝑥𝑥 − 3| implica 𝑓𝑓(𝑥𝑥) = � «se aplicó la definición de valor absoluto»
−(2𝑥𝑥 − 3) si 2𝑥𝑥 − 3 < 0
3
2𝑥𝑥 − 3 si 𝑥𝑥 ≥
2
Así, 𝑓𝑓(𝑥𝑥) = � 3 «se resolvió la inecuación de cada rama para especificar el dominio de ellas»
−(2𝑥𝑥 − 3) si 𝑥𝑥 <
2

Especificado el dominio de cada rama de la función se procede a elegir los números en


1 3
los que ellas serán evaluadas, así −1, − , 0, 1 y números muy próximos a son números que
2 2
3 5
pertenecen a la segunda rama de la función, mientras que , 2, , 3 son números de la primera
2 2

rama. En atención a ello se tiene:

𝑓𝑓(−1) = −[2(−1) − 3] = 5, así 𝑓𝑓(−1) = 5 3


«−1 es menor que 2 se evalúa en la segunda rama»

1 1 1
𝑓𝑓 �− � = − �2 �− � − 3� = 4, así 𝑓𝑓 �− � = 4 1 3
«− 2 es menor que 2 se evalúa en la segunda rama»
2 2 2

𝑓𝑓(0) = −[2(0) − 3] = 3, así 𝑓𝑓(0) = 3 3


«0 es menor que 2 se evalúa en la segunda rama»

𝑓𝑓(1) = −[2(1) − 3] = 1, así 𝑓𝑓(1) = 1 3


«1 es menor que 2 se evalúa en la segunda rama»

3 3 3
� � = 2 � � − 3 = 0, así 𝑓𝑓 � � = 0 3
«2 es igual a
3
se evalúa en la primera rama»
2 2 2 2

CLEMENTE MORENO (MATERIAL DIDÁCTICO EN VALIDACIÓN 44


𝑓𝑓(2) = 2 ∙ 2 − 3 = 1, así 𝑓𝑓(2) = 1 3
«2 es mayor que 2 se evalúa en la primera rama»

5 5 5
𝑓𝑓 � � = 2 � � − 3 = 2, así 𝑓𝑓 � � = 2 5
«2 es mayor que
3
se evalúa en la primera rama»
2 2 2 2

𝑓𝑓(3) = 2 ∙ 3 − 3 = 3, así 𝑓𝑓(3) = 3 3


«3 es mayor que 2 se evalúa en la primera rama»

Con estos valores se construye la tabla y dibuja la gráfica, como en casos anteriores, así:

+∞
4
1 3
𝑥𝑥 −1 − 0 1 2 3
2 2 3
𝑦𝑦 5 4 3 1 0 1 3
2

−∞ −1 − 1 1 3 2 3 +∞
2 2

Si comparas las gráficas de 𝑓𝑓(𝑥𝑥) = |𝑥𝑥| y 𝑓𝑓(𝑥𝑥) = |2𝑥𝑥 − 3| puedes observar que la
segunda es una traslación de la primera y más cerrada debido al coeficiente de «𝑥𝑥», lo cual es
convergente con lo planteado en los ejemplos 1 y 2 de las páginas 9 y 10 respectivamente.
Finalmente, 𝐷𝐷𝐷𝐷𝐷𝐷(𝑓𝑓) = ℝ y 𝑅𝑅𝑅𝑅𝑅𝑅(𝑓𝑓) = [0, +∞)

Ejemplo 2. Realizar el estudio completo de la función 𝑔𝑔(𝑥𝑥) = |6𝑥𝑥 2 + 5𝑥𝑥 − 6| se tiene que:

6𝑥𝑥 2 + 5𝑥𝑥 − 6 si 6𝑥𝑥 2 + 5𝑥𝑥 − 6 ≥ 0


𝑔𝑔(𝑥𝑥) = |6𝑥𝑥 2 + 5𝑥𝑥 − 6| implica 𝑔𝑔(𝑥𝑥) = � «definición valor absoluto»
−(6𝑥𝑥 2 + 5𝑥𝑥 − 6) si 6𝑥𝑥 2 + 5𝑥𝑥 − 6 < 0

Estudiar el cambio de signo la inecuación 6𝑥𝑥 2 + 5𝑥𝑥 − 6 ≥ 0 genera los intervalos que
definen los dominios de cada rama de la función, es decir

6𝑥𝑥 2 + 5𝑥𝑥 − 6 ≥ 0 equivale a (2𝑥𝑥 + 3)(3𝑥𝑥 − 2) ≥ 0 «factorizando el 1er miembro de la desigualdad»

CLEMENTE MORENO (MATERIAL DIDÁCTICO EN VALIDACIÓN 45


«elaboración de la tabla, pasos 1 y 2 de los comentarios respectivos»
3 2

2 3 «estudio de signos, paso 2 de los comentarios respectivos»

− + + 2𝑥𝑥 + 3
«multiplicación de signos, paso 3 de los comentarios respectivos»
− − + 3𝑥𝑥 −2
+ − +

3 2
6𝑥𝑥 2 + 5𝑥𝑥 − 6 si 𝑥𝑥 ∈ �−∞, − � ∪ � , +∞�
2 3
Así, 𝑓𝑓(𝑥𝑥) = � 3 2
«especificación del dominio de las ramas de 𝑔𝑔(𝑥𝑥)»
−(6𝑥𝑥 2 + 5𝑥𝑥 − 6) si 𝑥𝑥 ∈ �− , �
2 3

3
Luego −3, −2 y − son números del primer intervalo de la primera rama, mientras que
2
5 1 2
−1, − , 0 y son números de la segunda rama, finalmente los números , 1 y 2 del segundo
12 3 3

intervalo de la primera rama. Con ellos se construye la tabla y dibuja la gráfica de la función, tal
como se muestra a continuación

+∞
10

8
3 5 1 2
𝑥𝑥 −3 −2 − −1 − 0 1 2 6
2 12 3 3
169 11 4
𝑦𝑦 33 8 0 5 6 0 5 28
24 3
2

−∞ −3 −2 −1 1 2 3 +∞

Como se apuntó arriba, la parte de la parábola coloreada en rojo es una reflexión con
respecto al eje «𝑥𝑥» de la parábola coloreada en azul, es decir que para graficar la parábola
interna del valor absoluto basta con dibujarla de manera normal, luego doblar la hoja por el eje
«𝑥𝑥» y remarcar desde el dorso de la hoja la parte de la parábola que estaba abajo del eje «𝑥𝑥».
Además, 𝐷𝐷𝐷𝐷𝐷𝐷(𝑓𝑓) = ℝ y el 𝑅𝑅𝑅𝑅𝑅𝑅(𝑓𝑓) = [0, +∞)

CLEMENTE MORENO (MATERIAL DIDÁCTICO EN VALIDACIÓN 46


Funciones definidas a trozos
Las funciones definidas a trozos, también llamadas funciones por partes o funciones
seccionadas, cambian su definición «regla de correspondencia» conforme cambia su variable
independiente, es decir, su definición cambia en cada conjunto disjunto de su dominio. Las
funciones con valor absoluto estudiadas arriba, son un caso particular de este tipo de funciones

2 si − 3 < 𝑥𝑥 ≤ −1
Ejemplo 1. Estudiar la función particionada 𝑓𝑓(𝑥𝑥) = � 𝑥𝑥 2
si − 1 < 𝑥𝑥 ≤ 2
𝑥𝑥 − 1 si 2 < 𝑥𝑥 ≤ 3

COMENTARIOS

(i) Lo primero a observar es que la intersección de los dominios entre las ramas que definen la
función es el conjunto vacío. Aquí, la intersección de (−3. −1] ∩ (−1,2] ∩ (2,3] = ∅, pues el
extremo derecho del primer intervalo contiene a «−1» por ser cerrado «]» pero el extremo
inicial del segundo no lo contiene por ser abierto «(», por ello su intersección es el vacío; de
igual modo, el extremo derecho del segundo intervalo contiene a «2» por ser cerrado «]»
pero el extremo inicial del tercer intervalo no lo contiene por ser abierto «(», luego su
intersección es el vacío. Esta situación se ilustra en la figura adjunta

( ]( ]( ] ℝ
−∞ −3 −1 2 3 +∞

(ii) Si la intersección de (−3. −1] ∩ (−1,2] ∩ (2,3] ≠ ∅ habría existo pares ordenados en el
grafo de «𝑓𝑓» con la primera componente igual, indicando que la relación no era función

De regreso al estudio de la función se tiene que en el primer intervalo −3 < 𝑥𝑥 ≤ −1 o


bien (−3. −1], la rama 𝑓𝑓1 (𝑥𝑥) = 2 es una función constante, luego su gráfica es el segmento
de recta paralelo al eje «𝑥𝑥» que va de −3 a −1 abierto «∘» en −3; de igual modo, en el segundo
intervalo −1 < 𝑥𝑥 ≤ 2 o bien (−1,2], la rama 𝑓𝑓2 (𝑥𝑥) = 𝑥𝑥 2 es una parábola que va desde −1
hasta 2, abierta «∘» en −1; finalmente, en el tercer intervalo 2 < 𝑥𝑥 ≤ 3 o bien (2,3], la rama

CLEMENTE MORENO (MATERIAL DIDÁCTICO EN VALIDACIÓN 47


𝑓𝑓3 (𝑥𝑥) = 𝑥𝑥 − 1 es una función afín que va desde 2 hasta 3, abierta «∘» en 2. La figura adjunta,
muestra la representación gráfica de la función

Y
4
3
2
° 1 °

-3-2 -1 1 2 3 X

El lector pudo haber realizado el estudio tabular para cada rama y luego dibujarlas todas
en el mismo plano cartesiano como se exhibe en el ejemplo

Ejercicio N° 𝟔𝟔
En los casos siguientes, utilice la definición de valor absoluto y reescriba la función dada como
su equivalente definida en dos ramas

1) 𝑓𝑓(𝑥𝑥) = |2𝑥𝑥 − 5|; 2) 𝑔𝑔(𝑥𝑥) = |3 − 4𝑥𝑥|; 1


3) ℎ(𝑥𝑥) = � 𝑥𝑥 − �;
2
2 3

4) 𝑙𝑙(𝑥𝑥) = �√𝑥𝑥 − 3�;


3
5) 𝑝𝑝(𝑥𝑥) = � √3 − 2𝑥𝑥 �; 6) 𝑚𝑚(𝑥𝑥) = |4 − 𝑥𝑥 2 |;

7) 𝑛𝑛(𝑥𝑥) = |2𝑥𝑥 2 − 5𝑥𝑥 − 3|; 8) 𝑞𝑞(𝑥𝑥) = |3𝑥𝑥 2 + 𝑥𝑥 − 2|; 9) 𝑟𝑟(𝑥𝑥) = |2 + 3𝑥𝑥 − 2𝑥𝑥 2 |;

Realice el estudio completo de las siguientes funciones

10) 𝑓𝑓(𝑥𝑥) = |3𝑥𝑥 + 5|; 11) 𝑔𝑔(𝑥𝑥) = |4 − 3𝑥𝑥|; 2


12) ℎ(𝑥𝑥) = � 𝑥𝑥 + �;
1
3 2

13) 𝑙𝑙(𝑥𝑥) = |𝑥𝑥 2 − 9|; 14) 𝑝𝑝(𝑥𝑥) = |𝑥𝑥 2 + 𝑥𝑥 − 6| 15) 𝑚𝑚(𝑥𝑥) = |3 − 2𝑥𝑥 − 𝑥𝑥 2 |

Realice el estudio completo de las siguientes funciones


−2 si 𝑥𝑥 ≤ −1 2 − 3𝑥𝑥 si − 6 < 𝑥𝑥 ≤ −1
16) 𝑓𝑓(𝑥𝑥) = �2𝑥𝑥 − 3 𝑠𝑠i − 1 < 𝑥𝑥 ≤ 2 17) 𝑔𝑔(𝑥𝑥) = � 2𝑥𝑥 + 3 si − 1 < 𝑥𝑥 ≤ 2
√3𝑥𝑥 + 1 si 2 < 𝑥𝑥 ≤ 5 √4𝑥𝑥 − 3 si 2 < 𝑥𝑥 ≤ 7

CLEMENTE MORENO (MATERIAL DIDÁCTICO EN VALIDACIÓN 48


2𝑥𝑥 + 1 si − 3 < 𝑥𝑥 < −1 2𝑥𝑥 + 3 si − 2 ≤ 𝑥𝑥 < 0
18) ℎ(𝑥𝑥) = � 𝑥𝑥 2 − 1 si − 1 ≤ 𝑥𝑥 < 2 19) 𝑙𝑙(𝑥𝑥) = � √3𝑥𝑥 − 1 si 0 ≤ 𝑥𝑥 < 4
3 − 2𝑥𝑥 si 2 ≤ 𝑥𝑥 < 5 1 2
1 − 𝑥𝑥 si 4 ≤ 𝑥𝑥 < 6
4

3
2 − 3𝑥𝑥 si − 7 < 𝑥𝑥 ≤ 1 3𝑥𝑥 − 1 si − 5 < 𝑥𝑥 ≤ 0
Dadas 𝑓𝑓(𝑥𝑥) = � y 𝑔𝑔(𝑥𝑥) = � √ determine
√2𝑥𝑥 − 3 si 2 < 𝑥𝑥 ≤ 7 √3𝑥𝑥 + 1 si 1 < 𝑥𝑥 ≤ 5
20) (𝑓𝑓 + 𝑔𝑔)(−1); 21) (𝑔𝑔 − 𝑓𝑓)(3)

22) (𝑓𝑓 + 𝑔𝑔)(𝑥𝑥); 𝑓𝑓


23) � � (𝑥𝑥);
𝑔𝑔

La función compuesta
Al igual que el álgebra entre funciones «operaciones estudiadas arriba» genera una nueva
función, si la intersección de sus dominios es un conjunto distinto del conjunto vacío, la
composición entre las funciones reales 𝑓𝑓 y g denotada por 𝑔𝑔 ∘ 𝑓𝑓 es otra función, a la que se
llama función compuesta de 𝑓𝑓 y g escrita 𝑔𝑔 ∘ 𝑓𝑓 y es tal que [𝑔𝑔 ∘ 𝑓𝑓] (𝑥𝑥) = 𝑔𝑔[𝑓𝑓(𝑥𝑥)]
El dominio de la función 𝑔𝑔 ∘ 𝑓𝑓 está formado por el
A C D
conjunto de los números «𝑥𝑥» del dominio de la función 𝑓𝑓, B

tales que 𝑓𝑓(𝑥𝑥) se encuentra en el dominio de la función 𝑔𝑔,


°
en símbolos se escribe: x° ° g[ f ( x)]
f (x)
𝐷𝐷𝐷𝐷𝐷𝐷(𝑔𝑔 ∘ 𝑓𝑓) = {𝑥𝑥 ∈ 𝐷𝐷𝐷𝐷𝐷𝐷(𝑓𝑓) ∕ 𝑓𝑓(𝑥𝑥) ∈ 𝐷𝐷𝐷𝐷𝐷𝐷(𝑔𝑔)}. En la figura
se ilustra esquemáticamente la función compuesta entre
𝑓𝑓 y 𝑔𝑔

Ejemplo 1. Considere los conjuntos 𝐴𝐴 = {1, 2, 3}; 𝐵𝐵 = {𝑎𝑎, 𝑏𝑏, 𝑐𝑐, 𝑑𝑑} y 𝐶𝐶 = {5, 6}, con ellos las
funciones 𝑓𝑓: 𝐴𝐴 → 𝐵𝐵 y 𝑔𝑔: 𝐵𝐵′ → 𝐶𝐶 definidas por:
f g
A B C
B′
𝑓𝑓 = {(1, 𝑎𝑎), (2, 𝑏𝑏), (3, 𝑑𝑑)} y 𝑔𝑔 = {(𝑎𝑎, 5), (𝑏𝑏, 5), (𝑑𝑑, 6)} a
1 °
° ° 5
2 ° b
° °6
d
De este modo, la función compuesta entre las 3°
°
c
funciones 𝑓𝑓 y 𝑔𝑔 se define por gof

𝑔𝑔 ∘ 𝑓𝑓 = {(1, 5), (2, 5), (3, 6)}


los diagramas de Venn dados en la figura exhiben la composición entre las funciones 𝑓𝑓 y 𝑔𝑔

CLEMENTE MORENO (MATERIAL DIDÁCTICO EN VALIDACIÓN 49


COMENTARIOS

(i) La función que aplica de primero se encuentra más cerca de «𝑥𝑥» y se llama función interna;
la que aplica de último está más lejos de «𝑥𝑥» y se llama función externa
(ii) Para que la función compuesta exista es necesario que el rango de la función que aplica
primero sea subconjunto del dominio de la función que aplica después, es decir:
(𝑔𝑔 ∘ 𝑓𝑓)(𝑥𝑥) existe si y sólo sí 𝐷𝐷𝐷𝐷𝐷𝐷(𝑔𝑔) ∩ 𝑅𝑅𝑅𝑅𝑅𝑅(𝑓𝑓) ≠ ∅
(𝑓𝑓 ∘ 𝑔𝑔)(𝑥𝑥) existe si y sólo sí 𝐷𝐷𝐷𝐷𝐷𝐷(𝑓𝑓) ∩ 𝑅𝑅𝑅𝑅𝑅𝑅(𝑔𝑔) ≠ ∅
Siempre es posible componer dos funciones reales si la función que se aplica de último
«externa» tiene como dominio el conjunto de los números reales, debido a que el rango de
la que aplica primero «interna» será siempre un subconjunto de ℝ.

Ejemplo 2. Entre 𝑓𝑓(𝑥𝑥) = 𝑥𝑥 2 y 𝑔𝑔(𝑥𝑥) = 2𝑥𝑥 − 3 la compuesta 𝑔𝑔 ∘ 𝑓𝑓 está definida, debido a que:
𝐷𝐷𝐷𝐷𝐷𝐷(𝑔𝑔) ∩ 𝑅𝑅𝑅𝑅𝑅𝑅(𝑓𝑓) = ℝ ∩ [0, +∞) ≠ ∅ «ya que 𝐷𝐷𝐷𝐷𝐷𝐷(𝑓𝑓) = ℝ; 𝑅𝑅𝑅𝑅𝑅𝑅(𝑓𝑓) = [0, +∞) y 𝐷𝐷𝐷𝐷𝐷𝐷(𝑔𝑔) = 𝑅𝑅𝑅𝑅𝑅𝑅(𝑔𝑔) = ℝ»

El esquema muestra un modo de hallar la compuesta de la función 𝑓𝑓 «interna» mediante


la cual toda «𝑥𝑥» es transformada en su cuadrado «𝑥𝑥 2 », seguida de la función 𝑔𝑔 «externa» por
la cual toda «𝑥𝑥 2 » se transforma en su doble disminuido en 3
𝑓𝑓 𝑔𝑔
𝑥𝑥 𝑥𝑥 2 2𝑥𝑥 2 − 3
(𝑔𝑔 ∘ 𝑓𝑓)(𝑥𝑥)

Dicho esquema señala que (𝑔𝑔 ∘ 𝑓𝑓)(𝑥𝑥) = 𝑔𝑔[𝑓𝑓(𝑥𝑥)] = 2𝑥𝑥 2 − 3 que es la función compuesta
entre 𝑓𝑓 y 𝑔𝑔. Por otra parte, el modo algebraico de realizar la composición es como se indica:
(𝑔𝑔 ∘ 𝑓𝑓)(𝑥𝑥) = 𝑔𝑔[𝑓𝑓(𝑥𝑥)] «aplicación de la definición de composición»

= 𝑔𝑔(𝑥𝑥 2 ) «𝑓𝑓(𝑥𝑥) se sustituye por 𝑥𝑥 2 debido a que por su definición 𝑓𝑓(𝑥𝑥) = 𝑥𝑥 2 »

= 2𝑥𝑥 2 − 3 «se aplicó la regla que define a la función 𝑔𝑔, que es 𝑔𝑔(𝑥𝑥) = 2𝑥𝑥 − 3»

Luego (𝑔𝑔 ∘ 𝑓𝑓)(𝑥𝑥) = 2𝑥𝑥 2 − 3 o bien 𝑔𝑔[𝑓𝑓(𝑥𝑥)] = 2𝑥𝑥 2 − 3 «función compuesta requerida»

Como 𝑔𝑔[𝑓𝑓(𝑥𝑥)] = 2𝑥𝑥 2 − 3, entonces 𝐷𝐷𝐷𝐷𝐷𝐷(𝑔𝑔 ∘ 𝑓𝑓)(𝑥𝑥) = ℝ

CLEMENTE MORENO (MATERIAL DIDÁCTICO EN VALIDACIÓN 50


Del mismo modo (𝑓𝑓 ∘ 𝑔𝑔)(𝑥𝑥) existe, debido a que 𝐷𝐷𝐷𝐷𝐷𝐷(𝑓𝑓) ∩ 𝑅𝑅𝑅𝑅𝑅𝑅(𝑔𝑔) = ℝ ∩ ℝ = ℝ ≠ ∅
𝑔𝑔 𝑓𝑓
𝑥𝑥 2𝑥𝑥 − 3 (2𝑥𝑥 − 3)2 = 4𝑥𝑥 2 − 12𝑥𝑥 + 9
(𝑓𝑓 ∘ 𝑔𝑔)(𝑥𝑥)

El esquema se lee como en el caso anterior, aquí 𝑔𝑔 es la «interna», ella por definición a
cualquier «𝑥𝑥» le toma su doble y le quita 3, le sigue 𝑓𝑓 que es la «externa» y según su definición
a cualquier «𝑥𝑥» que en este caso es «2𝑥𝑥 − 3» le toma su cuadrado; el resultado de ello es la
función compuesta (𝑓𝑓 ∘ 𝑔𝑔)(𝑥𝑥) = 𝑓𝑓[𝑔𝑔(𝑥𝑥)] = 4𝑥𝑥 2 − 12𝑥𝑥 + 9

El cálculo algebraico de esta composición es como se indica a continuación

(𝑓𝑓 ∘ 𝑔𝑔)(𝑥𝑥) = 𝑓𝑓[𝑔𝑔(𝑥𝑥)] «aplicación de la definición de composición»

= 𝑓𝑓(2𝑥𝑥 − 3) «𝑔𝑔(𝑥𝑥) se sustituye por 2𝑥𝑥 − 3 debido a que por su definición 𝑔𝑔(𝑥𝑥) = 2𝑥𝑥 − 3»

= (2𝑥𝑥 − 3)2 = 4𝑥𝑥 2 − 12𝑥𝑥 + 9 «se aplicó la regla que define a la función 𝑓𝑓, que es 𝑓𝑓(𝑥𝑥) = 𝑥𝑥 2 »

Luego (𝑓𝑓 ∘ 𝑔𝑔)(𝑥𝑥) = 4𝑥𝑥 2 − 12𝑥𝑥 + 9 o bien 𝑓𝑓[𝑔𝑔(𝑥𝑥)] = 4𝑥𝑥 2 − 12𝑥𝑥 + 9 «función compuesta requerida»

Siendo (𝑓𝑓 ∘ 𝑔𝑔)(𝑥𝑥) = 4𝑥𝑥 2 − 12𝑥𝑥 + 9 entonces 𝐷𝐷𝐷𝐷𝐷𝐷(𝑓𝑓 ∘ 𝑔𝑔)(𝑥𝑥) = ℝ. Además, se puede
observar que (𝑔𝑔 ∘ 𝑓𝑓)(𝑥𝑥) ≠ (𝑓𝑓 ∘ 𝑔𝑔)(𝑥𝑥)

Ejemplo 3. Si 𝑓𝑓(𝑥𝑥) = √𝑥𝑥 y 𝑔𝑔(𝑥𝑥) = 𝑥𝑥 2 , ¿Cómo se definen 𝑔𝑔 ∘ 𝑓𝑓 y 𝑓𝑓 ∘ 𝑔𝑔, en caso de que


existan?
Dado que 𝐷𝐷𝐷𝐷𝐷𝐷(𝑓𝑓) = ℝ; 𝑅𝑅𝑅𝑅𝑅𝑅(𝑓𝑓) = [0, +∞); 𝐷𝐷𝐷𝐷𝐷𝐷(𝑔𝑔) = ℝ y 𝑅𝑅𝑅𝑅𝑅𝑅(𝑔𝑔) = [0, +∞) entonces:
𝐷𝐷𝐷𝐷𝐷𝐷(𝑔𝑔) ∩ 𝑅𝑅𝑅𝑅𝑅𝑅(𝑓𝑓) = ℝ ∩ [0, +∞) = [0, +∞) ≠ ∅ «la composición (𝑔𝑔 ∘ 𝑓𝑓)(𝑥𝑥) se puede realizar»

Y, (𝑔𝑔 ∘ 𝑓𝑓)(𝑥𝑥) = 𝑔𝑔[𝑓𝑓(𝑥𝑥)] «aplicando de la definición de composición»

= 𝑓𝑓(√𝑥𝑥) «𝑓𝑓(𝑥𝑥) se sustituye por √𝑥𝑥 debido a que por su definición 𝑓𝑓(𝑥𝑥) = √𝑥𝑥»

= (√𝑥𝑥)2 «se aplicó la regla que define a la función 𝑔𝑔, que es 𝑔𝑔(𝑥𝑥) = 𝑥𝑥 2 »

Luego (𝑔𝑔 ∘ 𝑓𝑓)(𝑥𝑥) = (√𝑥𝑥)2 o bien 𝑔𝑔[𝑓𝑓(𝑥𝑥)] = (√𝑥𝑥)2 «función compuesta requerida»

CLEMENTE MORENO (MATERIAL DIDÁCTICO EN VALIDACIÓN 51


Aquí (𝑔𝑔 ∘ 𝑓𝑓)(𝑥𝑥) = (√𝑥𝑥)2 o bien 𝑔𝑔[𝑓𝑓(𝑥𝑥)] = (√𝑥𝑥)2 , implica que 𝐷𝐷𝐷𝐷𝐷𝐷(𝑔𝑔 ∘ 𝑓𝑓) = [0, +∞) (1)

Asimismo, como 𝐷𝐷𝐷𝐷𝐷𝐷(𝑓𝑓) ∩ 𝑅𝑅𝑅𝑅𝑅𝑅(𝑔𝑔) = [0, +∞) ∩ [0, +∞) = [0, +∞) ≠ ∅ la composición
(𝑓𝑓 ∘ 𝑔𝑔)(𝑥𝑥) también se puede realizar, entonces:

(𝑓𝑓 ∘ 𝑔𝑔)(𝑥𝑥) = 𝑓𝑓[𝑔𝑔(𝑥𝑥)] «aplicando de la definición de composición»

= 𝑓𝑓(𝑥𝑥 2 ) «𝑔𝑔(𝑥𝑥) se sustituye por 𝑥𝑥 2 debido a que por su definición 𝑔𝑔(𝑥𝑥) = 𝑥𝑥 2 »

= √𝑥𝑥 2 «se aplicó la regla que define a la función 𝑓𝑓, que es 𝑓𝑓(𝑥𝑥) = √𝑥𝑥»

Luego, (𝑓𝑓 ∘ 𝑔𝑔)(𝑥𝑥) = 𝑓𝑓[𝑔𝑔(𝑥𝑥)] = |𝑥𝑥| «función compuesta requerida. Por definición √𝑥𝑥 2 = |𝑥𝑥|»

Aquí (𝑓𝑓 ∘ 𝑔𝑔)(𝑥𝑥) = √𝑥𝑥 2 o bien (𝑓𝑓 ∘ 𝑔𝑔)(𝑥𝑥) = √𝑥𝑥 2 , implica que el 𝐷𝐷𝐷𝐷𝐷𝐷(𝑓𝑓 ∘ 𝑔𝑔) = ℝ (2)

COMENTARIOS

Algebraicamente, las reglas de correspondencia de (𝑔𝑔 ∘ 𝑓𝑓)(𝑥𝑥) = (√𝑥𝑥)2 y (𝑓𝑓 ∘ 𝑔𝑔)(𝑥𝑥) = √𝑥𝑥 2
parecen equivalentes, sin embargo lo indicado en (1) y (2) respecto a los dominios de estas
dos funciones, muestran claramente que dichas funciones son absolutamente distintas

Ejemplo 4. Entre 𝑓𝑓(𝑥𝑥) = √−𝑥𝑥 − 1 y 𝑔𝑔(𝑥𝑥) = √𝑥𝑥 − 1 hallar 𝑔𝑔 ∘ 𝑓𝑓 y 𝑓𝑓 ∘ 𝑔𝑔 en caso de que existan
Dado que 𝐷𝐷𝐷𝐷𝐷𝐷(𝑓𝑓) = (−∞, −1]; 𝑅𝑅𝑅𝑅𝑅𝑅(𝑓𝑓) = [0, +∞); 𝐷𝐷𝐷𝐷𝐷𝐷(𝑔𝑔) = [1, +∞) y 𝑅𝑅𝑅𝑅𝑅𝑅(𝑔𝑔) = [0, +∞)
entonces:
𝐷𝐷𝐷𝐷𝐷𝐷(𝑔𝑔) ∩ 𝑅𝑅𝑅𝑅𝑅𝑅(𝑓𝑓) = [1, +∞) ∩ [0, +∞) = [1, +∞) ≠ ∅ «la composición (𝑔𝑔 ∘ 𝑓𝑓)(𝑥𝑥) se puede realizar» y

(𝑔𝑔 ∘ 𝑓𝑓)(𝑥𝑥) = 𝑔𝑔[𝑓𝑓(𝑥𝑥)] «aplicando de la definición de composición»

= 𝑔𝑔(√−𝑥𝑥 − 1) «𝑓𝑓(𝑥𝑥) se sustituye por √−𝑥𝑥 − 1 debido a que por su definición 𝑓𝑓(𝑥𝑥) = √−𝑥𝑥 − 1»

= �√−𝑥𝑥 − 1 − 1 «se aplicó la regla que define a la función 𝑔𝑔, que es 𝑔𝑔(𝑥𝑥) = √𝑥𝑥 − 1»

Luego (𝑔𝑔 ∘ 𝑓𝑓)(𝑥𝑥) = ��−(𝑥𝑥 + 1) − 1 «es la función compuesta requerida»

Aquí, (𝑔𝑔 ∘ 𝑓𝑓)(𝑥𝑥) es un número real si �−(𝑥𝑥 + 1) − 1 ≥ 0 «inecuación que conduce al dominio»

CLEMENTE MORENO (MATERIAL DIDÁCTICO EN VALIDACIÓN 52


Así �−(𝑥𝑥 + 1) ≥ 1 o bien −(𝑥𝑥 + 1) ≥ 1 «trasponiendo términos y elevando al cuadrado»

Y, (𝑥𝑥 + 1) ≤ −1 o bien 𝑥𝑥 ≤ −2 «al multiplicar por (−1) la desigualdad se invierte, luego se despeja»

Luego, 𝐷𝐷𝐷𝐷𝐷𝐷(𝑔𝑔 ∘ 𝑓𝑓) = (−∞, −2] «cálculo del dominio de la función compuesta (𝑔𝑔 ∘ 𝑓𝑓)»

Por su parte, (𝑓𝑓 ∘ 𝑔𝑔)(𝑥𝑥) no existe, pues 𝐷𝐷𝐷𝐷𝐷𝐷(𝑓𝑓) ∩ 𝑅𝑅𝑅𝑅𝑅𝑅(𝑔𝑔) = (−∞, −1] ∩ [1, +∞) = ∅

En la composición de funciones, un problema interesante consiste en encontrar la función


interna o la función externa a partir de la función compuesta

Ejemplo 1. Siendo 𝑓𝑓[𝑔𝑔(𝑥𝑥)] = 4𝑥𝑥 2 − 12𝑥𝑥 + 9 y 𝑓𝑓(𝑥𝑥) = 𝑥𝑥 2 la función externa, entonces la función
interna es

𝑓𝑓(𝑥𝑥) = 𝑥𝑥 2 implica 𝑓𝑓[𝑔𝑔(𝑥𝑥)] = [𝑔𝑔(𝑥𝑥)]2 (1) «si 𝑓𝑓 transforma «𝑥𝑥» en «𝑥𝑥 2 », «𝑔𝑔(𝑥𝑥)» se transforma en «[𝑔𝑔(𝑥𝑥)]2 »»

Y, 𝑓𝑓[𝑔𝑔(𝑥𝑥)] = 4𝑥𝑥 2 − 12𝑥𝑥 + 9 (2) «(2) es el modo en que fue definida la compuesta»

Así, [𝑔𝑔(𝑥𝑥)]2 = 4𝑥𝑥 2 − 12𝑥𝑥 + 9 «igualando las identidades (1) y (2)»

Luego, 𝑔𝑔(𝑥𝑥) = √4𝑥𝑥 2 − 12𝑥𝑥 + 9 «es la función interna requerida»

Ejemplo 2. Siendo 𝑓𝑓[𝑔𝑔(𝑥𝑥)] = 4𝑥𝑥 2 − 12𝑥𝑥 + 9 y 𝑓𝑓(𝑥𝑥) = 𝑥𝑥 2 + 3𝑥𝑥 + 1, ¿Quién es 𝑔𝑔(𝑥𝑥)?

𝑓𝑓(𝑥𝑥) = 𝑥𝑥 2 + 3𝑥𝑥 + 1 implica 𝑓𝑓[𝑔𝑔(𝑥𝑥)] = [𝑔𝑔(𝑥𝑥)]2 + 3[𝑔𝑔(𝑥𝑥)] + 1 «aplicado definición de 𝑓𝑓(𝑥𝑥)»

Así, [𝑔𝑔(𝑥𝑥)]2 + 3[𝑔𝑔(𝑥𝑥)] + 1 = 𝑥𝑥 2 + 3𝑥𝑥 + 1 «por igualación de imágenes de 𝑓𝑓»

9 9
Y, [𝑔𝑔(𝑥𝑥)]2 + 3[𝑔𝑔(𝑥𝑥)] + + 1 − = 𝑥𝑥 2 + 3𝑥𝑥 + 1 «completando cuadrados en el primer lado de la igualdad»
4 4

3 2 5
Así, �𝑔𝑔(𝑥𝑥) + � − = 𝑥𝑥 2 + 3𝑥𝑥 + 1 «factorizando el primer lado de la igualdad»
2 4

3 2 9
Y, �𝑔𝑔(𝑥𝑥) + � = 𝑥𝑥 2 + 3𝑥𝑥 + 5
«trasponiendo − 4 al segundo lado de la igualdad y operando»
2 4

3 4𝑥𝑥 2 + 12𝑥𝑥 + 9
Así, 𝑔𝑔(𝑥𝑥) + = � «tomando raíz en ambos lados y resolviendo suma de fracciones»
2 4

√4𝑥𝑥 2 + 12𝑥𝑥 + 9 3
Luego, 𝑔𝑔(𝑥𝑥) = − «despejando 𝑔𝑔(𝑥𝑥) y extrayendo 4 de la raíz»
2 2

√4𝑥𝑥 2 + 12𝑥𝑥 + 9
En consecuencia, 𝑔𝑔(𝑥𝑥) = «es la función interna requerida»
2

CLEMENTE MORENO (MATERIAL DIDÁCTICO EN VALIDACIÓN 53


Ejemplo 3. Dada 𝑓𝑓[𝑔𝑔(𝑥𝑥)] = √𝑥𝑥 2 + 1 y la función interna 𝑔𝑔(𝑥𝑥) = 𝑥𝑥 − 1 ¿Quién 𝑓𝑓(𝑥𝑥)?
Si 𝑢𝑢 = 𝑔𝑔(𝑥𝑥) entonces 𝑢𝑢 = 𝑥𝑥 − 1 «redefinición de 𝑔𝑔(𝑥𝑥)»

Así, 𝑥𝑥 = 𝑢𝑢 + 1 y 𝑓𝑓(𝑥𝑥) = 𝑓𝑓(𝑢𝑢 + 1) «la evaluación de 𝑓𝑓 en 𝑥𝑥 y 𝑢𝑢 + 1 debe ser igual dado que 𝑥𝑥 = 𝑢𝑢 + 1»

Y, 𝑓𝑓(𝑥𝑥) = 𝑓𝑓(𝑢𝑢 + 1) = �(𝑢𝑢 + 1)2 + 1 = √𝑢𝑢2 + 2𝑢𝑢 + 2 «sustituyendo y operando»

Luego 𝑓𝑓(𝑥𝑥) = √𝑥𝑥 2 + 2𝑥𝑥 + 2 «es la función externa requerida»

En efecto
𝑓𝑓[𝑔𝑔(𝑥𝑥)] = 𝑓𝑓(𝑥𝑥 − 1) «𝑔𝑔(𝑥𝑥) se sustituyó por 𝑥𝑥 − 1, debido a que es la definición de 𝑔𝑔»

= �(𝑥𝑥 − 1)2 + 2(𝑥𝑥 − 1) + 2 «se aplicó la definición de 𝑓𝑓»

= √𝑥𝑥 2 − 2𝑥𝑥 + 1 + 2𝑥𝑥 − 2 + 2 «ejecutando operaciones»

Luego 𝑓𝑓[𝑔𝑔(𝑥𝑥)] = √𝑥𝑥 2 + 1 «esto evidencia que 𝑓𝑓(𝑥𝑥) = √𝑥𝑥 2 + 2𝑥𝑥 + 2 es la función requerida»

Ejemplo 4. Siendo 𝑓𝑓[𝑔𝑔(𝑥𝑥)] = √𝑥𝑥 2 + 2 y 𝑔𝑔(𝑥𝑥) = 3𝑥𝑥 + 2, ¿Quién es 𝑓𝑓(𝑥𝑥)?


𝑢𝑢 − 2
Si 𝑢𝑢 = 𝑔𝑔(𝑥𝑥) entonces 𝑢𝑢 = 3𝑥𝑥 + 2 y 𝑥𝑥 = «se realizó un cambio de variable»
3

𝑢𝑢−2 𝑢𝑢−2 2
Así, 𝑓𝑓(𝑥𝑥) = 𝑓𝑓 � � = �� � +2 «se evaluó a 𝑓𝑓 en la nueva variable»
3 3

𝑢𝑢2 −4𝑢𝑢+4
=� +2 «desarrollando el producto notable»
9

√𝑢𝑢2 −4𝑢𝑢+22
= «ejecutando operaciones»
3

√𝑥𝑥 2 −4𝑥𝑥+22
Luego 𝑓𝑓(𝑥𝑥) = «es la función externa requerida»
3

En efecto
𝑓𝑓[𝑔𝑔(𝑥𝑥)] = 𝑓𝑓(3𝑥𝑥 + 2) «𝑔𝑔(𝑥𝑥) se sustituyó por 3𝑥𝑥 + 2, debido a que es la definición de 𝑔𝑔»

�(3𝑥𝑥+2)2 −4(3𝑥𝑥+2)+22
= «se aplicó la definición de 𝑓𝑓»
3

√9𝑥𝑥 2 +12𝑥𝑥+4−12𝑥𝑥−8+22
= «ejecutando operaciones»
3

√9𝑥𝑥 2 +18 3√𝑥𝑥 2 +2


= = «operando, tomando 9 como factor común se extrajo del radial»
3 3

Luego 𝑓𝑓[𝑔𝑔(𝑥𝑥)] = √𝑥𝑥 2 + 2 «esto evidencia que 𝑓𝑓(𝑥𝑥) =


√𝑥𝑥 2 − 4𝑥𝑥 + 22
3
es la función requerida»

CLEMENTE MORENO (MATERIAL DIDÁCTICO EN VALIDACIÓN 54


Ejercicio N° 𝟕𝟕
Dadas las funciones 𝑓𝑓(𝑥𝑥) = 𝑥𝑥 2 + 2𝑥𝑥 − 1 y 𝑔𝑔(𝑥𝑥) = 3𝑥𝑥 + 5, ¿Quienes son las imágenes de?

1) (𝑔𝑔 ∘ 𝑓𝑓)(2); 2) (𝑓𝑓 ∘ 𝑔𝑔)(2); 3) (𝑔𝑔 ∘ 𝑓𝑓)(𝑥𝑥);


1 1 6) (𝑓𝑓 ∘ 𝑔𝑔)(𝑥𝑥)
4) (𝑔𝑔 ∘ 𝑓𝑓) � �; 5) (𝑓𝑓 ∘ 𝑔𝑔) � �;
3 3

Para 𝑓𝑓(𝑥𝑥) = √1 − 𝑥𝑥 2 ; 𝑔𝑔(𝑥𝑥) = √2𝑥𝑥 − 3 y ℎ(𝑥𝑥) = √𝑥𝑥 2 + 1 de ser posible, halle:


7) (𝑓𝑓 ∘ 𝑓𝑓)(𝑥𝑥); 8) (𝑓𝑓 ∘ 𝑔𝑔)(𝑥𝑥); 9) (𝑔𝑔 ∘ 𝑓𝑓)(𝑥𝑥);

10) (𝑓𝑓 ∘ ℎ)(𝑥𝑥); 11) (ℎ ∘ 𝑓𝑓)(𝑥𝑥); 12) (𝑔𝑔 ∘ ℎ)(𝑥𝑥);

13) Siendo (𝑔𝑔 ∘ 𝑓𝑓)(𝑥𝑥) = √3𝑥𝑥 2 + 1 y 𝑓𝑓(𝑥𝑥) = √3𝑥𝑥 − 2 ¿Quién es 𝑔𝑔(𝑥𝑥)?

14) Siendo (𝑔𝑔 ∘ 𝑓𝑓)(𝑥𝑥) = √𝑥𝑥 2 − 1 y 𝑔𝑔(𝑥𝑥) = 𝑥𝑥 2 + 1, ¿Quién es 𝑓𝑓(𝑥𝑥)?

Funciones racionales
1
Iniciemos con la función 𝑓𝑓(𝑥𝑥) = (∗) «a todo número real 𝑥𝑥 ≠ 0, le asigna su inverso»
𝑥𝑥

Nótese que la función 𝑓𝑓 dada en (∗) es el cociente entre la función constante 𝑓𝑓1 (𝑥𝑥) = 1
y la función idéntica 𝑓𝑓2 (𝑥𝑥) = 𝑥𝑥, en tales condiciones se tiene:

𝐷𝐷𝐷𝐷𝐷𝐷(𝑓𝑓) = 𝐷𝐷𝐷𝐷𝐷𝐷(𝑓𝑓1 ) ∩ 𝐷𝐷𝐷𝐷𝐷𝐷(𝑓𝑓2 )~{𝑥𝑥 = 0} «por definición de dominio de la función cociente. Ver pág. 8»

𝐷𝐷𝐷𝐷𝐷𝐷(𝑓𝑓) = ℝ ∩ ℝ~{0} así 𝐷𝐷𝐷𝐷𝐷𝐷(𝑓𝑓) = ℝ~{0} «𝑓𝑓 se puede evaluar en cualquier número real menos en cero»

«la gráfica del 𝐷𝐷𝐷𝐷𝐷𝐷(𝑓𝑓) con la perforación en «0» indica su ausencia»


)∘( ℝ
−∞ 0 +∞ «𝐷𝐷𝐷𝐷𝐷𝐷(𝑓𝑓) = ℝ~{0} = (−∞, 0) ∪ (0, +∞), otra forma de escribir el Dom.»

Establecido el dominio, se pueden elegir los puntos «de muestra» para construir la tabla y
posterior representación gráfica de la función. En este caso, se recomienda tomar cuatro grupos
de números para diseñar la tabla. En el primero, se pueden tomar números pequeños, digamos
−1000, −100, −10 y −1; en el segundo números próximos a cero por la izquierda, tales como
−0,5, −0,1, −0,01 y −0,001; en el tercero números próximos a cero por la derecha, digamos 0,5,

CLEMENTE MORENO (MATERIAL DIDÁCTICO EN VALIDACIÓN 55


0,1, 0,01 y 0,001; en el cuarto, números grandes, digamos 1, 10, 100, 100. Así, el estudio tabular
se resume en la tabla adjunta «el lector debe sustituir en la función y hacer los cálculos»

𝑥𝑥 −1000 −100 −10 −1 −0,5 −0,1 −0,01 −0,001 0 0,001 0,01 0,1 0,5 1 10 100 1000

−0,001 −0,01 −0,1 −100 −1000 ? 1000 100 10 2 1 0,1 0,01 0,001
𝑦𝑦 −1 −2 −10

Nótese que en la medida que los números «𝑥𝑥» pequeños «primer grupo» se alejan de
«0» las imágenes «𝑦𝑦» se acercan a cero a través de valores negativos, pero cuando estos se
aproximan a «0» por la izquierda «segundo grupo» las imágenes «𝑦𝑦» decrecen. Por su parte,
cuando los números «𝑥𝑥» se aproximan a «0» por la derecha «tercer grupo» las imágenes «𝑦𝑦»
crecen, finalmente en la medida que los números «𝑥𝑥» grandes «cuarto grupo» se alejan de «0»
las imágenes «𝑦𝑦» se acercan a cero a través de valores positivos

1
Como es habitual, para el estudio gráfico de 𝑓𝑓(𝑥𝑥) = se ubican los puntos de la tabla
𝑥𝑥

en el plano cartesiano, solo que esta vez tendremos que utilizar la imaginación, debido a la
dificultad que implica elegir una escala que englobe a la vez tanto números próximos a «0»
como números alejados «0». Sin embargo, la gráfica de 𝑓𝑓 es una curva hiperbólica como la
indicada en la figura

CLEMENTE MORENO (MATERIAL DIDÁCTICO EN VALIDACIÓN 56


Mirando la gráfica, es fácil ver que cuando «𝑥𝑥» tiende a «−∞», en símbolos «𝑥𝑥 → −∞»
las imágenes de la función tienden a «0», en símbolos «𝑓𝑓(𝑥𝑥) → 0»; asimismo, cuando «𝑥𝑥»
tiende a «+∞», en símbolos «𝑥𝑥 → +∞» las imágenes de la función tiende a «0», en símbolos
«𝑓𝑓(𝑥𝑥) → 0». Por su parte, cuando «𝑥𝑥» tiende a «0» por la izquierda, en símbolos «𝑥𝑥 → 0− » las
imágenes de la función tienden a «−∞», en símbolos «𝑓𝑓(𝑥𝑥) → −∞» y cuando «𝑥𝑥» tiende a «0»
por la derecha, en símbolos «𝑥𝑥 → 0+ » las imágenes de la función tienden a «+∞», en símbolos
«𝑓𝑓(𝑥𝑥) → +∞»

En la gráfica también se ve que el 𝑅𝑅𝑅𝑅𝑅𝑅(𝑓𝑓) = ℝ~{0} o bien 𝑅𝑅𝑅𝑅𝑅𝑅(𝑓𝑓) = (−∞, 0) ∪ (0, +∞).
Sin embargo, éste puede calcularse a partir del dominio de la inversa de 𝑓𝑓, toda vez que la
1
función 𝑓𝑓(𝑥𝑥) = es inyectiva
𝑥𝑥

COMENTARIO

Un modo intuitivo de saber que la función es inyectiva, consiste en imaginarse una recta
horizontal que corte la curva de la función, si tal recta intersecta a la curva una sola vez, la
función es inyectiva. La inyectividad se prueba de manera algebraica, pero acá la omitiremos
debido a que requiere un apartado teórico «estudiado en cursos anteriores» donde tal prueba
se sustente, cuya evocación por razones de tiempo no abordaremos en este curso

De regreso en la determinación de la inversa, se tiene:


1
Como 𝑦𝑦 = 𝑓𝑓(𝑥𝑥) entonces 𝑦𝑦 = «forma alterna de escribir la función»
𝑥𝑥
1 1
𝑦𝑦 = implica 𝑥𝑥 ∙ 𝑦𝑦 = 1 o bien 𝑥𝑥 = «se despejó «𝑥𝑥», función escrita en términos de «𝑥𝑥»»
𝑥𝑥 𝑦𝑦

1
𝑓𝑓 −1 (𝑥𝑥) = «se escribe la inversa intercambiando los papeles de «𝑥𝑥» y «𝑦𝑦»»
𝑥𝑥

𝐷𝐷𝐷𝐷𝐷𝐷(𝑓𝑓 −1 ) = ℝ~{0} implica 𝑅𝑅𝑅𝑅𝑅𝑅(𝑓𝑓) = ℝ~{0} «Dominio de la inversa 𝑓𝑓 −1 es el rango de la directa 𝑓𝑓»

CLEMENTE MORENO (MATERIAL DIDÁCTICO EN VALIDACIÓN 57


COMENTARIO

1
El estudio realizado a la función 𝑓𝑓(𝑥𝑥) = se puede extender a cualquier función racional
𝑥𝑥
𝑓𝑓1 (𝑥𝑥)
𝑓𝑓(𝑥𝑥) = , donde 𝑓𝑓1 (𝑥𝑥) y 𝑓𝑓2 (𝑥𝑥) son funciones polinomicas de grados mayores o iguales a uno,
𝑓𝑓2 (𝑥𝑥)

acá por la carencia de las herramientas del cálculo «rama de la matemática», limitaremos el
estudio hasta polinomios de segundo grado

𝑥𝑥 + 1
Ejemplo 1. Al realizar el estudio completo de la función 𝑓𝑓(𝑥𝑥) = se tiene que:
𝑥𝑥 − 1

(i) Determinación del dominio de 𝑓𝑓


Sean 𝑓𝑓1 (𝑥𝑥) = 𝑥𝑥 + 1 y 𝑓𝑓2 (𝑥𝑥) = 𝑥𝑥 − 1 «identificación de las funciones que componen a 𝑓𝑓(𝑥𝑥)»

𝐷𝐷𝐷𝐷𝐷𝐷(𝑓𝑓1 ) = ℝ y 𝐷𝐷𝐷𝐷𝐷𝐷(𝑓𝑓2 ) = ℝ «determinación del dominio de las funciones que componen a 𝑓𝑓(𝑥𝑥)»

𝐷𝐷𝐷𝐷𝐷𝐷(𝑓𝑓) = 𝐷𝐷𝐷𝐷𝐷𝐷(𝑓𝑓1 ) ∩ 𝐷𝐷𝐷𝐷𝐷𝐷(𝑓𝑓2 )~{𝑓𝑓2 (𝑥𝑥) = 0} «adaptación de la definición»

𝐷𝐷𝐷𝐷𝐷𝐷(𝑓𝑓) = ℝ ∩ ℝ~{𝑥𝑥 − 1 = 0} «sustitución de valores»

𝐷𝐷𝐷𝐷𝐷𝐷(𝑓𝑓) = ℝ~{1} o bien 𝐷𝐷𝐷𝐷𝐷𝐷(𝑓𝑓) = (−∞, 1) ∪ (1, +∞) «intersectando ℝ con ℝ y resolviendo 𝑥𝑥 − 1 = 0»

(ii) Cortes con los ejes


0+1
Si 𝑥𝑥 = 0 se tiene 𝑓𝑓(0) = = −1 así 𝑦𝑦 = −1 y P1 = (0, −1) «la curva corta al eje «𝑦𝑦» en «−1»»
0−1

Si 𝑦𝑦 = 0 se tiene 𝑥𝑥 + 1 = 0 así 𝑥𝑥 = −1 y P2 = (−1,0) «la curva corta al eje «𝑥𝑥» en «−1»»

Nota. Solo el numerador se iguala a «0» pues el dominio de la función retiro del denominador
el número «𝑥𝑥 = 1» que lo hacía «0»
(iii) Estudio tabular y representación gráfica de la función
En la elección de los números para construir la tabla y dibujo de la gráfica se pueden seguir
1
las sugerencias dadas en el estudio de 𝑓𝑓(𝑥𝑥) = . En tal sentido, se tiene:
𝑥𝑥

CLEMENTE MORENO (MATERIAL DIDÁCTICO EN VALIDACIÓN 58


𝑥𝑥 −10 −1 0 0,5 0,9 0,99 1 1,01 1,1 1,5 2 10

𝑦𝑦 0,82 0 −1 −3 −19 −199 ? 201 21 5 3 1,22

(iv) Determinación del rango


𝑥𝑥+1
𝑦𝑦 = implica 𝑦𝑦(𝑥𝑥 − 1) = 𝑥𝑥 + 1 «se multiplicó por «𝑥𝑥 − 1» ambos lados para quitar denominador»
𝑥𝑥−1

𝑦𝑦𝑦𝑦 − 𝑦𝑦 = 𝑥𝑥 + 1 implica 𝑦𝑦𝑦𝑦 − 𝑥𝑥 = 𝑦𝑦 + 1 «se operó y reunió los términos con «𝑥𝑥» en 1er lado de la igualdad»

𝑦𝑦+1
𝑥𝑥(𝑦𝑦 − 1) = 𝑦𝑦 + 1 implica 𝑥𝑥 = «se tomó «𝑥𝑥» como factor común y luego se despejó»
𝑦𝑦−1

𝑥𝑥+1
𝑓𝑓 −1 (𝑥𝑥) = «se escribió la inversa 𝑓𝑓 −1 e intercambiaron los papeles de «𝑥𝑥» y «𝑦𝑦»»
𝑥𝑥−1

𝐷𝐷𝐷𝐷𝐷𝐷(𝑓𝑓 −1 ) = ℝ~{1} implica 𝑅𝑅𝑅𝑅𝑅𝑅(𝑓𝑓) = ℝ~{1} «Dominio de la inversa 𝑓𝑓 −1 es el rango de la directa 𝑓𝑓»

COMENTARIOS

(i) En la gráfica puedes observar dos rectas perpendiculares discontinuas «punteadas, excepto
1
si coinciden con los ejes rectangulares, como ocurre en 𝑓𝑓(𝑥𝑥) = » llamadas asíntotas, cuya
𝑥𝑥

función es orientar el trazado de la curva, debido a que éstas no se dejan cruzar por las
curvas «excepto algunas horizontales». Las asíntotas pueden ser verticales, horizontales y
oblicuas, pero en una curva sólo pueden existir horizontales y verticales u oblicuas y
verticales, nunca las tres, pues la horizontal con pendiente «𝑚𝑚 = 0» es un caso particular de
la oblicua cuya pendiente «𝑚𝑚 ≠ 0». Sin embargo, las curvas a tratar en este curso solo
tendrán asíntotas verticales y/o horizontales
(ii) La asíntota vertical está presente en todos aquellos números que anulan el denominador de
la función «razón por la que se excluyen del dominio de la función»; en el estudio tabular se

CLEMENTE MORENO (MATERIAL DIDÁCTICO EN VALIDACIÓN 59


intuyen al observar que cuando «𝑥𝑥» tiende a un número «𝑥𝑥 → 𝑎𝑎» por la izquierda y/o por la
derecha, las imágenes de la función tienden a «𝑓𝑓(𝑥𝑥) → ∞» esto es a «−∞» y/o «+∞»
(iii) La asíntota horizontal está presente en los números excluidos del rango, pero en el estudio
tabular se intuyen al observar que cuando «𝑥𝑥» tiende a «−∞» y/o «+∞» las imágenes de la
función tienden a un número «𝑥𝑥 → 𝑎𝑎»

𝑥𝑥
Ejemplo 2. Al realizar el estudio completo de la función 𝑓𝑓(𝑥𝑥) = se tiene que:
𝑥𝑥 2 − 4

(i) Determinación del dominio de 𝑓𝑓


Sean 𝑓𝑓1 (𝑥𝑥) = 𝑥𝑥 y 𝑓𝑓2 (𝑥𝑥) = 𝑥𝑥 2 − 4 «identificación de las funciones que componen a 𝑓𝑓(𝑥𝑥)»

𝐷𝐷𝐷𝐷𝐷𝐷(𝑓𝑓1 ) = ℝ y 𝐷𝐷𝐷𝐷𝐷𝐷(𝑓𝑓2 ) = ℝ «determinación del dominio de las funciones que componen a 𝑓𝑓(𝑥𝑥)»

𝑓𝑓2 (𝑥𝑥) = 0 implica 𝑥𝑥 2 − 4 = 0 o bien 𝑥𝑥 = −2 y 𝑥𝑥 = 2 «números a excluir del denominador de 𝑓𝑓»

𝐷𝐷𝐷𝐷𝐷𝐷(𝑓𝑓) = 𝐷𝐷𝐷𝐷𝐷𝐷(𝑓𝑓1 ) ∩ 𝐷𝐷𝐷𝐷𝐷𝐷(𝑓𝑓2 )~{𝑓𝑓2 (𝑥𝑥) = 0} «adaptación de la definición»

𝐷𝐷𝐷𝐷𝐷𝐷(𝑓𝑓) = ℝ ∩ ℝ~{−2, 2} «sustitución de valores»

𝐷𝐷𝐷𝐷𝐷𝐷(𝑓𝑓) = ℝ~{−2,2} o bien 𝐷𝐷𝐷𝐷𝐷𝐷(𝑓𝑓) = (−∞, −2) ∪ (−2,2) ∪ (2, +∞) «especificación del dominio»

(ii) Cortes con los ejes rectangulares


0
Si 𝑥𝑥 = 0 se tiene 𝑓𝑓(0) = = 0 así 𝑦𝑦 = 0 y P1 = (0,0) «la curva corta al eje «𝑦𝑦» en «0»»
02 −4

Si 𝑦𝑦 = 0 se tiene 𝑥𝑥 = 0 así P2 = (0, 0) «la curva corta al eje «𝑥𝑥» en «0»

(iii) Estudio tabular y representación gráfica de la función


En atención al dominio de 𝑓𝑓 la muestra de números para el estudio tabular y gráfico consta
de tres grupos a saber. El primero viene desde números muy pequeños alejados de −2
hasta números próximos a −2 por la izquierda, digamos −10, −3, −2,5, −2,1, −2,01; el
segundo va desde números próximos a −2 por la derecha hasta números próximos a 2 por
la izquierda, digamos −1,99, −1,9, −1,5, −1, 0, 1, 1,5, 1,9, 1,99; el tercero va desde números
próximos a 2 por la derecha hasta números grandes alejados de 2, digamos 2,01, 2,1 2,5, 3,
10. Tal como se muestra en la tabla adjunta

CLEMENTE MORENO (MATERIAL DIDÁCTICO EN VALIDACIÓN 60


𝑥𝑥 −10 −3 −2,5 −2,1 −2,01 −2 −1,99 −1,9 −1,5 −1 0 1 1,5 1,9 1,99 2 2,01 2,1 2,5 3 10

𝑦𝑦 −0,104 −0,6 −1,11 −5,12 −50,12 ? 49,87 4,87 0,86 0,33 0 −0,33 −0,86 −4,87 −49,87 ? 50,12 5,12 1,11 0,6 0,104

La correspondiente representación gráfica se muestra en la figura adjunta

En la gráfica se observa una asíntota horizontal de ecuación 𝑦𝑦 = 0 y dos asíntotas


verticales de ecuaciones 𝑥𝑥 = −2 y 𝑥𝑥 = 2 .
En efecto, cuando 𝑥𝑥 → −∞ «𝑥𝑥 tiende a −∞» y 𝑥𝑥 → +∞ «𝑥𝑥 tiende a +∞» las imágenes
de 𝑓𝑓(𝑥𝑥) se aproximan a «0» es decir «𝑓𝑓(𝑥𝑥) → 0», luego la recta 𝑦𝑦 = 0 es una asíntota horizontal
de la curva; asimismo, cuando 𝑥𝑥 → −2− «𝑥𝑥 tiende a −2 por la izquierda», las imágenes de la
función decrecen sin fin, es decir «𝑓𝑓(𝑥𝑥) → −∞» y cuando 𝑥𝑥 → −2+ «𝑥𝑥 tiende a −2 por la
derecha», las imágenes de la función crecen sin fin, es decir «𝑓𝑓(𝑥𝑥) → +∞», por tal razón, la
recta 𝑥𝑥 = −2 es una asíntota vertical de la curva de la función; finalmente, cuando 𝑥𝑥 → 2− «𝑥𝑥
tiende a 2 por la izquierda», las imágenes de la función decrecen sin fin, es decir «𝑓𝑓(𝑥𝑥) → −∞»
y cuando 𝑥𝑥 → 2+ «𝑥𝑥 tiende a 2 por la derecha», las imágenes de la función crecen sin fin, es
decir «𝑓𝑓(𝑥𝑥) → +∞», por tal razón, la recta 𝑥𝑥 = 2 es una asíntota vertical de la curva de la función

CLEMENTE MORENO (MATERIAL DIDÁCTICO EN VALIDACIÓN 61


Ejemplificados y analizados tanto el modo de representación tabular como el modo de
𝑓𝑓1 (𝑥𝑥)
representación gráfico de las funciones racionales 𝑓𝑓(𝑥𝑥) = donde 𝑓𝑓1 (𝑥𝑥) y 𝑓𝑓2 (𝑥𝑥) son
𝑓𝑓2 (𝑥𝑥)

funciones como las estudiadas en el capítulo, en lo que sigue se proponen otros ejemplos que
ilustran el modo de determinar el dominio de esas funciones, dejando el resto del estudio como
actividad para el estudiante
√3𝑥𝑥 − 1
Ejemplo 1. Hallar el dominio de la función 𝑓𝑓(𝑥𝑥) = implica que:
𝑥𝑥 2 − 1

Sean 𝑓𝑓1 (𝑥𝑥) = √3𝑥𝑥 − 1 y 𝑓𝑓2 (𝑥𝑥) = 𝑥𝑥 2 − 1 «identificación de las funciones que componen a 𝑓𝑓(𝑥𝑥)»

Y, 𝐷𝐷𝐷𝐷𝐷𝐷(𝑓𝑓) = 𝐷𝐷𝐷𝐷𝐷𝐷(𝑓𝑓1 ) ∩ 𝐷𝐷𝐷𝐷𝐷𝐷(𝑓𝑓2 )~{𝑓𝑓2 (𝑥𝑥) = 0} (∗) «aplicación de la definición»

𝑓𝑓1 (𝑥𝑥) es un número real si 3𝑥𝑥 − 1 ≥ 0 «planteamiento de inecuación que lleva a 𝐷𝐷𝐷𝐷𝐷𝐷(𝑓𝑓1 )»

1 1
Y, 3𝑥𝑥 − 1 ≥ 0 implica 𝑥𝑥 ≥ o bien 𝑥𝑥 ∈ � , +∞� «solución a la inecuación lineal»
3 3

1
Luego 𝐷𝐷𝐷𝐷𝐷𝐷(𝑓𝑓1 ) = � , +∞� «determinación del 𝐷𝐷𝐷𝐷𝐷𝐷(𝑓𝑓1 )»
3

𝐷𝐷𝐷𝐷𝐷𝐷(𝑓𝑓2 ) = ℝ «𝑓𝑓2 (𝑥𝑥) es una función cuadrática, su dominio es el conjunto ℝ»

Y, 𝑓𝑓2 (𝑥𝑥) = 0 implica 𝑥𝑥 2 − 1 = 0 o bien 𝑥𝑥 = −1 y 𝑥𝑥 = 1 «números a quitar del denominador»

1
Así, 𝐷𝐷𝐷𝐷𝐷𝐷(𝑓𝑓) = [ , +∞) ∩ ℝ~{−1, 1} «sustituyendo lo encontrado en la identidad (∗)»
3

1
Luego, 𝐷𝐷𝐷𝐷𝐷𝐷(𝑓𝑓) = � , +∞� ~{1} «−1 queda descartado porque no está incluido en el �3 , +∞�»
1
3

3 2𝑥𝑥+3
Ejemplo 2. Hallar el dominio de la función 𝑔𝑔(𝑥𝑥) = �
𝑥𝑥 2 +𝑥𝑥−2

2𝑥𝑥 + 3
𝐷𝐷𝐷𝐷𝐷𝐷(𝑔𝑔) = 𝐷𝐷𝐷𝐷𝐷𝐷(𝑔𝑔1 ) y 𝑔𝑔1 (𝑥𝑥) = «el 𝐷𝐷𝐷𝐷𝐷𝐷 de la función raíz cúbica es igual al 𝐷𝐷𝐷𝐷𝐷𝐷 de la fun. subradical»
𝑥𝑥 2 + 𝑥𝑥 − 2

Y, 𝑔𝑔11 (𝑥𝑥) = 2𝑥𝑥 + 3 y 𝑔𝑔12 (𝑥𝑥) = 𝑥𝑥 2 + 𝑥𝑥 − 2 «identificación de las funciones que componen a 𝑔𝑔1 (𝑥𝑥)»

𝐷𝐷𝐷𝐷𝐷𝐷(𝑔𝑔1 ) = 𝐷𝐷𝐷𝐷𝐷𝐷(𝑔𝑔11 ) ∩ 𝐷𝐷𝐷𝐷𝐷𝐷(𝑔𝑔12 )~{𝑔𝑔12 (𝑥𝑥) = 0} (∗) «aplicación de la definición»

𝐷𝐷𝐷𝐷𝐷𝐷(𝑔𝑔11 ) = ℝ y 𝐷𝐷𝐷𝐷𝐷𝐷(𝑔𝑔12 ) = ℝ «𝑔𝑔11 (𝑥𝑥) es una función afín y 𝑔𝑔12 (𝑥𝑥) una función cuadrática»

Y, 𝑔𝑔12 (𝑥𝑥) = 0 implica 𝑥𝑥 2 + 𝑥𝑥 − 2 = 0 o bien 𝑥𝑥 = −2 𝑦𝑦 𝑥𝑥 = 1 «números a quitar del denominador»

Así, 𝐷𝐷𝐷𝐷𝐷𝐷(𝑔𝑔1 ) = ℝ ∩ ℝ~{−2,2} o bien 𝐷𝐷𝐷𝐷𝐷𝐷(𝑔𝑔1 ) = ℝ~{−2,2} «sustituyendo lo hallado en identidad (∗)»

Luego, 𝐷𝐷𝐷𝐷𝐷𝐷(𝑔𝑔) = ℝ~{−2,2} «por definición 𝐷𝐷𝐷𝐷𝐷𝐷(𝑔𝑔) = 𝐷𝐷𝐷𝐷𝐷𝐷(𝑔𝑔1 )»

CLEMENTE MORENO (MATERIAL DIDÁCTICO EN VALIDACIÓN 62


2𝑥𝑥+3
Ejemplo 3. Hallar el dominio de la función ℎ(𝑥𝑥) = � implica que:
𝑥𝑥 2 +𝑥𝑥−2

2𝑥𝑥+3
ℎ(𝑥𝑥) es un número real si ≥0 «planteamiento de inecuación que lleva a 𝐷𝐷𝐷𝐷𝐷𝐷(ℎ)»
𝑥𝑥 2 +𝑥𝑥−2
2𝑥𝑥+3 2𝑥𝑥+3
Y, ≥ 0 implica ≥ 0 (∗) «factorizando el denominador «factor cuadrático»»
𝑥𝑥 2 +𝑥𝑥−2 (𝑥𝑥+2)(𝑥𝑥−1)

Nota. para resolver la inecuación dada en (∗) se debe estudiar con detalle, los comentarios
dados en la página 38
(i) «ubicación de los números que anulan los factores de la inecuación
3
−2 − 1 dada en (∗) sobre la recta ℝ»
2

− − + + 2𝑥𝑥 + 3 (ii) «disposición de los factores de (∗) en la última columna»

− + + + 𝑥𝑥 + 2 (iii) «estudio del cambio de signo de los factores según lo explicado en


los comentarios de la página 38»
− − − + 𝑥𝑥 − 1
(iv) «multiplicación de los signos para llegar a los signos de la última
− + − + ℎ(𝑥𝑥) ≥ 0
fila, ver comentarios página 38»

3
Luego, 𝐷𝐷𝐷𝐷𝐷𝐷(ℎ) = �−2, − � ∪ (1, +∞) «se eligen los intervalos «+» porque ℎ(𝑥𝑥) ≥ 0»
2

Nota. Los extremos −2 y 1 son abiertos porque esos números anulan el denominador de la
3
función y la división entre «0» no está definida, − va cerrado porque anula el numerador
2

y ahí «0» está permitido

Ejercicio N° 𝟖𝟖
Realice el estudio completo de las siguientes funciones
2 2𝑥𝑥 𝑥𝑥 + 2
1) 𝑓𝑓(𝑥𝑥) = ; 2) 𝑔𝑔(𝑥𝑥) = ; 3) ℎ(𝑥𝑥) = ;
𝑥𝑥 − 1 3𝑥𝑥 − 2 𝑥𝑥 − 3

1 + 𝑥𝑥 2𝑥𝑥+1 2𝑥𝑥 + 3
4) 𝑙𝑙(𝑥𝑥) = ; 5) 𝑚𝑚(𝑥𝑥) = ; 6) 𝑛𝑛(𝑥𝑥) = ;
2 − 𝑥𝑥 𝑥𝑥−2 3𝑥𝑥 − 4

3 2𝑥𝑥 𝑥𝑥−1
7) 𝑓𝑓(𝑥𝑥) = ; 8) 𝑔𝑔(𝑥𝑥) = ; 9) 𝑔𝑔(𝑥𝑥) = ;
𝑥𝑥 2 − 1 𝑥𝑥 2 − 4 𝑥𝑥 2 −4

2𝑥𝑥 + 1 𝑥𝑥 2 2𝑥𝑥 2
10) 𝑙𝑙(𝑥𝑥) = 11) 𝑚𝑚(𝑥𝑥) = ; 12) 𝑛𝑛(𝑥𝑥) = ;
𝑥𝑥 2 − 3𝑥𝑥 + 2 𝑥𝑥 2 − 1 𝑥𝑥 2 − 4

CLEMENTE MORENO (MATERIAL DIDÁCTICO EN VALIDACIÓN 63


Encuentre el dominio de cada una de las siguientes funciones
𝑥𝑥 2 + 3 √𝑥𝑥 + 3 3
√2 − 𝑥𝑥 2
13) 𝑓𝑓(𝑥𝑥) = ; 14) 𝑔𝑔(𝑥𝑥) = ; 15) ℎ(𝑥𝑥) = ;
𝑥𝑥 − 1 𝑥𝑥 2 − 3 𝑥𝑥 2 + 3

𝑥𝑥 + 3 √3𝑥𝑥 + 5
16) 𝑙𝑙(𝑥𝑥) =
𝑥𝑥 − 1
√3 − 𝑥𝑥 2 ; 17) 𝑓𝑓(𝑥𝑥) = ; 18) 𝑔𝑔(𝑥𝑥) = �
3𝑥𝑥 + 5
;
√2 − 3𝑥𝑥 2 − 3𝑥𝑥

𝑥𝑥 2 − 3𝑥𝑥 + 2 3
√3𝑥𝑥 − 1
19) ℎ(𝑥𝑥) = ; 20) 𝑙𝑙(𝑥𝑥) = �
3 𝑥𝑥 2 + 1
; 21) 𝑔𝑔(𝑥𝑥) = ;
√𝑥𝑥 2 + 3𝑥𝑥 + 2 𝑥𝑥 3 − 4𝑥𝑥
𝑥𝑥 2 −1

5 1 4
22) 𝑓𝑓(𝑥𝑥) = ; 23) 𝑙𝑙(𝑥𝑥) = (4 − 𝑥𝑥 2 )−2 24) 𝑓𝑓(𝑥𝑥) = √4 − 9𝑥𝑥 2 ;
√9 − 𝑥𝑥 2

CLEMENTE MORENO (MATERIAL DIDÁCTICO EN VALIDACIÓN 64


República Bolivariana de Venezuela
Ministerio del Poder Popular para la Educación
Unidad Educativa: Colegio San Agustín – El Marqués
Material elaborado por el Prof. Clemente Moreno

Función Exponencial
Definición. Si «a» es un número real positivo con a ≠ 1, se llama función exponencial a la
función Expa (x) = ax , donde la constante «a» es base y el exponente «x» es la variable.

La definición indica que las imágenes de la función exponencial son números reales si a > 0,
1
pues si a ∈ ℝ− , por ejemplo a = −2, se tiene a2 = √−2 ∉ ℝ.
Siendo a ≠ 1 y a > 0 entonces a ∈ (0, 1) o bien a ∈ (1, +∞), genera dos posibilidades:
Expa (x) = ax con 0 < a < 1 (1) ; y, Expa (x) = ax con a > 1 (2)

Las tablas exhiben el comportamiento de Expa (x) para algunos valores de a ∈ (0, 1)
x 1 x x 1 x x 2 x
� � � � � � 1 x
4 2 3 ( 12 ) x ( 4 ) Y
2 x 6
−3 64 −3 8 −3 3,3 ()
3
5
−2 16 −2 4 −2 2,25
4
−1 4 −1 2 −1 1,5
3
0 1 0 1 0 1
2
1 0,5 1 0,7 1 0,81
2 2 2 1

1 0,25 1 0,5 1 0,66


-3 -2 -1 1 2 3 X
2 0,06 2 0,25 2 0,44
3 0,01 3 0,12 3 0,29

COMENTARIOS

1. Si el exponente de «a» es «x», entonces Dom[Expa (x)] = ℝ.


2. Las curvas bajan de izquierda a derecha, indicando que la función decrece para las bases
señaladas, este decrecimiento es más rápido cuando los valores de las bases se
aproximan a cero.

CLEMENTE MORENO (MATERIAL DIDÁCTICO EN VALIDACIÓN) 1


3. En cada caso la gráfica de Expa (x) pasa por los puntos (0, 1) y (1, a), indicando que
a0 = 1 y a1 = a.
4. El eje X es una asíntota horizontal para cada una de las gráficas de la función. Por ello, si
el exponente de «a» es «x», el Rgo[Expa (x)] = (0, +∞).
5. Si a ∈ (0,1), la función Expa (x) se comporta como en los casos anteriores.

En efecto
a. Como a0 = 1, la gráfica de Expa (x) pasa por el punto (0, 1)
b. Como a1 = a, la gráfica de Expa (x) pasa por el punto (1, a)
c. Al ser ax > 0 para cada a ∈ ℝ, la gráfica de Expa (x) se sitúa en el semiplano superior
respecto al eje de las abscisas, luego el eje X es una asíntota horizontal de Expa (x)

d. Si «x» es el exponente de «a», entonces Dom[Expa (x)] = ℝ y Rgo[Expa (x)] = ℝ∗+ , en estas
condiciones Expa : ℝ → ℝ∗+ ⁄Expa (x) = ax con a ∈ (0, 1) es una función sobreyectiva.

e. Expa : ℝ → ℝ∗+ ⁄Expa (x) = ax con a ∈ (0, 1) es una función inyectiva, debido a que:
ax1
Expa (x1 ) = Expa (x2 ) implica ax1 = ax2 entonces = 1 o también ax1 −x2 = a0 , luego x1 −
ax2

x2 = 0, esto es x1 = x2 . Por tanto, Expa (x) = ax es una función inyectiva.

De los incisos (d) y (e) se sigue que Expa (x) = ax con a ∈ (0, 1) es una función biyectiva.

f. Expa : ℝ → ℝ∗+ ⁄Expa (x) = ax con a ∈ (0, 1) es Y


estrictamente decreciente. En efecto
x1 < x2 implica ax1 > ax2 = Expa (x1 ) > Expa (x2 ) ∞ Expa ( x) = a x

Expa ( x) = a x 1 0
La figura ilustra el caso general de la gráfica de la X
-∞ x x ∞
función Expa (x) = a , cuando 0 < a < 1
x

CLEMENTE MORENO (MATERIAL DIDÁCTICO EN VALIDACIÓN) 2


Ahora veamos el comportamiento de Expa (x) = ax para valores de a ∈ (1, ∞)

x (1,5)x x 2x x 4x Y
4x 2x
−3 0,3 −3 0,12 −3 0,01 6
(1.5) x
−2 0,4 −2 0,25 −2 0,06 5

−1 0,6 −1 0,5 −1 0,25 4

0 1 0 1 0 1 3

1 1,2 1 1,4 1 2 2
2 2 2
1
1 1,5 1 2 1 4
2 2,3 2 4 2 16 -3 -2 -1 1 2 3 X

3 3,4 3 8 3 64

COMENTARIOS

1. Si el exponente de «a» es «x», entonces Dom[Expa (x)] = ℝ.


2. Las curvas suben de izquierda a derecha, indicando que la función crece para las bases
señaladas, crecimiento que se acentúa en la medida que la base «a» crece en el
intervalo (1, +∞).
3. En cada caso la gráfica de Expa (x) pasa por los puntos (0, 1) y (1, a), lo cual significa
que a0 = 1 y a1 = a.
4. El eje X es una asíntota horizontal para cada una de las gráficas de la función. Por ello,
si el exponente de «a» es «x», el Rgo[Expa (x)] = (0, +∞).
5. Si a ∈ (1, +∞) la función Expa (x) se comporta como en los casos anteriores.
En efecto:
a. Como a0 = 1, la gráfica de Expa (x) pasa por el punto (0, 1).
b. Como a1 = a, la gráfica de Expa (x) pasa por el punto.
c. Al ser ax > 0 para cada a ∈ ℝ, la gráfica de Expa (x) está situada en el semiplano superior
respecto al eje de las abscisas, así el eje X es asíntota horizontal de la gráfica de Expa (x).
d. Si «x» es el exponente de «a», entonces Dom[Expa (x)] = ℝ y Rgo[Expa (x)] = ℝ∗+ , en estas
condiciones

CLEMENTE MORENO (MATERIAL DIDÁCTICO EN VALIDACIÓN) 3


Expa : ℝ → ℝ∗+ ⁄Expa (x) = ax con a ∈ (1, +∞) es una función sobreyectiva.
e. Expa : ℝ → ℝ∗+ ⁄Expa (x) = ax con a ∈ (1, +∞) es una función inyectiva, debido a que:
a x1
Expa (x1 ) = Expa (x2 ) implica ax1 = ax2 entonces = 1 o bien ax1−x2 = a0 , o también x1 −
a x2

x2 = 0, esto es x1 = x2 . Por tanto, Expa (x) = ax es inyectiva.


De los incisos (d) y (e) se sigue que Expa (x) = ax con a ∈ (1, +∞) es una función biyectiva.
f. Expa : ℝ → ℝ∗+ ⁄Expa (x) = ax con a ∈ (1, +∞) es Y
estrictamente creciente. En efecto:
x1 < x2 implica ax1 < ax2 = Expa (x1 ) < Expa (x2 ) Expa ( x) = a x ∞
La figura a la derecha, ilustra el caso general de la
0 Expa ( x) = a x
1
gráfica de Expa (x) = ax , cuando a > 1
-∞ x X
x ∞

Función Logarítmica
Si a ∈ ℝ∗+ ~{1} entonces la función Expa : ℝ → ℝ∗+ ⁄Expa (x) = ax es una función biyectiva.
Por ello, dado cualquier número real b > 0, existe un único número real α ∈ ℝ tal que
Exp(α) = b y Expa (α) = ax , luego todo número real positivo se puede escribir como una
potencia de «a», es decir ax = b, «con b > 0». Esta ecuación tiene a x = α por solución única.
En particular la ecuación 3x = 9 tiene por solución x = 2, mientras que 10x = 0,01 tiene por
solución x = −2. En estos casos la solución es trivial, pero la ecuación 2x = 5 carece de
solución evidente. En su solución, interviene la función inversa a la función exponencial, que
es la función logarítmica.

Definición. La función exponencial de base «a», Expa : ℝ → ℝ∗+ es biyectiva, luego admite
función inversa. A ésta, se llama función logarítmica de base «a» y se denota por lg a . Esto es
lg a : ℝ∗+ → ℝ⁄lg a (x) = y

A la imagen de un número real x > 0, mediante la función lg a , denotada por lg a (x) se


llama logaritmo con base «a» de «x».
De acuerdo a la definición de función inversa lg a (x) = y implica Expa (y) = x, luego
lg a x = y si y sólo sí ay = x (1)

CLEMENTE MORENO (MATERIAL DIDÁCTICO EN VALIDACIÓN) 4


Ejemplo 1. lg10 100 = 2 , debido a que 102 = 100.
1
3 1 3
Del mismo modo, lg 2 √2 = debido a que 23 = √2
3

La definición ilustrada en el ejemplo, indica que las identidades logarítmicas se pueden


escribir de manera exponencial. De igual modo, las identidades exponenciales se pueden
1 3 1 1
escribir como igualdades logarítmicas, así � � = implica log 1 � � = 3
2 8 2 8

Ejemplo 2. Los casos del (1) al (8) ilustran la relación existente entre la función exponencial
y la función logarítmica

1) log 4 4096 = 6 ya que 46 = 4096 1 −6


2) log 1 64 = −6 debido a que � � = 64
2 2

3) log10 0,0001 = −4 pues 10−4 = 0,0001 4) log 3 �


1
� = −5 debido a que 3−5 =
1
243 243

5) 57 = 78125 implica log 5 78125 = 7 3 4 81 81


6) � � = implica log 3 � �=4
4 256 4 256

1
7) 243�5� = 3 implica log 243 3 =
1 8) 10−3 = 0,001 implica log10 0,001 = −3
5

COMENTARIO

aloga b = b (2)
Dado que an = b implica log a b = n entonces � conforman las identidades
log a an = n (3)
exponencial y logarítmica, respectivamente

Ejemplo 3. Calcular los siguientes logaritmos

1) Calcular log 2 256,


Como 256 = 28 entonces log 2 256 = log 2 28 «tomando log 2 en ambos lados de la igualdad»

Luego log 2 256 = 8 «aplicando (3) en segundo miembro de la igualdad»


1
2) Hallar log 3 �243�,
1 1 1
Como �243� = 35 = 3−5 entonces log 3 �243� = log 3 3−3 «tomando log 3 en ambos lados de la igualdad»

CLEMENTE MORENO (MATERIAL DIDÁCTICO EN VALIDACIÓN) 5


1
Luego log3 � � = −3 «aplicando (3) en segundo miembro de la igualdad»
243

Al aplicar la definición (1) en los casos anteriores, resulta:

1) Sea log 2 256 = x entonces 2x = 256 «aplicación de la definición (1)»

Pero 2x = 28 implica log 2 2x = log 2 27 «tomando log 2 en ambos lados de la igualdad»

Entonces x = 7 «aplicando (3) en ambos miembro de la igualdad»

En consecuencia, log 2 256 = 7


1 1
2) Sea log 3 �243� = x entonces 3x = 243 «aplicación de la definición (1)»

Pero 3x = 3−5 implica log 3 3x = log 3 3−5 «tomando log 3 en ambos lados de la igualdad»

Entonces x = −5. «aplicando (3) en ambos miembro de la igualdad»


1
Por tanto log 3 � � = −5
243

COMENTARIO

En la práctica no es preciso especificar los pasos que conducen a la determinación


del logaritmo solicitado, actuar como se indica en los dos ejemplos siguientes es suficiente

Ejemplo 4. ¿Cuál es el log 2 8?


Sea log 2 8 = x entonces 2x = 8 o su equivalente 2x = 23 , luego x = 3. Así log 2 8 = 3

1
Ejemplo 5. ¿Cuál es el log 3√3 � �?
243
x
1 5
1 x
1 x 1
Sea log 3√3 �243� = x entonces �33 � = �3� o bien 33 = 3−5 , luego = −5. Así log 3√3 �243� = −15
3

Ejemplo 6. ¿Cuál es el valor de «x» en la identidad log x 128 = 7?


1 1
Como log x 128 = 7 implica x 7 = 128 o bien x 7 = 27 entonces (x 7 )7 = (27 )7 , luego x = 2

CLEMENTE MORENO (MATERIAL DIDÁCTICO EN VALIDACIÓN) 6


Ejercicio N° 1
Según sea el caso, exprese en forma exponencial las identidades logarítmicas o en forma
logarítmica las identidades exponenciales siguientes:

1) log 5 15625 = 6; 2) log 2 4096 = 12; 3) log 3 6561 = 8;

4) log 1 256 = −8; 4) log 1 2187 = −7; 6) log 1 15625 = −6;


2 3 5

7) log 5 �
1
� = −4; 8) log 3 (3−5 ) = −5; 9) log 7 �
1
� = −4;
625 2041

32 243 32
10) log 2 � � = 4; 11) log 3 � � = 5; 12) log 2 � � = 5;
3 81 2 64 5 3125

13) 26 = 64; 1 0
1
14) � � = 1; 15) 31255 = 5;
5

7
16) √16384 = 4 4 5 1024 18) 100−3 = 0,00000;
17) � � = ;
5 3125

Calcule los siguientes logaritmos

19) log 2 128; 20) log 3 243; 21) log 4 1024;

22) log 1 2401; 23) log 1 2187; 24) log 1 14641;


7 3 11

243 256 729


25) log 3 � �; 26) log 4 � �; 27) log 3 � �;
5 3125 5 625 4 4096

5
28) log 4√8 √16; 4 125 6 81
29) log 4
� ; 30) log 3 8
� ;
� 8 � 16
25 27

Determine el valor de «x» en los casos siguientes:

31) log x 128 = 4; 33) log x √16 = 4 34) log x �


81
�=6
625

35) log 2 x = 7; 36) log 2 x = 5 37) log 3√5 x = 6


3

38) x = log 2 32 + log 3 27 − log 5 125 39) x = log 4 0,0625 + log 7 343 + log 2 � �
1
16

log8 64 + log64 8 81 log25 5−log13 1 log4 64


40) x = − log 2 41) x =
log9 3 + log3 9 3 16 log10 0,0001+log2 16

CLEMENTE MORENO (MATERIAL DIDÁCTICO EN VALIDACIÓN) 7


La calculadora en el cálculo de logaritmos y solución de ecuaciones exponenciales

COMENTARIO

Los logaritmos más utilizados son los logaritmos decimales o vulgares, llamados así
porque tienen base 10, es decir log10 a; en este case se escribe log a entendiendo que la
base es 10. También son de uso frecuente los logaritmos naturales, llamados así debido a
que tienen base «e = 2,718281827 …» y en lugar de escribir log e a se escribe ln a, en el
entendiendo que es el logaritmo natural

Las calculadoras electrónicas calculan en forma directa estos dos tipos de logaritmos,
en otro caso, el log a x se usa la calculadora y la propiedad N° 10 que se estudiar más adelante,

por ahora, una calculadora como la de la figura tiene


un par de teclas marcadas con [log] y [ln] las cuales
permiten hallar estos logaritmos, con solo presionar
la tecla [log] seguida del número si se quiere el
logaritmo decimal o la tecla [ln] y luego el número, si
se quiere el logaritmo natural
Nótese que encima de las teclas [log] y [ln] se
encuentran las funciones inversas correspondientes:
x ⟶ 10x y x ⟶ ex que se activan presionando
previamente la tecla [SHIFT] o [INV]
Así, si presiona la tecla [log] y luego tecla [5], se
obtiene el número 0, 6989700043, diciendo que:
log10 5 = 0,6989700043

Cosa que se puede verificar usando la función de potenciación [∧], para ello presione
10 luego [∧] seguido de 0, 6989700043 es decir 10∧0,6989700043 = 5. A este valor se llega si se
utiliza la función 10x , para ello presione [SHIFT] seguido de 10x y luego 0, 6989700043

CLEMENTE MORENO (MATERIAL DIDÁCTICO EN VALIDACIÓN) 8


De igual modo, si presiona [ln] seguido 7 se obtiene el número 1,945910149, diciendo
que log e 7 = ln 7 = 1,945910149
Para comprobarlo se escribe [SHIFT] seguido de ex y el número 1,945910149

Ejemplo 1. Halle los valores de «x» que satisfacen la igualdad 10𝑥𝑥 = 23

Como 10𝑥𝑥 = 23 implica log10 23 = 𝑥𝑥 «aplicando definición 1»

Entonces 𝑥𝑥 = 1,361727836 «presionando log 23 en la calculadora»

Prueba. [SHIFT]10x 1,3617278336 = 22, 9�

Ejemplo 2. Halle los valores de «x» que satisfacen la igualdad 104x = 1,045

Dado que 104x = 1,045 implica log10 1,045 = 4x «aplicando definición 1»

Entonces 4x = 0,01911629045 «presionando log 23 en la calculadora»


1
Luego x = 0,004779072613 «multiplicando ambos lados por el inverso de 4 que es »
4

Prueba. 0,004779072613 ∙ 4 = 0,01911962905 y 10∧0,0191196905 = 1,045008181

Ejemplo 3. Halle los valores de «x» que satisfacen la igualdad ex = 0,73

Como ex = 0,73 implica log 𝑒𝑒 0,73 = ln 0,73 = 𝑥𝑥 «aplicando definición 1»

Entonces x = −0,3147107448 «presionando ln 0,73 en la calculadora»

Prueba. [SHIFT] ex (−0,3147107448) = 0,73

Ejemplo 4. Halle los valores de «x» que satisfacen la igualdad e−2x+1 = 5

Dado que e−2x+1 = 5 implicca log e 5 = ln 5 = −2x + 1 «aplicando definición 1»

Entonces −2x + 1 = 1,609437912 «presionando ln 5 en la calculadora»

Luego 𝑥𝑥 = −0,3047189562 «resolviendo la ecuación»

Prueba. e−2∙(−0,3047189562) = e1,609437912 = 4, 9�

CLEMENTE MORENO (MATERIAL DIDÁCTICO EN VALIDACIÓN) 9


COMENTARIO

1. El gráfico de la función lg a (x) se obtiene a partir de la gráfica de la función exponencial


Expa (x) por simetría con respecto a la recta y = x , tal como se ilustra en la figura

Y Y
Expa x lg a x Expa x
y=x y=x
0 <a <1
a>1
lg a x
1
1
1 X
X
1

A partir de las gráficas se pueden deducir algunas de las propiedades de los logaritmos

Propiedades de la función logarítmica


1. Si la base a > 1 la función lg a x es estrictamente creciente
En efecto. En la figura de la derecha se observa que si x1 > x2 , entonces lg a x1 > lg a x2 ;
pues si supone que lg a x1 ≤ lg a x2 entonces algax1 ≤ algax2 luego x1 ≤ x2 , lo cual es una
contradicción.
Por consiguiente, si x1 > x2 entonces lg a x1 > lg a x2

2. El logaritmo de 1 es cero, es decir «log 𝑎𝑎 1 = 0»


En efecto. Sea lg a 1 = x, luego ax = 1 pero a0 = 1, entonces ax = a0 . Así log 𝑎𝑎 1 = 0

COMENTARIO

La propiedad indica que para escribir el número 0 desde el campo de los logaritmos se
debe escribir log 𝑎𝑎 1

CLEMENTE MORENO (MATERIAL DIDÁCTICO EN VALIDACIÓN) 10


3. El logaritmo de cualquier número real mayor que 1 es positivo
En efecto. Si x > 1 entonces lg a x > lg a 1 «tomando log a en ambos lados de la igualdad»

Entonces lg a x > 0 «por propiedad 2; log 𝑎𝑎 1 = 0»

4. El logaritmo de cualquier número real entre 0 y 1, es negativo


En efecto. Si 0 < x < 1 entonces lg a x < lg a 1 «tomando log a en ambos lados de la igualdad»

Entonces lg a x < 0 «por propiedad 2; log a 1 = 0»

5. El logaritmo en base «a» de «a» es 1, es decir «log a a = 1»


En efecto. lg a a = x implica ax = a1 , luego x = 1. En consecuencia, log a a = 1

COMENTARIO

La propiedad indica que para escribir el número 1 desde el campo de los logaritmos se
debe escribir log a a

6. El logaritmo en base «𝑎𝑎» del producto de dos números es igual a la suma de los logaritmos
de los factores, es decir lg a (x1 . x2 ) = lg a x1 + lg a x2
En efecto. Sea y = lg a (x1 . x2 ); y1 = lg a x1 y y2 = lg a x2 «asignación de valor a los logaritmos»
Luego ay = x1 . x2 (a); ay1 = x1 (b); y ay2 = x2 (c) «aplicando definición (1) en los tres casos»
Entonces ay = ay1 . ay2 «sustituyendo en (a) los valores de x1 y x2 dados en (b) y (c)»

Por ello ay = ay1 +y2 «multiplicación de potencias de igual base en el segundo miembro»

Así y = y1 + y2 . «aplicando identidad 3 en ambos lados de la igualdad»


En consecuencia, lg a (x1 . x2 ) = lg a x1 + lg a x2

7. El logaritmo en base «a» del cociente de dos números es igual a la diferencia entre el
x
logaritmo del dividendo y el logaritmo del divisor, Esto es: lg a � 1� = lg a x1 − lg a x2
x2

CLEMENTE MORENO (MATERIAL DIDÁCTICO EN VALIDACIÓN) 11


x
En efecto. Sea y = log a � 1�; y1 = lg a x1 y y2 = lg a x2 «asignación de valor a los logaritmos»
x2
x1
Luego ay = (a); ay1 = x1 (b); y ay2 = x2 (c) «aplicando definición (1) en los tres casos»
x2

a y1
Entonces ay = «sustituyendo en (a) los valores de x1 y x2 dados en (b) y (c)»
a y2

Por ello ay = ay1 − y2 «división de potencias de igual base en el segundo miembro»

Así y = y1 − y2 «aplicando identidad 3 en ambos lados de la igualdad»


x
En consecuencia, lg a � 1 � = lg a x1 − lg a x2
x2

8. El logaritmo en base «a» de una potencia es igual al producto del exponente por el logaritmo
de la base, es decir lg a x n = nlg a x
En efecto. Sea y = lg a x n y y1 = lg a x «asignación de valor a los logaritmos»

Luego ay = x n (a); y ay1 = x (b); «aplicando definición (1) en los dos casos»

Entonces ay = (ay1 )n «sustituyendo en (a) el valor de x dados en (b)»

Por ello ay = any1 «producto de potencia de en una potencia en el segundo miembro»

Así y = ny1 «aplicando identidad 3 en ambos lados de la igualdad»

En consecuencia, lg a x n = nlg a x

9. El logaritmo en base «a» de una raíz ene-sima es igual al logaritmo de la cantidad subradical
n lga x
dividido entre el índice de la raíz, es decir lg a √x =
n
𝑛𝑛
En efecto. Sea 𝑦𝑦 = log 𝑎𝑎 √𝑥𝑥 y 𝑦𝑦1 = log 𝑎𝑎 𝑥𝑥 «asignación de valor a los logaritmos»
1
Luego ay = x n (a) y ay1 = x (b) «aplicando definición (1) en los dos casos»
1
Entonces ay = (ay1 )n «sustituyendo en (a) el valor de x dados en (b)»
y1
Por ello ay = a n «producto de potencia de en una potencia en el segundo miembro»
y1
Así y = «aplicando identidad 3 en ambos lados de la igualdad»
n
n lga x
En consecuencia, lg a √x =
n

CLEMENTE MORENO (MATERIAL DIDÁCTICO EN VALIDACIÓN) 12


lg10 x
10. Si a > 1 entonces lg a x = , indicando que lg a x se puede escribir en términos de
lg10 a
logaritmos decimales, cuyo valor se determina con la calculadora
En efecto. Sea y = lg a x; y1 = lg10 x y y2 = lg10 a «asignación de valor a los logaritmos»

Luego ay = x (a); 10y1 = x (b) y 10y2 = a (c) «aplicando definición (1) en los dos casos»

Entonces, (10y2 )y = 10y1 «sustituyendo (c) en el primer miembro y (b) en el segundo»

Por ello, 10y∙y2 = 10y1 «producto de potencia de en una potencia en el primer miembro»
y1
Así, y. y2 = y1 o bien y = «aplicando identidad 3 en ambos lados de la igualdad»
y2
lg10 x
En consecuencia, lg a x =
lg10 a

Nota. La propiedad también se cumple hasta el orden de las milésimas para logaritmos
ln x
naturales, luego log a x = es también una buena aproximación
ln a

COMENTARIO

Esta propiedad fue referida cuando se mostró el modo de utilizar la calculadora en el


cálculo de logaritmos, allí quedo pendiente el cálculo digamos de log 2 7, aquí se tiene:
log10 7
log 2 7 = «aplicando la propiedad 10»
log10 2
0,84509804
log 2 7 = «hallando los logaritmos en la calculadora»
0,3010299957

log 2 7 = 2,806702626 «ejecutando la división»

Nota. Si se utilizan el logaritmo natural se tiene


ln 7 1,945910149
log 2 7 = entonces log 2 7 = luego log 2 7 = 2,807354922
ln 2 0,6931471806

La propiedad también se aplica en la solución de ecuaciones del tipo 3𝑥𝑥 = 0,78


En efecto. 3𝑥𝑥 = 0,78 implica log 3 3𝑥𝑥 = log 3 0,78 «tomando log 3 en ambos lados»

Entonces x = log 3 0,78 «aplicando identidad 3»


log10 0,78
Luego x = o bien x = −0,2261592755 «aplicando la propiedad 10 y dividiendo»
log10 3

CLEMENTE MORENO (MATERIAL DIDÁCTICO EN VALIDACIÓN) 13


En los ejemplos siguientes se muestra la utilidad de los logaritmos, en el trabajo operativo
con este objeto matemático, tanto en su operatividad como en la solución de ecuaciones
exponenciales y logarítmicas
Ejemplo 1. Si x = a ∙ b ∙ c ∙ d, determinar el log a x implica

log a x = log a (a ∙ b ∙ c ∙ d)

= log a a + log a b + log a c + log a d «aplicación de propiedad 6»

= 1 + log a b + log a c + log a d «aplicación de propiedad 5»

Luego log x x = 1 + log a b + log a c + log a d

a5 ∙ b3
Ejemplo 2. Siendo x = , hallar el log a x implica
c4

a5 ∙b3
log a x = log a � �
c4

= log a (a5 . b3 ) − log a c 4 «aplicación de propiedad 7»

= log a a5 + log a b3 − log a c 4 «aplicación de propiedad 6»

= 5 ∙ log a a + 3 ∙ log a b − 4 ∙ log a c «aplicación de propiedad 8»

= 5 ∙ 1 + 3 log a b − 4 log a c «aplicación de propiedad 5»

Luego log a x = 5 + 3 log a b − 4 log a c

(125,47).(0,0324).(130087)
Ejemplo 3. Calcular el valor de A = (22,45).(0,00879).(937820)

(125,47)∙(0,0324)∙(130.087)
log A = log «tomando log decimal en ambos lados de la igualdad»
(22,45)∙(0,00879)∙(937.829)

= lg(125,47) + lg(0,0324) + lg(130087) − lg(22,45) − lg(0,00879) − lg(937820)


= 2,098539 + (−1,4894549) + 5,11423389 − 1,35121635 − (−2,05601113) − 5,9721195

lgA = 0,45599409 «ejecutando la suma algebraica»

10∧(0,45599409) = A «aplicando la definición 1»

En consecuencia, A ≈ 2,857551657

Este ejemplo muestra la utilidad práctica de los logaritmos, ellos fueron y son utilizados en
el cálculo de operaciones combinadas de productos y cocientes con muchos factores

CLEMENTE MORENO (MATERIAL DIDÁCTICO EN VALIDACIÓN) 14


3 4
√a2 b ∙ √a3 b2
Ejemplo 4. Si x = 5 calcular log 𝑎𝑎 𝑥𝑥
√a3 b2

3 4
√a2 b∙ √a3 b2
log a x = log a � 5 � «tomando log a en ambos lados de la igualdad»
√a3 b2

3 4 5
= log a � √a2 b ∙ √a3 b 2 � − log a √a3 b 2 «aplicando propiedad 7»
3 4 5
= log a √a2 b + log a √a3 b 2 − log a √a3 b 2 «aplicando propiedad 6»

loga (a2 b) loga �a3 b2 � loga �a3 b2 �


= + − «aplicando propiedad 9»
3 4 5
2 loga a+loga b 3 loga a+2 logb b 3 loga a+2 loga b
= + − «aplicando propiedades 6 y 8»
3 4 5
(2+loga b) (3+2 loga b) (3+2 loga b)
= + − «aplicando propiedad 5»
3 4 5
20(2+loga 6)+15(3+2 loga b)−12(3+2 loga b)
= «tomando denominador común»
60
40+20 loga b+45+30 loga b−36−24 loga b
= «aplicando propiedad distributiva»
60
49 + 26 loga b
= «sumando términos semejantes»
60

3 4
49 + 26 loga b √a2 b ∙ √a3 b2
En consecuencia, log a x = donde x = 5
60 √a3 b2

COMENTARIO

Al mismo resultado se llega, si previamente se escribe una raíz equivalente a la operación


3 4
√a2 b ∙ √a3 b2
combinada entre los radicales que conforman la expresión x = 5
√a3 b2
3 4 3∙20 4∙15
√a2 b ∙ √a3 b2 �(a2 b)20 ∙ �(a3 b2 )15
En efecto. 5 = 5∙12 «ampliando el índice a 60, «m. c. i. (3,4,5) = 60»
√a3 b2 �(a3 b2 )12
𝟔𝟔𝟔𝟔 𝟔𝟔𝟔𝟔
�𝐚𝐚𝟒𝟒𝟒𝟒 𝐛𝐛 𝟐𝟐𝟐𝟐 ∙ �𝐚𝐚𝟒𝟒𝟒𝟒 𝐛𝐛 𝟑𝟑𝟑𝟑
= 𝟔𝟔𝟔𝟔 «operando las potencias»
�𝐚𝐚𝟑𝟑𝟑𝟑 𝐛𝐛 𝟐𝟐𝟐𝟐

60 a85 b50
= � «multiplicación y división de radicales con igual índice»
a36 b24
3 4
√a2 b ∙ √a3 b2 60
En consecuencia x = 5 , implica x = √a49 b 26
√a3 b2
60
Luego log a x = log a √a49 b 26 «tomando log a en ambos lados de la igualdad»

CLEMENTE MORENO (MATERIAL DIDÁCTICO EN VALIDACIÓN) 15


log𝑎𝑎 �𝑎𝑎49 𝑏𝑏26 �
= «aplicando propiedad 9»
60
49 log𝑎𝑎 𝑎𝑎+26 log𝑎𝑎 𝑏𝑏
= «aplicando propiedades 6 y 8»
60
(49+26 loga b)
Por ello log a x =
60

(3,505)2 (2,812)3
Ejemplo 5. Siendo x = 3 calcule su valor utilizando logaritmos decimales
√9,783

(3,505)2 ∙(2,812)3
log10 x = log10 3 «tomando log10 en ambos lados de la igualdad»
√9,783

1
= 2 log10 3,505 + 3 log10 2,812 − log10 9,783 «aplicando propiedades 6, 7, 8 y 9»
3
1
= 2 ∙ (0,544688) + 3 ∙ (0,449015) − (0,990472) «calculando log10 en la calculadora»
3

= 1,089376 + 1,347045 − 0,330157 «operando»

Así log10 x = 2,106264 mientras que x = 102,106264 o bien x = 127,721497

Ejercicio N° 2
En cada uno de los casos siguientes, determine el logaritmo de «x» en la base que se indica
en el paréntesis

1) x = abc (2) 2) x = mnpq (m)

3) x = y2z3w5 (z) 4) x = 243a4 b3 (3)


32b3 c2 (a) 2−4 a−3 b−2 c (b)
5) x= 6) x=
81a3 3−3 a2 b

1
(m) 1 5 6 (b)
36 b2 c4 2 73 a2 b6 c
7) x=� 6� 8) x=� �
22 n2 m 2
52 n3 m2

4
35 ∙ √32 a3 (b) 5 x3 y2 z4 (m)
9) x= 10) x = �� �
a2 b2 n2 m3

5
(7) √a3 b2 c4 (a)
7 2
5m3 n2 3 12) x = 3
11) x = �� � √a2 bc2
7p4 q3

4 3
n2 �n3 m2 p3 ∙ �nm2 p (p) 5 3 (2)
13) x = 36
23 �23 a2 √2ab2
m �n3 m4 p5 14) x = 3
�22 a3 5√24 a2 b

CLEMENTE MORENO (MATERIAL DIDÁCTICO EN VALIDACIÓN) 16


�8 3�16 √2 (2) 27�35 √243a (9)
16) x =
15) x = 5
4
√2187a
�64 √2

60 ∙ 502 √1000 (5) (2)


17) x = 3 20 �3 √2
√30 18) 𝑥𝑥 = � 3
�6 √10

En cada caso determine el valor de «x»

19) log a x = 3 log a b + log a 7 − 5; 1 1


20) log b x = log b p − log b q + a
3 2

1
21) log a x = 2√10 + log a 5 − 3 log a b − log a 11; 22) log a x = (2 log a n − 3 log a m) + 2 log a p − 3;
3

1 1 1
23) log a x = (2 log a p + 3 log a q − 4 log a k); 24) log 2 𝑥𝑥 = (3 log 2 𝑎𝑎 + 1) − (1 + log 2 3);
5 3 2

Determine el valor «x» utilizando en primer lugar, logaritmos decimales y en segundo lugar,
logaritmos naturales
215,4 ∙25,314 (13,052)3 ∙ (101,121)4
25) x = ; 26) x = ;
231,17 ∙ 12,157 (15,003)4 ∙ (0,032)5

2 3,022 ∙ √27,44
17,233 ∙ 12,0134
27) x = � 28) x = ;
3� ;
3
13,045 ∙ 11,021 √0,00051

3 27,552 3
29) x = � 3
30) x = �12 �21 √7
3
11,114

Halle el valor de «x» para el cual se hacen ciertas las igualdades siguientes:

31) 10x = 12; 32) ex = 0,64;

33) e−2x+1 = 5; 34) 10x


2 −1
= 3;
1
35) ex+1 = 3; 36) lg 2 32 = x;

2
37) 5lgx 3 = 3; 38) lg x 4 = ;
5

39) lg 2 (3x − 1) = −3; 40) lg 3 (2x + 1) = 2;

41) 32x+3 = 74x+3 ; 42) lg(𝑥𝑥 2 − 144) − lg(𝑥𝑥 + 12) = 1;

CLEMENTE MORENO (MATERIAL DIDÁCTICO EN VALIDACIÓN) 17


República Bolivariana de Venezuela
Ministerio del Poder Popular para la Educación
Unidad Educativa: Colegio San Agustín – El Marqués
Profesor: Clemente Moreno P

Razones trigonométricas

Consideraciones previas − Medida de ángulos


Al igual que como se miden longitudes, para medir ángulos se elige un ángulo unidad y
los demás se miden viendo cuantas veces cabe el ángulo unidad en aquél que se quiere Medir.

Para precisar esta medida consideremos un sistema de coordenadas


𝑦𝑦
en el plano y una circunferencia de radio 1 con centro en el origen y B

sobre esta un punto B que se mueve desde el punto A, en este caso, 𝜃𝜃 𝑥𝑥


0 A(1, 0)
en sentido contrario a las agujas del reloj, como muestra la figura. En
� corresponde la misma
ella, tanto al ángulo θ = AOB como el arco AB

amplitud, lo cual habla de dos sistemas para medir ángulos, el sistema circular y el sistema
sexagesimal

Sistema circular
Este sistema toma al radián como unidad de medida y corresponde al arco cuya longitud

es igual a la longitud del radio de la circunferencia y dado que la π


𝑦𝑦
2
longitud de la circunferencia es 2πr y en este caso r = 1 se tiene que
la medida del ángulo completo «una vuelta» es 2π radianes. Aquí el 𝜋𝜋 0
𝑥𝑥
𝜋𝜋 2𝜋𝜋
ángulo llano es π radianes, el ángulo recto radianes y el ángulo que
2
3π 3π
corresponde a 3 rectos es radianes, tal como se ilustra en la figura 2
2

Al representar ángulos expresados en radianes, es recomendable distinguir el factor


fraccionario que acompaña a π, si éste corresponde a una fracción propia, el denominador de
la fracción corresponde al número de partes en que se ha de dividir los arcos que corresponden
a los dos primeros cuadrantes y el numerador la cantidad de partes que ha tomado la fracción

CLEMENTE MORENO (MATERIAL DIDÁCTICO EN VALIDACIÓN) 1


3 7 11
Ejemplo. La representación de los ángulos π; π y π es como se ilustra en las figuras
8 12 16

π 𝑦𝑦 π
𝑦𝑦 3 7 𝑦𝑦 π
2 π π 11
8 12 2 π 2
16

𝜋𝜋 0 𝑥𝑥 𝜋𝜋 0 𝜋𝜋 0
𝑥𝑥 𝑥𝑥
2𝜋𝜋 2𝜋𝜋 2𝜋𝜋

3π 3π 3π
2 2 2

Si el factor fraccionario que acompaña a π corresponde a una fracción impropia, esta se


transforma en número mixto y el sector que va desde π a 2π se divide en la cantidad de partes

que indica el denominador y se toman las que indica el 𝑦𝑦


π
2
numerador de la fracción que conforma el número mixto.
5 5 2
Ejemplo. Para el ángulo π se tiene π = π + π, entonces 𝜋𝜋 0
𝑥𝑥
3 3 3 2𝜋𝜋
el sector de π a 2π se divide en 3 partes y la 2da corresponde a
5
𝜋𝜋
5 3π 3
π, tal como se ilustra en la figura 2
3

Sistema sexagesimal
En este sistema la unidad de medida de un arco es el grado sexagesimal, el cual resulta
de dividir un ángulo completo en 360 partes iguales. Para indicar la medida en este sistema se
utiliza el símbolo «°», así 23 grados sexagesimales se indican por 23°, se mide con transportador

El transportador en la medida de ángulos


(a) «el transportador es un instrumento para medir ángulos en forma
de círculo o de semicírculo que cuenta con una escala métrica
graduada para medir grados. El más común de los transportadores
es el de tipo semicircular con su borde esférico graduado en una
escala que va desde el ángulo nulo «0° » hasta el ángulo llano «180° »,
dispuesta en forma horaria y antihoraria a fin de que pueda utilizarse
del modo en que se requiera; además, en el centro de la regla
horizontal hay dos segmentos de rectas perpendiculares, si imagina
la prolongación del segmento vertical, éste coincidirá con «90° », y la
prolongación del segmento horizontal coincidirá con «0° » y «180° »,
lo que permite la medición»

CLEMENTE MORENO (MATERIAL DIDÁCTICO EN VALIDACIÓN) 2


(b) «para medir un ángulo cualquiera con el transportador, basta hacer
coincidir el punto de su vértice «en caso A» con el punto de
intersección de los segmentos perpendiculares en el transportador y
un lado del ángulo, de preferencia el inicial «aquí el lado AC» con el
segmento de recta horizontal del transportador «0° », tal como se
indica en la figura; de este modo, el lado final del ángulo «aquí el lado
AB» coincide con un número del borde esférico del transportador que
indica la medida del ángulo. En el caso indicado en la figura, la
medida del ángulo de «39° » y se escribe m ≮ CAB = 39∘ »

Cuando la medida del ángulo incluye decimales, digamos 27,39° significa que se toman
27 ángulos de 1 grado más 39 veces la centésima parte del ángulo unidad. También es común
utilizar subunidades no decimales, como el minuto, que es la 60ava parte de un grado, así
«1∘ = 60′ un grado es igual a 60 minutos» y el segundo, que es la 60ava parte de un minuto,
«1′ = 60′′ un minuto es igual a 60 segundos». En el trabajo con ángulos, a menudo se requiere
pasar del sistema sexagesimal al sistema decimal y viceversa
22 18
Ejemplo 1. El ángulo 42°22′18′′ equivale a 42 + + = 42,371666 grados;
60 3600

Ejemplo 2. Para el ángulo 98,73° se tiene 0,73 ∙ 60 = 43,8 minutos y 0,8 ∙ 60 = 48 segundos,
luego 98,73° = 98°43′48′′

Conversión de medidas
Dado que 2π = 360° o bien π = 180°, en la conversión de un sistema a otro, se tiene:
π
(i) Para llevar de grados a radianes, basta multiplicar el valor del ángulo dado por
180°

Ejemplo. Al expresar en radianes el ángulo de 105° se tiene

α = 105° = 105° ∙
π
luego α =
7
π radianes «se renombra el ángulo y multiplica por
π
180°
, luego se simplifica»
180° 12

180°
(ii) Para llevar de radianes a grados, basta multiplicar el valor del ángulo dado por
π

12
Ejemplo. Al expresar en grados el ángulo π radianes, se tiene
27

12 12 180° «se renombra el ángulo y se multiplica por


180°
, luego se simplifica»
α= π radian= π∙ luego 𝛼𝛼 = 80° π
27 27 π

CLEMENTE MORENO (MATERIAL DIDÁCTICO EN VALIDACIÓN) 3


Operaciones con ángulos
En la adición y sustracción de ángulos se ha de tener en cuenta que cuando:
(i) La suma de los minutos o segundos excede a 60, se toman 60 minutos y se transforman en
1 grado, de igual modo 60 segundos se transforman en 1 minuto
(ii) Al restar un ángulo de otro, hacen falta minutos o segundos en el minuendo, se toma 1 grado
de éste se transforma en 60 minutos y se añade a los minutos existentes, del igual modo se
procede en los segundos, se toma 1 minuto del minuendo se transforma en 60 segundos y
se añade a los segundos existentes
Ejemplo 1. Dados α = 45°37′36′′ y β = 52°42′36′′ entonces α + β se corresponde con:
(i) «al efectuar la suma se tiene 81′′, luego se debe pasar 1 minuto al lugar de los minutos y
α = 45° 37′ 45′′ los 60′′ que le corresponden se deben restar de los 81′′ iniciales»
β = 52° 42′ 36’’
α + β = 97° 79′ 81′′

(ii) «la transformación (i) se opera con el resultado de la suma inicial y se obtienen 80′»
α + β = 97° 79′ 81′′
1′ − 60′′ (iii) «de los 80′ se pasa 1 grado al lugar de los grados y los 60′ que le corresponden se deben
restar de los 80′ que están en lugar de los minutos»
α + β = 97° 80’ 21’’
(iv) «la transformación (ii) se opera con el resultado de la segunda suma y se obtiene el
α + β = 97° 80’ 21’’ resultado final»
1° − 60′
α + β = 98° 20′ 21′′

Ejemplo 2. Dados α = 85°15′27′′ y β = 37°16′28′′ entonces α − β se corresponde con:

𝛼𝛼 = 85°15′27′′ implica 𝛼𝛼 = 84° 75′ 27′′ (i) «como 15′ < 16′ se pasa 1° = 60′ al lugar de los minutos y se suma a
los minutos allí existentes»

𝛼𝛼 = 84° 75′ 27′′ implica 𝛼𝛼 = 84° 74′ 87′′ (ii) «como 27′ < 28′ se pasa 1′ = 60′′ al lugar de los segundos y se suma
a los segundos allí existentes»
(iii) «se realiza la resta debido a que el ángulo del minuendo expresado en
α = 84° 74′ 87′′ grados, minutos y segundos es mayor el ángulo del sustraendo que
β = 37° 16′ 28′′ también está dado en grados, minutos y segundos»

α − β = 47° 58′ 59′′

Para la multiplicación de un ángulo por un escalar «un número», una vez efectuado el
producto, el ángulo resultante se transforma como se indicó en la suma
Ejemplo 3. Dado α = 25° 34′ 17′′ entonces 3α se corresponde con
(i) «el escalar 3 se multiplica por los segundos, minutos y grados que componen el ángulo»
α = 25° 34′ 17′′
x3
3α = 75° 102′ 51′′

CLEMENTE MORENO (MATERIAL DIDÁCTICO EN VALIDACIÓN) 4


(ii) «de los 102′ se pasa 1 grado al lugar de los grados y los 60′ que le corresponden se
3α = 75° 102′ 51′′ restan de los 102′ que están en lugar de los minutos»
1° − 60′
3α = 76° 42′ 51′′

Ejemplo 4. Dado el ángulo α = 17° 23′ 15′′ entonces α ÷ 3 se corresponde con:


(i) «el resto 2° se transforma en minutos y se traslada a la posición de los minutos,
120′ 120′’ luego se realiza la suma y se procede a dividir»
α = 17° 23′ 15′′ 3°
2° 143′ 135′′ 5° 47′ 45′′ (ii) «el resto 2′ se transforma en segundos y se traslada a la posición de los
23′ segundos, luego se realiza la suma y se procede a dividir»
15′′
2′ 0′′

Ejercicio N° 1
1. Convertir en grados, minutos y segundos los siguientes ángulos

a) 29,17°; b) 35,43°; c) 24,46°; d) 18,54°;

2. Convertir en decimales los ángulos dados

a) 57° 18′ 49′′; b) 23° 17′ 32′′ c) 47° 19′ 23′′; d) 19° 14′ 19′′;

3. Convertir a radianes o a grados sexagesimales según sea el caso

a) 30°; b) 150°; c) 225°; d) 285°;


π 5 7 19
e) ; f) π; g) π; h) π;
18 6 6 12

4. Dados 𝛼𝛼 = 12° 15′ 17′′; 𝛽𝛽 = 27° 19′ 37′′; 𝛾𝛾 = 17° 23′ 54′′ y 𝛿𝛿 = 16° 23′ 14′′ hallar:

a) α + β; b) β − γ; c) α + γ; d) γ + δ;

e) α − β; d) β + γ; f) α − γ; g) γ − δ;

5. Calcular el complemento de cada uno de los siguientes ángulos


7
a)
5
π b) 15° 37′ 43′′ c) π d) 32° 19′ 35′′
12 18

CLEMENTE MORENO (MATERIAL DIDÁCTICO EN VALIDACIÓN) 5


6. Dados α = 13° 34′ 45′′ y β = 29° 36′ 42′′ calcular

a) 4α; b) 3β; c) 2α + 3β; d) 3α + 4β;

7. Dados α = 15° 34′ 42′′ y β = 27° 36′ 52′′ calcular

a) α ÷ 3; b) (α + β) ÷ 4 c) β ÷ 2 d) (α − β) ÷ 3

Razones trigonométricas de un ángulo agudo


Por conocimientos previos sobre semejanza de triángulos, sabemos que dos triángulos

C′ rectángulos son semejantes si tienen igual un ángulo agudo,


C
así ≮ B =≮ B′ y ≮ A =≮ A′ = 1R entonces el ∆ABC~∆A′ B′ C ′
����
AC ����
AB
A B A′ B′ luego ������ = ������ lo cual indica que esos cocientes dependen
A′C′ A′B′

solo de la medida ≮ B del ángulo y no de las longitudes de los lados. A estos cocientes se les
llama razones trigonométricas de ≮ B

Considere el triángulo rectángulo ∆ABC de catetos ���� ���� e


AB y BC C

���� y α la medida del ángulo de vértice en A; así, se dice


hipotenusa AC b
a
que ����
BC es el cateto opuesto al ángulo y ����
AB el cateto adyacente,
𝛼𝛼
luego las razones trigonométricas del ángulo 𝛼𝛼 se definen del modo A c B

siguiente:
Seno del ángulo «𝛂𝛂». Corresponde al cociente entre la longitud del cateto opuesto y la longitud
de la hipotenusa, denotado por
����
BC a
sen α = ���� o bien sen α =
AC b

Coseno del ángulo «𝛂𝛂». Corresponde al cociente entre la longitud del cateto adyacente y la
longitud de la hipotenusa, denotado por
����
AB c
cos α = ���� o bien cos α =
AC b

Tangente del ángulo «𝛂𝛂». Corresponde al cociente entre la longitud del cateto opuesto y la
longitud del cateto adyacente, denotado por
����
BC a
tg α = ���� o bien tg α =
AB c

CLEMENTE MORENO (MATERIAL DIDÁCTICO EN VALIDACIÓN) 6


Al considerar los recíprocos de los cocientes anteriores se obtienen otras tres razones
trigonométricas, cuyas definiciones son:
Cosecante del ángulo «𝛂𝛂». Corresponde al cociente entre la longitud de la hipotenusa y la
longitud del cateto opuesto, denotado por
����
AC b
csc α = ���� o bien csc α =
BC a

Secante del ángulo «𝛂𝛂». Corresponde al cociente entre la longitud de la hipotenusa y la longitud
del cateto adyacente, denotada por
����
AC b
sec α = ����
o bien sec α =
AB c

Cotangente del ángulo «𝛂𝛂». Corresponde al cociente entre la longitud del cateto adyacente y
la longitud del cateto opuesto, denotado por
����
AB c
ctg α = ����
o bien ctg α =
BC a

COMENTARIO

Las calculadoras electrónicas permiten hallar en forma


directa las razones trigonométricas de un ángulo agudo. Así
para obtener el valor del seno, coseno y tangente, basta
introducir el valor del ángulo «dado en grados» y luego
apretar la tecla correspondiente. Para obtener el valor de la
cosecante, secante o cotangente de un ángulo, primero se
debe hallar el valor del seno, coseno y tangente «según el
caso» y luego se halla el reciproco del valor obtenido. Es
importante observar que las segundas opciones de las teclas
enmarcadas, con sen−1 , cos −1 y tan−1 , no son los recíprocos
de sen, cos y tan sino las funciones inversas de éstas, que se
estudiaran más adelante.

CLEMENTE MORENO (MATERIAL DIDÁCTICO EN VALIDACIÓN) 7


Ejemplo 1. Al calcular el sen 48° se tiene:

sen 48° = 0,743145 «la calculadora se coloca en modo grados (degree), se presiona 𝑠𝑠𝑠𝑠𝑠𝑠 y luego 48»

Ejemplo 2. Calcular el cos 19,19°

0,19° = 0,19 ∙ 60 = 11,4′ y 0,4′ ∙ 60 = 24′′ «la expresión decimal en grados se transforma en minutos y segundos»

«cos del ángulo se rescribió en grados minutos y segundos»


luego cos 19,19° = cos 19°11′24′′
11 24 «expresión del ángulo para incluirlo en la calculadora»
y 19°11′ 24′′ = 19 + + = 19,18999 …
60 3600
«se halla en la calculadora el cos 19,189� = 0,9444337596»

luego cos 19,19° = cos 19,189� = 0,944434

Ejemplo 3. En el triángulo rectángulo cuyos lados miden 5cm, 12cm y 13cm, halle las razones
trigonométricas definidas arriba
5cm 5
sen θ = o bien sen θ = «se aplicó definición de seno y simplificó unidades»
13cm 13

5cm 12cm 12
cos θ = o bien cos θ = «se aplicó definición de seno y simplificó unidades»
13cmo 13
θ
12cm 5cm 5cm
tg θ = o bien tg θ = «se aplicó definición de tangente y simplificó unidades»
12cm 12cm

13 13 12
Y csc θ = ; sec θ = ; ctg θ = «por reciproco de las razones sen θ, cosθ y tg θ halladas»
5 12 15

Relaciones entre las razones trigonométricas. Identidades trigonométricas

Considere de nuevo el triángulo rectángulo ∆ABC junto a C


𝛽𝛽
las razones definidas arriba, en estas condiciones se observa:
b
a b 1 a
(i) sen α = y csc α = entonces csc α =
b a sen α
c b 1 𝛼𝛼
cos α = y sec α = entonces sec α = A c
B
b c cos α

a c 1
tg α = y ctg α = entonces ctg α =
c a tg α

a c
(ii) sen α = implica b ∙ sen α = a (a) y cos α = implica b ∙ cos α = c (b), luego
b b
b∙sen α a sen α a sen α
= o = así tg α = «(a) y(b) se dividen miembro a miembro y simplifico por b»
b∙cos α c cos α c cos 𝛼𝛼

CLEMENTE MORENO (MATERIAL DIDÁCTICO EN VALIDACIÓN) 8


(iii) En el ∆ABC se tiene b2 = a2 + c 2 (∗) «a, b y c longitudes de los lados del ∆ABC y la identidad pitagórica»

a2 c2 b2 a 2 c 2
2 + = o bien � � + � � = 1 «(∗) se dividió miembro a miembro entre b2 se reescribió y simplificó»
b b2 b2 b b

(sen α)2 + (cos α)2 = 1 o bien sen2 α + cos 2 α = 1 (1) «identidad trigonométrica fundamental»

(iv) sen2 α cos2 α 1


+ = o bien tg 2 α + 1 = sec 2 α (2) «se divide (1) entre cos 2 α y tiene identidad de la 𝐭𝐭𝐭𝐭»
cos2 α cos2 α cos2 α

(v) sen2 α cos2 α 1


+ = o bien 1 + ctg 2 α = csc 2 α (3) «se divide (1) entre sen2 α y tiene identidad de la 𝐜𝐜𝐜𝐜𝐜𝐜
sen2 α sen2 α sen2 α

(vi) En el triángulo rectángulo ∆ABC los ángulos 𝛼𝛼 y 𝛽𝛽 son complementarios, luego


a a
(a) sen α = y cos β = entonces sen α = cos β
b b
c c
(b) cos α = y sen β = entonces cos α = sen β
b b
a a
(c) tg α = y ctg β = entonces tg α = ctgβ
c c
b b
(d) csc α = y sec β = entonces csc α = sec β
a a
b b
(e) sec α = y csc β = entonces sec α = csc 𝛽𝛽
c c
c c
(f) ctg α = y ctg β = entonces ctg α = ctg β
a a

Aplicaciones de las identidades trigonométricas


(i) Conocido el valor de una de las razones trigonométricas de un ángulo agudo, la identidad
fundamental permite conocer el valor de las otras razones trigonométricas de dicho ángulo
3
Ejemplo. Si sen α = con 𝛼𝛼 un ángulo agudo, entonces las otras razones trigonométricas del
4

ángulo son:

sen2 α + cos 2 α = 1 implica cos 2 α = 1 − sen2 α «aislando cos 2 α en el primer miembro de la igualdad»

3 2
Así cos α = √1 − sen2 α luego cos α = �1 − � � «despejando cos 𝛼𝛼 y sustituyendo el valor de sen α»
4

16 − 9 7 √7
O bien cos α = � =� es decir cos α = «operando y calculando raíz cuadrada»
16 16 4

3
sen α
Como tg α = entonces tg α = 4
«por definición de tg α dada en (ii) y sustitución de valores de seno y coseno»
cos 𝛼𝛼 √7
4

3 3√7
Así, tg α = o bien tg α = «operando y racionalizando»
√7 7

CLEMENTE MORENO (MATERIAL DIDÁCTICO EN VALIDACIÓN) 9


4 4 4√7 √7
Además, csc α = ; sec α = = y ctg α = «por aplicación de reciproco de las razones halladas»
3 √7 7 3

(ii) La identidades indicadas en (i), (ii), (iii), (iv) y (v) permiten establecer y/o demostrar nuevas
identidades. En este propósito se suele seguir las estrategias que se indican a continuación
(a) Se elige uno de los miembros de la identidad «usualmente el más complicado» y se van
haciendo transformaciones con apoyo en identidades ya demostradas, hasta llegar a la
expresión del segundo miembro de la igualdad en demostración

Ejemplo 1. Demostrar la igualdad cos 2 α − sen2 α = 2cos2 α − 1 se tiene que:

cos2 α − sen2 α = cos2 α − (1 − cos 2 α) «como sen2 α + cos2 α = 1 implica sen2 α = 1 − cos2 α, se sustituye en 1er lado»

= cos2 α − 1 + cos 2 α «los términos del paréntesis precedido del signo – cambia al ser operados»

«al sumar términos semejantes se llega a la expresión del 2do lado»


= 2 cos2 α − 1

Luego, cos2 α − sen2 α = 2cos2 α − 1 «quedando de este modo la identidad demostrada»

cosα 1 + senα
Ejemplo 2. Demostrar la identidad =
1 − senα cosα

cosα cos 𝛼𝛼 (1+𝑠𝑠𝑠𝑠𝑠𝑠 𝛼𝛼)


= ∙ «como
(1 + sen α)
= 1 si α ≠ 270°, el primer lado se multiplicó por 1 que es el neutro»
1 − senα (1−𝑠𝑠𝑠𝑠𝑠𝑠 𝛼𝛼) (1+𝑠𝑠𝑠𝑠𝑠𝑠 𝛼𝛼) (1 + senα)

cos α ∙ (1+sen α)
= «por diferencia de cuadrados (1 − sen α) ∙ (1 + sen α) = 1 − sen2 α»
1−sen2 α

cos α ∙ (1+sen α)
= «por la identidad trigonométrica fundamental 1 − sen2 α = cos2 α»
cos2 α

1+sen α
= «como
cos α
= 1 si α ≠ 90°, se puede dividir entre cos 𝛼𝛼»
cos α cos α

cosα 1 + senα
Luego, = «quedando de este modo la identidad demostrada»
1 − senα cosα

cos α sen α
Ejemplo 3. Demostrar la identidad − = sen α + cos α
1−tg α ctg α − 1

cos α sen α cos α sen α


− = sen α − cos α «como tg α =
sen α
y ctg α =
cos α
se sustituyen en el primer miembro»
1 − tg α ctg α − 1 1 − cos α −1 cos α sen α
sen α

CLEMENTE MORENO (MATERIAL DIDÁCTICO EN VALIDACIÓN) 10


cos α sen α
= cos α−sen α − cos α−sen α «resolviendo la suma de fracciones del denominador»
cos α sen α

cos2 α sen2 α
= − «aplicando la fracción compleja (doble C)»
cos α−sen α cos α−sen α

cos2 α−sen2 α
= «efectuando la resta de fracciones»
cos α−senα
(cos α−sen α)(cos α+sen α)
= «aplicando diferencia de cuadrados en el numerador»
cos α−sen α

= cos α + sen α para cos α ≠ sen α «


cos α − senα
cos α−sen α
= 1 siempre que cos α ≠ sen α»

cos α sen α
Luego − = sen α + cos α «quedando así demostrada la identidad»
1−tg α ctg α − 1

(b) La segunda forma de probar una identidad, consiste en ir trabajando con sustituciones
equivalentes cada uno de los dos miembros de la igualdad hasta lograr que ambos lados
sean iguales «estrategia que se aplica si la primera forma de proceder no da resultado»

Ejemplo 1. Demostrar que (sen3 α cosα + senα cose3 α)2 = sen4 αcos 2 α + sen2 αcos 4 α
(i) «se trascribió la igualdad dada»
3 3 2
? 4 2 2 4
(sen α cos α + senα cos α) = sen α cos α + sen cos α (ii) «se tomó sen α cos α como factor común en

2 ? ambos lados de la igualdad»


�senα cosα(sen2 α + cos2 α)� = sen2 α ∙ cos 2 α(sen2 α + cos 2 α� (iii) «se sustituyó sen2 α + cos2 α por 1, aplicando la
2 ? 2 2 identidad trigonométrica fundamental»
�senα cosα ∙ (1)� = sen α ∙ cos α ∙ (1)
(iv) «se efectuó el producto y estableció la igualdad»
sen2 α cos2 α = sen2 α cos2 α

Ejercicio N° 2
Halle las razones trigonométricas del ángulo que se indican en cada uno de los triángulos
rectángulos. Simplificar y racionalizar, cundo sea posible

(1) (2) (3) (4) 25


𝛾𝛾
2 10 13
12
𝛼𝛼 𝛽𝛽 24 7
14 𝜑𝜑
3 5

CLEMENTE MORENO (MATERIAL DIDÁCTICO EN VALIDACIÓN) 11


(5) 36 (6) 6 (7) 1 (8)
𝛽𝛽 𝛾𝛾
10
3 8
77 3
85
𝛼𝛼
6

En los casos que siguen, «c» representa a la hipotenusa y las otras dos letras los catetos de
un triángulo rectángulo. Dibuje una figura para cada uno indicando los ángulos opuestos a los
lados respectivos por las correspondientes letras mayúsculas. Partiendo de los lados dados
en cada caso, halle el tercero, luego determine las razones trigonométricas de los ángulos

(9) c = 42; b = 35; (10) m = 348; n = 526; (11) a = 346; b =?;


a =? c =? c = 500;

(12) x =?; y = 125; (13) c = 324; b = 180; (14) m = 608; n = 354;


c = 185; a =? c =?

15) Dado un triángulo rectángulo cuyos catetos miden 5,25cm y 6,05cm. Halle con ayuda de una
calculadora las razones trigonométricas que usted conoce

16) En un triángulo rectángulo isósceles se sabe que la hipotenusa es 4√2cm y uno de sus
catetos 4cm. ¿Cuánto valen todas las razones trigonométricas? ¿Tienes alguna observación
especial respecto a tus resultados?
17) Para un triángulo rectángulo con un ángulo agudo de 37°. Calcular los valores de todas las
razones trigonométricas para este ángulo y luego calcúlelas para el otro ángulo agudo del
mismo triángulo. ¿Observaste algunos valores iguales al calcular estas razones? ¿Cuáles?
18) Para un triángulo rectángulo con un ángulo agudo de 29°35′. Calcule los valores de todas
las razones trigonométricas para este ángulo y luego calcúlelas para el otro ángulo agudo
del mismo triángulo

Demostrar cada una de las identidades trigonométricas dadas a continuación


19) cos2 α − sen2 α = 1 − 2sen2 α; (20) cos2 α = (1 − senα) ∙ (1 + senα);
21) 2sen2 β + cos2 β = 1 + sen2 β; (22) sen2 αcos2 α + cos4 α = 1 + sen2 α;

CLEMENTE MORENO (MATERIAL DIDÁCTICO EN VALIDACIÓN) 12


1 1 2 sen α 1 + cosα 2
23) + = ; (24) + = ;
1 + sen α 1 − sen α cos2 α 1 + cos α senα sen α
cos α sen α 1 1
25) + = ; (26) senβ � − senβ� = cos2 β;
sen α 1+cos α sen α senβ

27) cos4 δ − sen4 δ + 1 = 2cos2 δ; (28) sen4 δ − cos 4 δ = 2cos 2 δ − 1;


29) (senα + cosα)2 + (senα − cosα)2 = 2; (30) sen3 αcosα + cos3 αsenα = senα cosα;

cos2 α sen2 α
31) = 1 + senα; (32) = 1 − cosα;
1 − senα 1 + cosα
1−senα cosα
33) = ; (34) sen6 α + cos6 α = cos4 + sen4 α − sen2 αcos 2 α;
cosα 1+senα

35) sen4 α − cos4 α + cos2 α = sen2 α; (36) (sen3 αcosα + senαcos3 α)3 = sen5 αcos3 α + sen3 αcos5 α;

Razones trigonométricas de ángulos notables «30°, 45° y 60°»


Aunque la calculadora, como se señaló arriba, proporciona los valores de las razones de
cualquier ángulo es útil hacer el ejercicio de calcular los valores que toman estas razones para
ángulos de uso frecuente como son los de 30°, 45° y 60°
π
Razones trigonométricas del ángulo de «45° o »
4
45°
√2
Considere un triángulo rectángulo con catetos de longitud a, en a

cuyo caso los ángulos agudos son iguales a 45° y la hipotenusa h por 45°
a
Pitágoras es h = √a2 + a2 o bien h = a√2, luego:

a √2
sen 45° = o bien sen45° = «se aplicó la definición de seno y racionalizó»
a√2 2

a √2
cos45° = o bien cos45° = «se aplicó la definición de coseno y racionalizó»
a√2 2
a
tg 45° = o bien tg 45° = 1 «se aplicó la definición de tangente y operó»
a

a√2
csc 45° = o bien csc 45° = √2 «se aplicó la definición de cosecante y operó»
a

a√2
sec 45° = o bien sec 45° = √2 «se aplicó la definición de secante y operó»
a
a
ctg 45° = o bien ctg 45° = 1 «se aplicó la definición de cotangente y operó»
a

CLEMENTE MORENO (MATERIAL DIDÁCTICO EN VALIDACIÓN) 13


π π
Razones trigonométricas de los ángulos «30° o y 60° o »
6 3

Considere un triángulo equilátero con lados de longitud 2 y B

trace la bisectriz del ángulo ABC que lo divide en dos ángulos de 30° 30°
a a
30°, la cual coincide con la altura BD trazada desde el lado AC. Con
60° 60°
ello el triángulo equilátero se divide en dos triángulos rectángulos A a D a C
2 2
como muestra la figura. En uno de esos triángulos rectángulos, la
B
altura BD se calcula a partir del teorema de Pitágoras, es decir: 30°
a a
√3
a 2 a 2
a2 = � � + BD2 , así BD = √3. En este triángulo rectángulo se 60°
2 2 A D
a
tiene: 2

𝑎𝑎 «30° + 60° = 90°, los ángulos son complementarios»


1 1
sen 30° = , 2
luego sen30° = y cos 60° = ;
a 2 2

a√3 «30° + 60° = 90°, los ángulos son complementarios»


√3 √3
cos 30° = 2
, luego cos30° = y sen 60° = ;
a 2 2

a «30° + 60° = 90°, los ángulos son complementarios»


√3 1 √3
tg 30° = 2
a√3
, luego tg30° = y ctg 60° = = ;
3 √3 3
2

a 2 «30° + 60° = 90°, los ángulos son complementarios»


csc 30° = a , luego csc30° = 2 y sec 60° = = 2;
1
2

a 2√3 2 2√3 «30° + 60° = 90°, los ángulos son complementarios»


sec 30° = a√3
, luego sec30° = y csc 60° = = ;
3 √3 3
2

a√3 «30° + 60° = 90°, los ángulos son complementarios»


√3
ctg 30° = 2
a , luego ctg30° = √3 y tg 60° = = √3;
1
2

Una regla que permite evocar y calcular las razones trigonométricas de los ángulos 0°,
30°, 45°, 60° y 90° es como se indica a continuación:
(i) «Se toma la raíz cuadrada de cantidad subradical una matriz con 3 filas y 6
0° 30° columnas, en la primera fila se colocan los ángulos, en la segunda la razón seno
45° 60° 90°
seguida de la secuencia 0, 1, 2, 3, 4 en ese orden, en la tercera la razón coseno
sen 0 1 2 3 4 seguida de la secuencia 4,3,2,1,0 en ese orden»
cos 4 3 2 1 0
(ii) «la raíz descripta en (i) es el numerador de la fracción de denominador 2»
2
(iii) «al número ubicado en la intersección de la columna con ángulo (en este caso 45°) y la fila con razón
√2
(a) sen45° = ; (en este caso seno) se le toma la raíz cuadrada y se divide entre 2»
2

√3 (iv) √3
(b) cos 30° = ; «3 se ubica en la intersección de la columna con ángulo 30° y la razón coseno, luego cos 30° =
2
»
2

CLEMENTE MORENO (MATERIAL DIDÁCTICO EN VALIDACIÓN) 14


1 (v) 1
«1 se ubica en la intersección de la columna con ángulo 30° y la razón seno, luego sen 30° = »
(c) sen 30° = ; 2
2
1
√3
(d) tg 30° = 2
√3
= ; (vi)
3 «como se sabe, conocido el seno y coseno del ángulo, se conocen las otras razones trigonométricas»
2

En ocasiones hay que efectuar operaciones combinadas con las razones de los ángulos
notables y/u otros ángulos que se identificaran más adelante

Ejemplo 1. Al calcular el valor de sen30° + cos 60° + tg 30° + ctg 60° se tiene:

1 1 √3 √3
sen30° + cos 60° + tg 30° + ctg 60° = + + + «las razones se sustituyen por su respectivo valor»
2 2 3 3

2√3
=1+ «operando fracciones con igual denominador»
3

3 + 2√3
Luego, sen30° + cos 60° + tg 30° + ctg 60° = «efectuando la suma de fracciones»
3

cos 60° + sen2 45°


Ejemplo 2. Al calcular el valor de , se tiene:
2sen 30° + cos 0°

2
1 √2
cos 60° + sen2 45° 2
+ �2�
= 1 «las razones se sustituyen por su respectivo valor»
2sen 30° + cos 0° 2∙2 + 1

1 1
+2
=2 «efectuando operaciones»
1+1

cos 60° + sen2 45° 1


Luego, = «efectuando operaciones»
2sen 30° + cos 0° 2

Resolución de triángulos rectángulos


Con lo estudiado hasta el momento se está en condiciones de resolver triángulos
rectángulos, ello significa que conocidos los valores de algunos de los elementos de un triángulo
rectángulo, como serian: un ángulo y un lado o dos lados, hallar los valores de los otros dos
lados y ángulos, aplicando las propiedades estudiadas y el valor de las razones trigonométricas
Todo triángulo rectángulo que se requiera resolver, está en uno de los casos siguientes:
(i) Se conoce un lado y un ángulo, el lado puede ser un cateto o la hipotenusa
(ii) Se conocen los dos catetos del triángulo rectángulo

CLEMENTE MORENO (MATERIAL DIDÁCTICO EN VALIDACIÓN) 15


(iii) Se conoce la hipotenusa y uno de los catetos

Ejemplo 1. Al hallar el valor de los catetos «x» y «y» del triángulo rectángulo, adjunto se tiene:
x
cos30° = implica x = 20 ∙ cos30° «aplicando definición de coseno y despejando»
20 20
y
√3
30° Así, x = 20 ∙ o bien x = 10√3 «sustituyendo y operando»
2
x
y
sen30° = implica y = 20 ∙ sen30° «aplicando definición de seno y despejando»
20

1
Luego, y = 20 ∙ o bien y = 10 «sustituyendo y operando»
2

Ejemplo 2. Al calcular el perímetro del triángulo rectángulo dado en la figura, se tiene:

P = x + y + 28 «el perímetro es la suma de la longitud de los lados del triángulo»


60°
y x
28 tg60° = así x = 28 ∙ tg60° o bien x = 28√3 «cálculo de longitud de x»
28
28 28
x cos60° = así y = «aplicación de coseno en cálculo de hipotenusa»
y cos 60°

28
Así, y = 1 entonces y = 48 «sustituyendo y operando para el cálculo de longitud de x»
2

Luego, P = 28√3 + 48 + 28 o bien P = 28√3 + 76 «longitud del perímetro del triángulo»

COMENTARIO

La visual es una recta imaginada que va desde el ojo


del observador «o punto de observación» hasta el objeto
observado; el horizonte es una recta horizontal pensada a la
α
altura de los ojos de quien observa «o punto de observación»; β
Horizonte

el ángulo 𝛼𝛼 arriba de la horizontal y la visual es de elevación;


el ángulo 𝛽𝛽 debajo de la horizontal y la visual es de depresión

CLEMENTE MORENO (MATERIAL DIDÁCTICO EN VALIDACIÓN) 16


Ejemplo 3. Desde un punto colocado a una distancia de 40m de la base de un edificio se midió
el ángulo de elevación de la parte alta y se vio que era de 60°30′. ¿Cuál es la altura del edificio?
30
C 60°30′ = 60 + o bien 60°30′ = 60,5 «transformación del ángulo»
60
����
BC
tg 60,5 = ���� o bien ����
BC = ����
AB ∙ tg60,5 «aplica tangente en cálculo de altura ����
BC»
AB

����
BC = 40 ∙ 1,76749 o bien ����
BC = 70,699 m «sustituyendo y operando»
60°30′
A
B 40m Luego, h = 70,7 m «es la altura del edificio aproximado a las decimas»

Ejemplo 4. Desde lo alto de un acantilado de 100m sobre el nivel del agua, el ángulo de
depresión de un barco es de 24°. ¿A qué distancia del acantilado se encuentra el barco?

𝛽𝛽 β = 24° « β coloreado en negro es el ángulo de depresión»


C
β=β « β en negro y β en rojo son alternos internos, luego son iguales»

����
BC ����
BC
���� =
tg24° = ���� luego AB ����»
«aplica tangente en cálculo de distancia AB
𝛽𝛽 AB tg24°
A B 100m
����
AB = o bien ����
AB = 224,6m «distancia del barco al acantilado»
0,445228

Ejemplo 5. A determinada hora del día la altura del sol sobre el horizonte es de 49° y en ese
momento un árbol proyecta una sombra de 27m sobre el suelo. ¿Cuál es la altura del árbol?
h
tg49° = así h = 27 ∙ tg49° «aplicando tangente en cálculo del a altura h»
27m

h = 27 ∙ (1,1504) o bien h = 31,06m


h= ?

«sustituyendo y operando»

49°
Luego, h = 31,06m «es la altura del árbol aproximado a las centésimas»

27m

Ejemplo 6. Dos observadores que están a una distancia de 140m, situados a lados opuestos
de un globo inmóvil en el aire, lo ven con ángulos de 44°56′ y 36°4′ ,respectivamente. ¿A qué
altura se encuentra el globo?

CLEMENTE MORENO (MATERIAL DIDÁCTICO EN VALIDACIÓN) 17


α = 44°56′ o bien α = 44,93� «transformación del ángulo α»

β = 36°4′ o bien β = 36,06� «transformación del ángulo β»

h
tgα = implica h = x ∙ tg�44,93� � (1) «aplicando tangente en cálculo h»
h x
𝛼𝛼 𝛽𝛽 h
140 − x
tgβ = así h = (140 − x)tg(36,06� ) (2) «aplicando tg en cálculo h»
x 140−x
140

h = 0,9976 ∙ x (1); y h = (140 − x) ∙ 0,7281 (2); «sustituyendo valores en (1) y (2)»

Así, 0,9976 ∙ x = 101,934 − 0,7281 ∙ x «los antecedentes (1) y (2) son iguales, los consecuentes son iguales»

y, 0,9976x + 0,7281x = 101,934 o bien 1,7257x = 101,934 «agrupando términos semejantes y operando»

101,934
Así, x = o bien x = 59,068 (3) «despejando y operando»
1,7257

Luego, h = 0,9976 ∙ (59,068) o bien h = 58,93 «es la altura del globo, obtenida al sustituir (3) en (1) y operar»

Ejemplo 7. Desde cierto punto se ve la parte más alta de un edificio con un angulo de elevación
de 43°. Si el punto se mueve 30m hacia el edificio, el ángulo de elevación aumenta en 7°. ¿Cuál
es la altura del edificio?

α = 43° y β = 43° + 7° = 50° «aplicando datos del problema»

h h
tgβ = o bien x = «aplicando definición de tangente para calcular h»
x tgβ
h
Asi, x = h ∙ ctgβ; (1) «aplicando el reciproco de tangente que es cotagnte»

h h
tgα = o bien 30 + x = «aplicando definición de tangente para hallar h»
30+x tgα
𝛼𝛼 𝛽𝛽
30
30 + x
x Así, x = h ∙ ctgα − 30; (2) «aplicando el reciproco de tangente y despejando a x»

x = h ∙ ctgβ y x = h ∙ ctgα − 30 implica h ∙ ctgβ = h ∙ ctgα − 30 «por (1) y (2) los consecuentes son iguales»

Así, h ∙ ctgβ − h ∙ ctgα = −30 o bien h(ctgα − ctgβ) = 30 «agrupando y tomando h como factor común»

30 30
Luego, h = o bien h = «despejando y sustituyendo los valres de α y β»
ctgα − ctgβ ctg43° − ctg50°

30 30
Y, h = o bien h = así h = 128,61m; «operando se tiees la altura del edificio»
1,0723687−0,83909963 0,2332724

CLEMENTE MORENO (MATERIAL DIDÁCTICO EN VALIDACIÓN) 18


Ejercicio N° 3
Calcule el valor de cada una de las siguientes expresiones
√3 sen30° π π π 3π
1) + cos2 45° + cos2 180°; (2) sen2 + sen − sen + cos 0° + sen + cosπ;
cos 30° 4 2 6 2

π π 3 1
2sen 4 + 2sen 3 csc 60° + tg�7π� − sec(4π)
3) π π; (4) 3 5 ;
cos2 6 + cos2 3 sen �8π� + cos�12π�

π π π π π π π π π
2ctg� 6 � + 2tg� 3 � − 2√3 cos� 6 � + 6sen� 4 � cos� 4 � �ctg� 3 � − ctg� 6 �� ∙ �tg� 3 � + tg� 6 ��
5) π π π π ; (6) π π π π ;
3tg� 3 � + 2sen� 3 � − 3ctg� 6 � + 6√3 cos� 3 � �ctg� 6 � − ctg� 3 �� ∙ �tg� 6 � + tg� 3 ��

En cada uno de los triángulos adjuntos, determine las incógnitas que se señalan

7) (8) (9)

y y

30° 60°
x x

10) (11) (12)


y x
3 h
x h
45° 30°
y 30° 45°
x y z

De cada uno de los triángulos adjuntos, determine su perímetro y área

13) (14) (15)


12
20
32
60°
30° 45° 30°

16) Resuelva el triángulo rectángulo en el cual 𝛼𝛼 = 23,7° y cuya hipotenusa mide 17,5cm
17) Resuelva el triángulo rectángulo si 𝛼𝛼 = 33,5° y cuyo cateto opuesto al ángulo mide 63,5cm
18) Resuelva el triángulo rectángulo si su hipotenusa mide 47,3cm y uno de sus catetos 18,4cm
19) Para hallar la distancia entre las orillas de una laguna, un topógrafo elige dos puntos A y B,
uno en cada orilla y directamente opuestos entre sí. En la orilla donde ubicó a A y a una
���� quedó en forma
distancia de 50m de éste, elige otro punto C, de modo que el segmento AC
perpendicular con el segmento ����
AB, además logró medir el ángulo que forman los segmentos
���� y CB
CA ���� encontrando que medía 67,5°. ¿Cuál es la distancia de una a otra orilla del lago?

CLEMENTE MORENO (MATERIAL DIDÁCTICO EN VALIDACIÓN) 19


20) Desde un punto A el ángulo de elevación con que Juan observa la cúspide de una colina es
de 37,5°, si camina 54m acercándose a ella horizontalmente, el nuevo ángulo de elevación
es de 47,3°. ¿Cuál es la altura de la colina?
21) A una determinada hora del día, el ángulo de elevación del sol es 43,7° ¿Cuál es la longitud
de la sombra que tu proyectas en el suelo?
22) Una vara de 9m está apoyada sobre un muro inclinado. El pie de la vara está a 4,3m del
pie del muro y, desde este punto hasta donde la vara toca al muro, hay una distancia de 7m.
¿Cuál es la medida del ángulo de inclinación del muro con respecto a la vertical?
23) Dos patrullas de reconocimiento parten del mismo punto, una hacia el Oeste, la otra al
Noroeste, cuando cada una ha recorrido 15 km, ¿qué distancia las separa?
24) Desde un barco se ven dos faros, uno a 19,7 km y otro a 26,2 km, con un ángulo de 107,15° ,
¿Qué distancia hay entre los faros?
25) A 9 km al norte del puerto de la Guaira se encuentra un barco y 11,3 km hacia el noreste
se encuentra un velero. ¿Qué distancia separa las embarcaciones?

COMENTARIO

El estudio de las razones trigonométricas realizado hasta el momento alcanza su grado


de generalización máxima, cuando estos objetos matemáticos se estudian a partir del concepto
de función, tal como se muestra a continuación

Funciones trigonométricas
Considere un sistema de coordenadas en el plano y una circunferencia de radio 1, con
centro en el origen y un ángulo de rotación «α» de vértice en el origen que transforma el vector
𝑦𝑦 �����⃗
OA «unitario» en el vector �����⃗
OP, el cual al ser el transformado de 𝚤𝚤⃗
�����⃗ = 𝚤𝚤⃗» mediante la rotación, también es unitario, así �OP
«OA �����⃗� = 1 y
sen α
𝜃𝜃 𝑥𝑥
sus componentes �����⃗
OP = (a, b). En tales condiciones se define:
0 𝚤𝚤⃗ A(1, 0)
cos α
(i) cos α = a �����⃗»
«el coseno de α es la abscisa, primera componente de OP

(ii) sen α = b «el seno de α es la ordenada, segunda componente de �����⃗


OP»

CLEMENTE MORENO (MATERIAL DIDÁCTICO EN VALIDACIÓN) 20


COMENTARIO

Nótese que al aislar el triángulo rectángulo de la circunferencia unitaria, la longitud del


cateto adyacente a «α» es «a», la del cateto opuesto es «b», la de la hipotenusa ����
OP = 1, así:
a
P(a, b) cos α = o bien cos α = a «por definición de razón coseno»
1
b
b sen α = o bien sen α = b «por definición de razón seno»
1
𝛼𝛼
O b
a tg α = para a ≠ 0 «por definición de razón tangente»
a

�����⃗� = r, donde
Lo cual coincide con la definición que se está construyendo. Además, si �OB
�����⃗ �����⃗, es decir (m, n) = (ra, rb) se tiene:
OB = (m, n) = rOP
m cateto adyacente
B(m, n) cos α = o bien cos α = «definición de coseno»
r hipotenusa
𝐫𝐫 n cateto opuesto
sen α = o bien sen α = «definición de seno»
P(a, b) n r hipotenusa
n cateto opuesto
𝛼𝛼 A(1, 0) tg α = o bien tg α = «definición de tangente»
m m cateto adyacente
0 C
r r m
sec α = ; csc α = y ctg α = «reciprocas de las primeras»
m n n

Hecho que lleva nuevo a la definición de las razones estudiadas arriba

De regreso a la definición de las funciones trigonométricas, las coordenadas del vector


�����⃗
OP = (cos α , sen α ) implican que en el primer cuadrante ambas funciones son positivas, pero
�����⃗ rota a los otros cuadrantes las funciones adquieren el signo de las componentes del
cuando OP
par propio de cada cuadrante, así:

𝑦𝑦 𝑦𝑦 𝑦𝑦

𝛼𝛼
𝛼𝛼 𝑥𝑥 𝑥𝑥
𝑥𝑥 𝛼𝛼

cos α = −a; sen α = b cos α = −a; sen α = −b cos α = a; sen α = −b

CLEMENTE MORENO (MATERIAL DIDÁCTICO EN VALIDACIÓN) 21


COMENTARIO

En matemática cuando se habla de funciones trigonométricas se entiende que el domino


de definición de ellas está dado en radianes «sistema circular», pues de este modo si «x» es
positivo «x > 0» a partir de A se recorre sobre la circunferencia una distancia «x» en sentido
positivo «figura a»; si «x» es negativo «x < 0» se recorre la distancia |x| en sentido negativo
«figura b». En ambos casos se obtiene un punto P sobre la circunferencia, además si «x = 0»
se tiene P = A y las coordenadas del punto P sobre la circunferencia se llaman coseno y seno

ℝ ℝ
P x

ℝ ℝ
A A
x

P
figura a figura b

De este modo se tiene: (i) cos(x) = abscisa de P


(ii) sen(x) = ordenada de P
O bien cos x y sen x «sin paréntesis», luego para cada número real «x» se tiene el
número real cos x; junto al número real sen x, es decir dos funciones de ℝ en ℝ
(i) 𝑓𝑓: ℝ → ℝ⁄𝑓𝑓(𝑥𝑥) = cos x
(ii) 𝑔𝑔: ℝ → ℝ⁄𝑓𝑓(𝑥𝑥) = sen x
No obstante, debido a la equivalencia entre el sistema sexagesimal «ángulos» y el
sistema circular «radianes», en este curso se hablara de funciones trigonométricas,
entendiendo que la equivalencia entre los sistemas permitirá trabajarlas como funciones reales
cuando el contexto al que se refieran, así lo amerite

La relación fundamental de la trigonometría

B(0,1)
�����⃗� = 1
Como �OP �����⃗ es unitario, su módulo es 1»
«el vector OP

P(a, b)
�����⃗� = √a2 + b 2
Y �OP «aplicando definición de módulo del vector»
sen α

𝜃𝜃 Luego √a2 + b 2 = 1 �����⃗� por su valor»


«sustituyendo �OP
0 A(1, 0)
cos α

CLEMENTE MORENO (MATERIAL DIDÁCTICO EN VALIDACIÓN) 22


Así, a2 + b2 = 1 «elevando ambos miembros al cuadrado para simplificar el radical»

En consecuencia cos2 α + sen2 = 1 (1) «es la relación fundamental de la trigonometría»

la cual es consistente con la identidad deducida en el inciso (iii) de la página 9

La relación (1) implica que cos2 α ≤ 1 y sen2 α ≤ 1 de donde se sigue que |cos 𝛼𝛼| ≤ 1 o
bien −1 ≤ cos α ≤ 1; asimismo |sen α| ≤ 1 o bien −1 ≤ sen α ≤ 1. Lo cual indica que el rango
de las funciones seno y coseno es el intervalo [−1, 1], es decir que toman valores comprendidos
entre −1 y 1

Además cos2 α + sen2 α = 1 implica cos 2 α = 1 − sen2 α o bien cos α = √1 − sen2 α; de


igual modo cos 2 α + sen2 α = 1 implica sen2 α = 1 − cos2 α o también senα = √1 − cos 2 α, que
son resultados equivalentes a los indicados en la página 9

1
Ejemplo 1. Sabiendo que sen α = y que α ∈ II « α termina en el 2do cuadrante», hallar el
3

valor de las otras funciones trigonométricas

Como cos 2 α + sen2 α = 1 implica cos α = √1 − sen2 α «despejando cos α de la relación fundamental»

1 2 9−1 2√2
cos α = �1 − � � o bien cos α = � así cos α = − «sustituyendo, operando y aplicando α ∈ II»
3 9 3

1
sen α 3
tg α = implica tg α = 3
2√2
o bien tg α = − «aplicando tg α, sustituyendo y operando»
cos α − 3(2√2)
3

1 √2
Así, tg α = − o también tg α = − «simplificando y racionalizando»
2√2 4
1 1 2
sec α = implica sec α = 2√2
o bien sec α = − «aplicando sec α, sustituyendo y operando»
cos α − 2√2
2

1 √2
Así, sec α = − o también sec α = − «simplificando y racionalizando»
√2 2
1 1
csc α = implica csc α = 1 o bien csc α = 3 «aplicando csc α, sustituyendo y operando»
sen α
3

2√2
cos α − 3
ctg α = implica ctg α = 1 o bien ctg α = −2√2 «aplicando ctg α, sustituyendo y operando»
sen α
3

CLEMENTE MORENO (MATERIAL DIDÁCTICO EN VALIDACIÓN) 23


3
Ejemplo 2. Sabiendo que cos β = − y que β está en el III cuadrante, hallar el valor de las
4

otras funciones trigonométricas

Como cos 2 α + sen2 α = 1 implica senα = √1 − cos 2 α «despejando sen α de la relación fundamental»

3 2 16−9 √7
senα = �1 − �− � o bien senα = � , así senα = − «sustituyendo, operando y aplicando β ∈ III»
4 16 4

√7
sen α − √7
tg α = implica tg α = 4
3 , luego tg α = «aplicando tg α, sustituyendo y operando»
cos α −4 3

1 1 4
sec α = implica sec α = 3 , luego sec α = − «aplicando sec α, sustituyendo y operando»
cos α −4 3

1 1 4√7
csc α = implica csc α = , luego csc α = − «aplicando csc α, sustituyendo, operando y racionalizando»
sen α −
√7 7
4

3
cos α −4 3√7
ctg α = implica ctg α = , luego ctg α = «aplicando ctg α, sustituyendo, operando y racionalizando»
sen α √7
−4 7

Otras funciones trigonométricas


A partir de las funciones seno y coseno se definen las otras funciones trigonométricas
sen α
Función tangente. tg α = , si cos α ≠ 0
cos α
1
Función secante. sec α = , si cos α ≠ 0
cos α
1
Función cosecante. csc α = , si sen α ≠ 0
sen α
cos α
Función cotangente. ctg α = , si sen α ≠ 0
sen α

Hecho que coincide con las definiciones de estos objetos matemáticos en términos de
razones, pero acá se excluyen los ceros en el denominador, indicando que por ejemplo las
funciones tg α y sec α no están definidas si cos α = 0, lo cual ocurre digamos cuando α = 90°,
270°, −90°, −270°, 450°, etc. De igual modo, csc α y ctg α no están definidas si sen α = 0, lo
cual ocurre digamos cuando α = 0°, 180°, 360°, −180°, −360°
Como se indicó arriba, las calculadoras por lo general dan, los valores de senα, cos α,
tg α para un valor dado α, al presionar la tecla que las identifica; para obtener los valores de las
otras funciones, se debe obtener una de esas tres razones y hallar el reciproco. No obstante,
para el estudio tabular y gráfico de estas funciones, es bueno tener presente el valor de las

CLEMENTE MORENO (MATERIAL DIDÁCTICO EN VALIDACIÓN) 24


funciones trigonométrica en los ángulos extremo, por ello, en el apartado que sigue se muestran
estos valores

π 3
Valor de las funciones trigonométricas en 0° o 0π; 90° o ; 180° o π; 270° o π y 270° o 2π
2 2

En la circunferencia unitaria, considere el vector �����⃗


0A «unitario», coincidente con 𝚤𝚤⃗, junto

ℝ a la rotación que transforma el vector �����⃗


0A en sí mismo, creando el
B(0,1)
ángulo nulo y las componentes de �����⃗
0A = (1,0), así los valores de:
𝚥𝚥⃗
𝚤𝚤⃗ ℝ cos 0° = 1 «cos(x) = abscisa de A que es 1»
C(−1,0) −𝚤𝚤
����⃗ 0 A(1,0)
����⃗
−𝚥𝚥 sen 0° = 0 «sen(x) = ordenada de A que es 0»

0
D(0, −1) tg 0° = o bien tg 0° = 0 «tg (x) =
sen x
, pues cos 0° ≠ 0»
1 cos x

En atención a la reciprocas de seno, coseno y tangente, se tiene sec 0° = 1; csc 0° = ∞ y


ctg 0° = ∞
����(𝚤𝚤⃗) en el vector ����
Considere ahora la rotación que transforma el vector OA OB(𝚥𝚥⃗), generando
el ángulo recto, luego las componentes del vector �����⃗
OB = (0,1), en cuyo caso se tiene:
π
cos 90° = 0 o su equivalente cos � � = 0 «cos(x) = abscisa de B que es 0»
2

π
sen 90° = 1 o su equivalente sen � � = 1 «sen(x) = ordenada de B que es 1»
2

1 π
tg 90° = así tg 90° = ∞ o su equivalente tg � � = ∞ «tg (x) =
sen x π
implica tg � � = ∞, pues cos 90° = 0»
0 2 cos x 2

Al considerar las reciprocas de coseno, seno y tangente resulta sec 90° = ∞; csc 90° = 1
y ctg 90° = 0
����(𝚤𝚤⃗) en el vector OC
De nuevo, piense la rotación que transforma el vector OA ����(−𝚤𝚤
����⃗), trazando
�����⃗ = (−1,0), en cuyo caso se tiene:
el ángulo llano, de modo que las componentes del vector OC

cos 180° = −1 o su equivalente cos π = −1 «cos(x) = abscisa de C que es −1»

sen 180° = 0 o su equivalente sen π = 0 «sen(x) = ordenada de C que es −1»

0
tg 180° = así tg 180° = 0 o su equivalente tgπ = 0 «tg (x) =
sen x
implica tg π = 0, pues cos 180° ≠ 0»
−1 cos x

Y según las reciprocas de coseno, seno y tangente resulta sec 180° = −1; csc 180° = ∞ y
ctg 180° = ∞

CLEMENTE MORENO (MATERIAL DIDÁCTICO EN VALIDACIÓN) 25


����(𝚤𝚤⃗) en el vector ����
Nuevamente, considere la rotación que transforma el vector OA OD(−𝚥𝚥
����⃗),
�����⃗ = (0, −1), en cuyo caso se tiene:
trazando el ángulo 270°, allí las componentes del vector OD
3
cos 270° = 0 o su equivalente cos � π� = 0 «cos(x) = abscisa de D que es 0»
2

3
sen 270° = −1 o su equivalente sen � π� = −1 «sen(x) = ordenada de D que es −1»
2

−1 3π
tg 270° = así tg 270° = ∞ o también tg � � = ∞ «tg (x) =
sen x 3
implica tg � π� = ∞, pues cos 270° = 0»
0 2 cos x 2

Al tomar las reciprocas de coseno, seno y tangente resulta sec 270° = ∞; csc 270° = −1 y
ctg 270° = 0
�����⃗ realiza un giro completo alcanza su posición inicial, luego las funciones coseno
Cuando OA
y seno del ángulo completo son iguales a las del ángulo nulo, es decir:

cos 360° = 1 o su equivalente cos 2π = 1

sen 360° = 0 o su equivalente sen 2π = 0


0
tg 360° = así tg 360° = 0 o también tg (2π) = 0 «tg (x) =
sen x
implica tg 360° = 0 pues cos 360° ≠ 0»
1 cos x

Y para las reciprocas de coseno, seno y tangente resulta sec 360° = 1; csc 360° = ∞ y
ctg 360° = ∞. Debido a que al alcanzar el giro, los valores de las funciones se repitan, asoma
la característica de la periodicidad de dichas funciones, elemento central en su estudio tabular
y gráfico a realizar en el próximo apartado

Ejercicio N° 4
Halle el valor del resto de las funciones trigonométricas, en cada uno de los casos siguientes:
2 2 13
1) sen α = , 𝛼𝛼 ∈ II; (2) sen α = , 𝛼𝛼 ∈ I; (3) sen α = , 𝛼𝛼 ∈ II;
3 √5 15

3 2 15
4) cos α = − , 𝛼𝛼 ∈ II; (5) cos α = , 𝛼𝛼 ∈ I; (6) cos α = − , 𝛼𝛼 ∈ II;
5 √7 17

3 4 7
7) sen α = − , 𝛼𝛼 ∈ III; (8) sen α = − , 𝛼𝛼 ∈ III; (9) sen α = − , 𝛼𝛼 ∈ III;
7 √11 9

4 3 121
10) cos α = − , 𝛼𝛼 ∈ III; (11) cos α = − , 𝛼𝛼 ∈ III; (12) cos α = − , 𝛼𝛼 ∈ III;
13 √13 143

12 2 11
13) sen α = − , 𝛼𝛼 ∈ IV; (14) cos α = , 𝛼𝛼 ∈ IV; (15) sen α = − , 𝛼𝛼 ∈ IV;
17 √13 12

CLEMENTE MORENO (MATERIAL DIDÁCTICO EN VALIDACIÓN) 26


Sin utilizar calculadora, halle el valor de cada una de las expresiones siguiente:

16) 3
2 − sen6π (17) π π π 2 π
π 21π 1 − cos3π tg 3 − tg 6 �sec 6 � − 3 csc 3
�tg + 18π � �cos + �; π 2π + π ;
4 1 + sen 3 1 + tg 2π sec 4 + tg 8 1 + tg 3 ∙ tg750°
4

18) 2 − cos6π
3
(19) π π π 2 π
π 21π 1 − csc3π ctg 3 − ctg 6 �csc 6 � − 3 sec 3
�ctg + 18π� �sec + �; π 2π + π ;
4 1 + sen 3 1 + ctg π csc 4 + ctg 8 1 + ctg 3 ∙ ctg1110°
4

20) En relación con los triángulos rectángulos de la figura adjunta,


4 3
el 𝑠𝑠𝑠𝑠𝑠𝑠 𝛼𝛼 = y 𝑡𝑡𝑡𝑡 𝛽𝛽 = . Entonces la longitud del cateto 𝛼𝛼
β
5 5
x
designado con «x» es

21) Desde lo alto de una montaña de 3.000 metros de altura se observaron dos pueblos,
situados en el llano, con ángulos de depresión de 60° y 45° respectivmente. ¿A qué distancia
está un pueblo del otro? «se supone los dos pueblos alineados al pie de la montaña»
22) La altura de una montaña cuya base y cumbre eran inaccesibles se determinó midiendo el
ángulo de elevación de la cumbre desde un punto situado a cierta distancia en una llanura
horizontal y se vio que era de 30°; se repitió la medida desde un punto alejado 1 km más
allá de la montaña y se vio que en este caso el ángulo era de 25°. ¿Cuál es la altura de la
montaña?
23) Desde un globo estático que se encuentra a una altura de 600m, un observador dirige una
visual a una trinchera en una llanura y halla que su ángulo de depresión es de 35°, dirige
otra visual a una ametralladora situada más allá de la trinchera y encuentra que su ángulo
de depresión es de 20°. ¿A qué distancia se encuentra la ametralladora de la trinchera?
24) Mientras vuela a una altura de 350 metros, el piloto observa que el ángulo de depresión de
un aeropuerto es de 15°. ¿Qué distancia hay en este momento entre el avión y el
aeropuerto?
25) un observador en el suelo ve hacia el este una cometa con un ángulo de elevación de 59°20′,
y otra persona lo ve hacia el oeste con un ángulo de elevación de 34°15. Si estos dos
observadores están separados 200 metros, ¿a qué altura está la cometa?

CLEMENTE MORENO (MATERIAL DIDÁCTICO EN VALIDACIÓN) 27


Representación gráfica de las funciones trigonométricas
Las funciones trigonométricas son centrales en su futuro estudio de la matemática, ellas
se aplican en el análisis de las ondas sonoras y de radio, rayos X y gamma, ondas sísmicas y
oceánicas, circuitos y generadores eléctricos, vibraciones, construcciones de puentes y
edificios, ondas ocasionadas por los remos de un bote, etc. En muchos de estos casos, las
gráficas de estas funciones pueden brindar insumos para dar solución a los problemas
provenientes de estos contextos, por ello, en lo que sigue se abordará el estudio gráfico de las
tres primeras funciones

Estudio de la función seno «𝑓𝑓(𝑥𝑥) = sen x»


Considere la circunferencia unitaria e imagine que el punto P se mueve sobre ésta, desde
π
el punto A(x = 0) hasta el punto B �x = � y observe lo que ocurre a ℝ
2
B P
la ordenada de P que es senx. Al respecto, se puede ver que a medida P
P

sen x
π
que «x» crece de 0 a , «senx» crece de 0 a 1, pasando una sola x
2
0 A ℝ
vez por los puntos intermedios, lo cual implica que la función «seno»
π
envía el intervalo �0, � de manera biyectiva sobre el intervalo [0, 1]
2

en forma creciente
π
De igual modo, imagine que el punto «P» se mueve sobre la circunferencia desde B �x = �
2

hasta C(x = π). En este caso, se observa que senx decrece desde 1

π
hasta 0 en la medida que «x» va desde a π; así, la función «seno» P B P
P
2 P
P P
π
envía de manera biyectiva el intervalo � , π� sobre el intervalo [0, 1]
sen x
sen x

2 x
A ℝ
en forma decreciente. C 0

Razonando de manera análoga, se observa que cuando «P» va


3
desde C(x = π) a D �x = π�, «senx» decrece desde 0 a −1, luego D
2
3
la función seno envía el intervalo �π, π� de manera biyectiva sobre el intervalo [−1, 0] en forma
2
3
decreciente. Por último, si «P» se mueve desde D �x = π� hasta A(x = 2π), «sen𝑥𝑥» crece
2
3
desde −1 a 0, lo cual significa que la función «seno» envía el intervalo � π, 2π� de manera
2

biyectiva sobre el intervalo [−1, 0] en forma creciente

CLEMENTE MORENO (MATERIAL DIDÁCTICO EN VALIDACIÓN) 28


Con lo apuntado arriba, se puede construir la gráfica de 𝑓𝑓(𝑥𝑥) = senx en el intervalo [0, 2π];
para ello, se divide tanto el intervalo como la circunferencia en «n» partes iguales; digamos en
este caso, 24 partes iguales, tal como se ilustra en la figura

ℝ ℝ

3
π
2 2π
ℝ π π ℝ
2

Cuando el arco «x» se describe en sentido horario se tiene la gráfica de 𝑓𝑓(𝑥𝑥) = senx en el
intervalo [−2π, 0]. Si el arco «x» corresponde a «n» vueltas completas, la grafica 𝑓𝑓(𝑥𝑥) = senx,
se repite en intervalos de longitud 2π a la derecha de 2π cuando el arco se describe en sentido
antihorario y a la izquierda de −2π si el arco se realiza en sentido horario, indicando que la
función es periódica de período p = 2π, tal como se ilustra en la figura adjunta

1
5 π 3
− π − π
2 2 2

−4π −3π −2π − 3 π −π π π 2π 5
π 3π 4π
2 2 2
−1

La gráfica de 𝑓𝑓(𝑥𝑥) = senx también puede construirse hallando algunas imágenes de 𝑓𝑓(𝑥𝑥)
tanto en x ∈ [0, −2π] como en x ∈ [0, 2π], la tabla adjunta muestra algunos de estos valores

3 π π π π π π π π 3
x −2π − π −π − − − − 0 π π 2π
2 2 3 4 8 8 4 3 2 2
senx 0 1 0 −1 −0,9 −0,7 −0,4 0 0,4 0,7 0,9 1 0 −1 0

CLEMENTE MORENO (MATERIAL DIDÁCTICO EN VALIDACIÓN) 29


Otras consideraciones al dibujar la gráfica de «𝑓𝑓(𝑥𝑥) = A sen kx» y «𝑓𝑓(𝑥𝑥) = Asen(kx + m)»
En el caso de «𝑓𝑓(𝑥𝑥) = A sen kx», la constante «A» es la amplitud de la gráfica de la función
y = sen x, medida a partir del eje de las abscisas, en el caso en mención A = 1, implica que la
máxima distancia entre los puntos de la curva y el eje «x» es 1, tal distancia se comprime para
0 < A < 1 y dilata cuando A > 1, además la curva se invierte cuando A < 0
Por su parte, «kx» habla de la periodicidad de las funciones trigonométricas. En general,
la expresión 𝑓𝑓(x + p) = 𝑓𝑓(x) define a la función periódica. Para el caso de la función y = sen x,
que tiene como periodo a p = 2π, al calcular el periodo de y = sen (kx), se tiene en cuenta que:
𝑓𝑓(x + p) = sen k(x + p) o bien 𝑓𝑓(x + p) = sen(kx + kp) «aplicando 1er lado de la definición y operando»

Y, sen(kx + kp) = sen kx = 𝑓𝑓(x) restando kp = 2π (∗) «aplicando 2do lado de la definición»


Luego, por (∗) p= «expresión para el cálculo del periodo de y = sen kx»
k

Ejemplo 1. Al graficar la función 𝑓𝑓(𝑥𝑥) = 2sen2x en el intervalo −π ≤ x ≤ 2π se tiene que:


(i) A = |2| o bien A = 2 «es la máxima amplitud de los puntos de la curva con respecto al eje x»

2π «el periodo cubre el intervalo [0, π], en él, se eligen 4 puntos para evaluar la función,
(ii) p= o bien p = π los más apropiados son donde alcanza el máximo, el mínimo y los cortes con los ejes»
2

𝑓𝑓(0) = 2 sen(2.0) o bien 𝑓𝑓(0) = 0 «evaluación de la función en el primer punto del intervalo»

𝜋𝜋 π π
𝑓𝑓 � � = 2 sen �2 ∙ � o bien 𝑓𝑓 � � = 2 «evaluación de la función en el punto donde alcanza el máximo»
4 4 4
(iii) π π π
𝑓𝑓 � � = 2sen �2 ∙ � o bien 𝑓𝑓 � � = 0 «evaluación de la función en el punto donde corta al eje de las abscisas»
2 2 2

3 3 3
𝑓𝑓 � π� = 2sen �2 ∙ π� o bien 𝑓𝑓 � π� = −2 «evaluación de la función en el punto donde alcanza el mínimo»
4 4 4

𝑓𝑓(𝜋𝜋) = 2 sen(2 ∙ π) o bien 𝑓𝑓(π) = 0 «evaluación de la función en el último punto del intervalo»

π «con estos valores se elabora una muestra del estudio tabular, que corresponde a
(iv) x 0
π 3
π π la parte periódica «a repetir» de la gráfica en el intervalo solicitado»
4 2 4

y 0 −2 «la parte periódica se repite en el intervalo requerido»


2 0 0

2 «gráfica de 𝑓𝑓(𝑥𝑥) = 2sen2x en el intervalo


π π 3
(v) − − π −π ≤ x ≤ 2π »
2 4 4
−π 3 π ℝ
− π
π π 2π
4 4 2

−2

(vi) 𝑓𝑓: [−π, 2π] → [−2, 2]⁄𝑓𝑓(𝑥𝑥) = 2 sen (2x) «notación de la función estudiada destacando dominio y rango»

CLEMENTE MORENO (MATERIAL DIDÁCTICO EN VALIDACIÓN) 30


m
En el caso de «𝑓𝑓(𝑥𝑥) = A sen(kx + m)» o su equivalente «𝑓𝑓(𝑥𝑥) = A senk �x + �», la gráfica
k
m
de y = A sen kx, sufre un desfasamiento «traslación en el domino» de magnitud « » unidades
k
m m m
hacia la derecha cuando < 0 y de « » unidades hacia la izquierda cuando >0
k k k

3
Ejemplo 2. Al graficar la función 𝑓𝑓(𝑥𝑥) = sen (2x + π) en el intervalo −π ≤ x ≤ π se tiene:
4

(i) 3 3
A = � � o bien A = «es la máxima amplitud de los puntos de la curva con respecto al eje x»
4 4


(ii) p= o bien p = π π π
«el periodo cubre el intervalo �− , �, en él, se eligen los puntos para evaluar la función»
2 2 2

m π π
(iii) = y >0 «la curva se traslada
𝜋𝜋 𝜋𝜋 𝜋𝜋
unidades a la izquierda de 0 y el período está en el intervalo �− , �»
k 2 2 2 2 2

𝜋𝜋 π π
«después de − , el �− , � se divide en 4 partes, se hallan las imágenes, ubican los
π π π π 2 2 2
x −
2

4
0 4 2 puntos y dibuja la parte periódica»
(iv) 3 3
y 0 0 − 0 «la parte periódica se repite en el intervalo requerido»
4 4


3
4
(v) π π

2 4
ℝ 3
«gráfica de 𝑓𝑓(𝑥𝑥) = sen(2x + π) en el intervalo −π ≤ x ≤ π»
−π 3

π 3 π 4
− π π
4 4 4
3

4

(vi) 3 3 3
𝑓𝑓: [−π, π] → �− , ��𝑓𝑓(𝑥𝑥) = sen (2x + π) «notación de la función estudiada destacando dominio y rango»
4 4 4

COMENTARIO

En párrafos precedentes, las funciones trigonométrica se aplicaron en la resolución de


triángulos «cálculo de longitudes», pero en ocasiones es necesario hallar el ángulo del cual se
conoce el valor del seno, coseno o tangente. Eso ocurre, digamos: al calcular los ángulos de
un triángulo conocida la longitud de sus lados; al calcular el ángulo con el cual se debe lanzar
un proyectil para dar en el blanco; al determinar la dirección de un barco o un avión que se

CLEMENTE MORENO (MATERIAL DIDÁCTICO EN VALIDACIÓN) 31


mueven bajo los efectos de una corriente. Estos cálculos sólo son posibles a partir de la
aplicación de las funciones trigonométricas inversas

Función arcoseno
π π
Cuando el dominio de la función seno se restringe al intervalo �− , � y el rango es [−1, 1]
2 2
π π
entonces 𝑓𝑓: �− , � → [−1,1]⁄𝑓𝑓(𝑥𝑥) = sen x, es una función estrictamente creciente, indicando
2 2

que es biyectiva y admite inversa, denominada seno inverso, denotada por «sen−1 » o arcoseno,
π π
de modo que 𝑓𝑓 −1 : [−1, 1] → �− , �⁄arcosen x = x
2 2

Algunos puntos del estudio tabular en el intervalo


considerado se muestran en la tabla y su gráfica en la figura

x √2 1 1 √2
−1 − − 0 1
2 2 2 2
π π π π π π
sen−1 (x) − − − 0
2 4 6 6 4 2

√3
Ejemplo. Al calcular α =? sabiendo que sen α = se tiene
2

√3
sen−1 (sen α) = sen−1 � � «tomando la inversa de seno en ambos lados de la igualdad»
2

√3
α = sen−1 � � o bien α = 60° «de la inversa compuesta con la directa resulta la idéntica»
2

Nota. Este aspecto se aplica en la solución de ecuaciones trigonométricas al final del capítulo

Estudio de la función coseno «𝑓𝑓(𝑥𝑥) = cosx»


La gráfica de la función 𝑓𝑓(𝑥𝑥) = cosx puede elaborarse siguiendo un procedimiento similar al
utilizado en la construcción de la gráfica de la función 𝑠𝑠𝑠𝑠𝑠𝑠𝑠𝑠, pero en este caso, el estudio se
hace sobre la abscisa del punto circular perteneciente a la circunferencia unitaria; esto es, para
cada 𝑥𝑥 ∈ [0, 2π] se toma la abscisa del punto, que es precisamente cos x y se copia como
ordenada del punto del gráfico correspondiente a «x». Para ello, tanto la circunferencia unitaria
como el intervalo [0, 2π] se dividen en «n» partes iguales, digamos por ejemplo en 24 partes
iguales, tal como se indica en la figura adjunta

CLEMENTE MORENO (MATERIAL DIDÁCTICO EN VALIDACIÓN) 32


ℝ ℝ


ℝ π π 3 2π
π
2 2

π
La gráfica de la función 𝑓𝑓(𝑥𝑥) = cosx coincide con la gráfica de 𝑓𝑓(𝑥𝑥) = sen �x + � debido
2
π π π
a que como veremos más adelante sen �x + � = senx. cos + cosx. sen = cosx. Además, la
2 2 2

gráfica de 𝑓𝑓(𝑥𝑥) = cosx puede dibujarse hallando algunas imágenes de 𝑓𝑓(𝑥𝑥) con x ∈ [0, −2π] y
también con en x ∈ [0, 2π], tal como se muestra a continuación

3 π π π π π π π π 3
x −2π − π −π − − − − 0 π π 2π
2 2 3 4 8 8 4 3 2 2
cosx 1 0 −1 0 0,5 0,7 0,9 1 0,9 0,7 0,5 0 −1 0 1


1

−3π −2π −π 0 π 2π 3π ℝ

−1

COMENTARIO

Al realizar el estudio gráfico de las funciones «𝑓𝑓(𝑥𝑥) = A cos kx» y «𝑓𝑓(𝑥𝑥) = Acos(kx + m)»
se tiene en cuanta lo que se indicó en estos casos para la función seno, es decir la amplitud
2π m
«A», el período «p = » y el desfasamiento « », calculados de manera similar a como se
k k

hizo en aquella función

CLEMENTE MORENO (MATERIAL DIDÁCTICO EN VALIDACIÓN) 33


πx
Ejemplo 1. Al graficar la función 𝑓𝑓(𝑥𝑥) = −3 cos � � en el intervalo −4 ≤ x ≤ 4 se tiene que:
2

(i) A = |−3| = 3, y como −3 < 0 la curva se invierte «amplitud e inversión de la curva por ser A < 0»

2π 2π
(ii) p= implica p = π o bien p = 4 «el periodo está [0, 4], en él, se eligen los puntos para evaluar la función»
k
2

π∙0
𝑓𝑓(0) = −3 cos � � o bien 𝑓𝑓(0) = −3 «evaluación de la función en el primer punto del intervalo»
2
π∙1
𝑓𝑓(1) = −3 cos � � o bien 𝑓𝑓(1) = 0 «evaluación de la función en el punto donde corta al eje de las abscisas»
2
π∙2
(iii) 𝑓𝑓(2) = −3 cos � � o bien 𝑓𝑓(2) = 3 «evaluación de la función en el punto donde alcanza el máximo»
2
π∙3
𝑓𝑓(3) = −3 cos � � o bien 𝑓𝑓(3) = 0 «evaluación de la función en el punto donde corta al eje de las abscisas»
2
π∙4
𝑓𝑓(4) = −3 cos � � o bien 𝑓𝑓(4) = −3 «evaluación de la función en el punto donde alcanza el mínimo»
2

(iv) «con estos valores se elabora una muestra del estudio tabular, que corresponde a
x 0 1 2 3 4 la parte periódica «a repetir» de la gráfica en el intervalo solicitado»

y −3 0 3 0 −3 «la parte periódica se repite en el intervalo requerido»


3

(v) πx
«gráfica de 𝑓𝑓(𝑥𝑥) = −3cos � � en el intervalo −4 ≤ x ≤ 4»
2

−4 −3 −2 −1 0 1 2 3 4 ℝ

−3

(vi) 𝑓𝑓: [−π, π] → [−3, 3]⁄𝑓𝑓(𝑥𝑥) = −3 cos �πx� «notación de la función estudiada destacando dominio y rango»
2

1
Ejemplo 2. Al graficar la función 𝑓𝑓(𝑥𝑥) = cos(4x − π) en el intervalo −π ≤ x ≤ π se tiene:
2

(i) 1 1
A = � � o bien A = «es la máxima amplitud de los puntos de la curva con respecto al eje x»
2 2
m π π
(ii) =− y − <0 π
«la curva se traslada � � unidades a la derecha de 0»
k 4 4 4

(iii) p = 2π implica p = 2π o bien p = π π 3


«el período está en � , π�, donde se toman los puntos para evaluar la función»
k 4 2 4 4

π 1 π π 1
𝑓𝑓 � � = cos �4 ∙ − π� o bien 𝑓𝑓 � � = «evaluación de la función en el primer punto del intervalo»
4 2 4 4 2
π 1 π π
𝑓𝑓 � � = cos �4 ∙ − π� o bien 𝑓𝑓 � � = 0 «evaluación de la función en el punto donde corta al eje de las x»
8 2 8 8
π 1 π π 1
(iv) 𝑓𝑓 � 2 � = 2 cos �4 ∙ 2 − π� o bien 𝑓𝑓 � 2 � = − 2 «evaluación de la función en el punto donde alcanza el mínimo»

CLEMENTE MORENO (MATERIAL DIDÁCTICO EN VALIDACIÓN) 34


5 1 5 5
𝑓𝑓 � π� = cos �4 ∙ π − π� o bien 𝑓𝑓 � π� = 0 «evaluación de la función en el punto donde corta al eje x»
8 2 8 8
3 1 3 3 1
𝑓𝑓 � π� = cos �4 ∙ π − π� o bien 𝑓𝑓 � π� = «evaluación de la función en el punto donde alcanza el máximo»
4 2 4 4 2
«con estos valores se elabora una muestra del estudio tabular, que corresponde a la
π π π 5 3 parte periódica «a repetir» de la gráfica en el intervalo solicitado»
x π π
4 8 2 8
(v) 4
«la parte periódica se repite en el intervalo requerido»
y 1 1 1
0 − 0
2 2 2

1 ℝ
2

(vi) −π 3 π π π π 3 π ℝ 1
«gráfica de 𝑓𝑓(𝑥𝑥) = cos(4x − π) en el intervalo
− π −2 −
4 4 2
π 2
4 4 −π ≤ x ≤ π
1

2
(vi) 𝑓𝑓: [−𝜋𝜋, 𝜋𝜋] → �− 1 , 1��𝑓𝑓(𝑥𝑥) = 1 cos(4x − π) «notación de la función estudiada destacando dominio y rango»
2 2 2

COMENTARIO

Función arcocoseno
Si el dominio de la función coseno se restringe al intervalo [0, π] y como su rango [−1,1],
entonces 𝑓𝑓: [0, 𝜋𝜋] → [− 1,1⁄𝑓𝑓(𝑥𝑥) = cos x, es una función estrictamente decreciente, luego es
biyectiva y admite inversa, llamada coseno inverso, denotada por «cos −1 » o bien arcocoseno,
definida como: 𝑓𝑓 −1 : [−1, 1] → [0, π]⁄ arcocos x = x
Algunos puntos del estudio tabular en el intervalo
considerado se muestran en la tabla y su gráfica en la figura
1 1
x −1 − 0 1
2 2
2 π π
cos −1 (x) π π 0
3 2 3
1
Ejemplo. Al calcular φ =? sabiendo que cos φ = se tiene:
2

1
cos −1 (cos φ) = cos −1 � � «tomando la inversa de coseno en ambos lados de la igualdad»
2
1
φ = cos −1 � � o bien φ = 60° «de la inversa compuesta con la directa resulta la idéntica»
2

Nota. Este aspecto se aplica en la solución de ecuaciones trigonométricas al final del capítulo

CLEMENTE MORENO (MATERIAL DIDÁCTICO EN VALIDACIÓN) 35


Estudio de la función tangente «𝑓𝑓(𝑥𝑥) = tg x»
La tangente del arco �
AP dado en la figura, se puede ℝ

interpretar como el segmento de tangente determinado por el P

tg x
x
punto circular de la circunferencia unitaria que corresponde al arco

π 0 A
«x», cuando «x» está entre 0 y . De este modo, la tg x es la
2

longitud del segmento de tangente dado por «x». A partir de estos


π π 3 3
segmentos se puede trazar la gráfica de la tangente en el intervalo �0, � ∪ � , π� ∪ � π, 2π�.
2 2 2 2
π π 3
Sin embargo, la figura adjunta exhibe la gráfica de la tangente en el Intervalo �0, � ∪ � , π�
2 2 2

Para dibujarla, se sigue un proceso similar al utilizado cuando se realizó el dibujo de las
gráficas de las funciones «seno» y «coseno». Esto es,

π
se divide la circunferencia y el intervalo �0, � ∪
2
π 3 3
� , π� ∪ � π, 2π� en «n» partes iguales, luego se
2 2 2
π
copian las longitudes de estos arcos en el eje de las 0

abscisas, en ellos se levantan rectas perpendiculares
que se intersectan con las rectas horizontales que
vienen de los puntos de tangencia. En particular, al
dividir la circunferencia en veinticuatro partes iguales
se genera la gráfica de 𝑓𝑓(𝑥𝑥) = tg x en el intervalo considerado
La gráfica de la función tangente entre −2π y 2π, puede dibujarse ubicando algunos pares
π π
del 𝑔𝑔𝑔𝑔𝑔𝑔(tg) en el plano, unidos por un trazo continuo entre sus asíntotas en x = − y x= ,
2 2

lo cual indica que la función es periódica de período p = π, la curva allí obtenida se traslada a
π π
la izquierda de − y a la derecha , tal como se exhibe en el estudio tabular y gráfico adjunto,
2 2

donde se muestra la curva entre −2π y 2π

3 π π π π π π π π 3
x −2π − π −π − − − − 0 π π 2𝜋𝜋
2 2 3 4 8 8 4 3 2 2
tg x 0 ∞ 0 ∞ −1,7 −1 −0,4 0 0,4 1 1,7 ∞ 0 ∞ 0

CLEMENTE MORENO (MATERIAL DIDÁCTICO EN VALIDACIÓN) 36



2

π π
−2π − 3 π −π − 0 π 3 2π ℝ
2 2 π
2 2
−1

−2

COMENTARIO

Como ocurre en las funciones «seno» y «coseno», al realizar el estudio tabular y gráfico
π
de «𝑓𝑓(𝑥𝑥) = A tg (kx)» y «𝑓𝑓(𝑥𝑥) = A tg(kx + m)» se tiene en cuenta que el período «p = » y el
k
m
desfasamiento « » se calculan siguiendo el proceso dado en las funciones «seno» y «coseno»
k

Ejemplo 1. Al graficar la función 𝑓𝑓(𝑥𝑥) = 3 tg 2x en el intervalo −π ≤ x ≤ π se tiene:

(i) A = |3| o bien A = 3 «la amplitud marca el inicio del comportamiento asintótico»

π π
(ii) p= implica p = π π
«el periodo cubre el intervalo �− , �, en él, se eligen los puntos para evaluar la función»
k 2 4 4

𝜋𝜋 𝜋𝜋 𝜋𝜋
𝑓𝑓 �− � = 3 tg �2 ∙ �− �� o bien 𝑓𝑓 �− � = −∞ «evaluación de la función en el punto que reporta la asíntota»
4 4 4

𝜋𝜋 π 𝜋𝜋
𝑓𝑓 �− � = 3tg �2 ∙ �− �� o bien 𝑓𝑓 �− � = −3 «evaluación de la función en el punto que muestra la amplitud»
8 8 8
(iii)
𝑓𝑓(0) = 3 tg(2 ∙ 0) o bien 𝑓𝑓(0) = 0 «evaluación de la función en el punto donde corta al eje de las abscisas»

𝜋𝜋 π 𝜋𝜋
𝑓𝑓 � � = 3 tg �2 ∙ � �� o bien 𝑓𝑓 � � = 3 «evaluación de la función en el punto que muestra la amplitud»
8 8 8

𝜋𝜋 π 𝜋𝜋
𝑓𝑓 � � = 3 tg �2 ∙ � �� o bien 𝑓𝑓 � � = +∞ «evaluación de la función en el punto que reporta la asíntota»
4 4 4
«con estos valores se elabora una muestra del estudio tabular, que corresponde a
(iv) x −
π

π
0
π π
la parte periódica «a repetir» de la gráfica en el intervalo solicitado»
4 8 4 8

y −∞ −3 0 3 +∞
«la parte periódica se repite en el intervalo requerido»

CLEMENTE MORENO (MATERIAL DIDÁCTICO EN VALIDACIÓN) 37


6
3
(v)
π π π π 3 «gráfica de 𝑓𝑓(𝑥𝑥) = 3 tg 2x en el intervalo −π ≤ x ≤ π»
−π 3 − 0 π ℝ
− π − 4 −3 π
4 2 4 2 4
−6

(vi) 𝑓𝑓: �−π, − 3 π� ∪ �− 3 π, − π� ∪ �− π , π� ∪ �π , 3 π� ∪ �3 π, π� → ℝ⁄f(x) = 3 tg 2x «notación matemática»


4 4 4 4 4 4 4 4

COMENTARIO

Función arcotangente
π π
Al tomar solo el intervalo �− , � como dominio de la función tangente y rango el conjunto
2 2
π π
ℝ, la función f: �− , � → ℝ�f(x) = tg x, es una función estrictamente decreciente, indicando
2 2

que es biyectiva y admite inversa, llamada tangente inverso, notada por «tg −1 » o arcotangente,
π π
de modo que 𝑓𝑓 −1 : ℝ → �− , ��arcotang x = x
2 2

Algunos puntos del estudio tabular en el intervalo


considerado se muestran en la tabla y su gráfica en la figura

−1 √3 √3
x x → −∞ −√3 − 0 1 √3 x→∞
3 3
π π π π π π π π
tg −1 (x) y → − − − − 0 y→
2 3 4 6 6 4 3 2

√3
Ejemplo. Al calcular ϑ =? sabiendo que tg ϑ = resulta:
3

√3
tg −1 (tg ϑ) = tg −1 � � «tomando la inversa de la tangente en ambos lados de la igualdad»
3

√3
ϑ = tg −1 � � o bien ϑ = 30° «de la inversa compuesta con la directa resulta la idéntica»
3

Nota. Este aspecto se aplica en la solución de ecuaciones trigonométricas al final del capítulo

CLEMENTE MORENO (MATERIAL DIDÁCTICO EN VALIDACIÓN) 38


Estudio de la función secante «𝑓𝑓(𝑥𝑥) = sec x»

COMENTARIO

π
Para la interpretación geométrica de la «sec x», considere un número «x» entre 0 y , en
2

el que «P» es el punto que le corresponde en la circunferencia ℝ


unitaria. Si por «P» se traza la tangente a la circunferencia, dicha P

tangente es perpendicular al radio ����


OP e intersecta al eje de las x ℝ
O Q
abscisas en el punto «Q», formando el triángulo ΔOPQ, que es
rectángulo en «P», tal como se ilustra en la figura a la derecha
hipotenusa OQ
Como sec x = implica sec x = entonces sec x = OQ. Dicho de otro modo,
cateto adyacente OP

la «sec x» es la abscisa del punto «Q» determinado sobre el eje de las abscisas por la tangente
a la circunferencia en el punto «P»

En atención al comentario, para dibujar la gráfica de la función 𝑓𝑓(𝑥𝑥) = sec x, basta calcular
OQ para arcos concordantes con algunos ángulos notables en cada uno de los cuadrantes, en
los que se ha dividido la circunferencia unitaria de la figura. Así en el primer cuadrante se tiene:
1
sec(0π) = implica sec(0π) = 1 ℝ
cos( 0𝜋𝜋)

π OQ 1 2 2√3
sec � � = implica OQ = √3
así OQ = o bien OQ =
6 1 √3 3
2
0 ℝ
π OQ 1 2
sec � � = implica OQ = así OQ = o bien OQ = √2
4 1 √2 √2
2

π OQ 1 π 1 π
sec � � = implica OQ = 1 así OQ = 2 y sec � � = π implica sec � � = ∞
3 1 2 cos� 2 � 2
2

En el segundo cuadrante OQ es negativo, pues allí el coseno es negativo, luego se tiene:


π OQ 1
−sec � � = implica −OQ = 1 entonces OQ = −2 π 2
«−sec � � = sec � π� respecto al I cuadrante»
3 1 3 3
2

π OQ 1 2
−sec � � = implica −OQ = así OQ = − = −√2 π 3
«−sec � � = sec � π� respecto al I cuadrante»
4 1 √2 √2 2 4
2

π OQ 1 2 2√3
−sec � � = implica −OQ = √3
así OQ = − =− π 5
«− sec � � = sec � π� respecto al I cuadrante»
6 1 √3 3 6 6
2

CLEMENTE MORENO (MATERIAL DIDÁCTICO EN VALIDACIÓN) 39


1
sec π = implica sec π = −1
cos π

Para el tercer cuadrante OQ también negativo, debido a que el coseno es negativo, así:
π OQ 1 2 2√3
−sec � � = implica −OQ = √3
así OQ = − =− π 7
«aquí − sec � � = sec � π� respecto al I cuadrante»
6 1 √3 3 6 6
2

π OQ 1 2
−sec � � = implica −OQ = así OQ = − = −√2 π 5
«aquí − sec � � = sec � π� respecto al I cuadrante»
4 1 √2 √2 4 4
2

π OQ 1
−sec � � = implica −OQ = 1 entonces OQ = −2 π 4
«aquí − sec � � = sec � π� respecto al I cuadrante»
3 1 3 3
2

3 1 3
sec � π� = 3 implica sec � π� = ∞
2 cos�2π� 2

En el IV cuadrante OQ es positivo, pues como se verá más adelante cos(−α) = cos α, luego
π OQ 1
sec �− � = implica OQ = 1 entonces OQ = 2 π 5
«aquí sec �− � = sec � π� respecto al I cuadrante»
3 1 3 3
2

π OQ 1 2
sec �− � = implica OQ = así OQ = = √2 π 7
«aquí sec �− � = sec � π� respecto al I cuadrante»
4 1 √2 √2 4 4
2

π OQ 1 2 2√3
sec �− � = implica OQ = √3
así OQ = = π 11
«aquí sec �− � = sec � π� respecto al I cuadrante»
6 1 √3 3 6 6
2

1
sec 2π = implica sec 2π = 1
cos 2π

Con estos valores se elabora una muestra del estudio tabular entre 0 y 2π, como se indica

π π π π 2π 3π 5π 7π 5π 4π 3π 5π 7π 11π
x 0 π 2π
6 4 3 2 3 4 6 6 4 3 2 3 4 6
sec x 1 1,15 1,41 2 ∞ −2 −1,41 −1,15 −1 −1,15 −1,41 −2 ∞ 2 1,41 1,15 1

La ubicación de estos puntos en el plano junto a las asíntotas genera el bosquejo de la


π π 3 3π
gráfica de función en el intervalo �0, � ∪ � , π� ∪ � , 2π�, tal como muestra la figura adjunta
2 2 2 2

0 π π 3
π 2π ℝ
−1 2 2

CLEMENTE MORENO (MATERIAL DIDÁCTICO EN VALIDACIÓN) 40


COMENTARIO

Tanto el estudio tabular como el dibujo de la gráfica de la función «𝑓𝑓(𝑥𝑥) = csc x», se
proponen como ejercicio al lector

Ejercicio N° 5
Aplicando lo indicado en los párrafos anteriores, dibuje la gráfica de las siguientes funciones
trigonométricas, destacando amplitud, periodo, desfasamiento y estudio tabular

1) 𝑓𝑓(𝑥𝑥) = 4 sen x, en −2π ≤ x ≤ 2π; (2) 2


𝑔𝑔(𝑥𝑥) = cos x, en −2π ≤ x ≤ 2π;
3

3) 2
𝑦𝑦 = − cos 𝑥𝑥, en −2π ≤ x ≤ 2π; (4) 𝑦𝑦 = −3 𝑠𝑠𝑠𝑠𝑠𝑠 𝑥𝑥, en −2π ≤ x ≤ 2π;
3

5) ℎ(𝑥𝑥) = 2 sen 3x, en −π ≤ x ≤ 2π; (6) 𝑙𝑙(𝑥𝑥) = 3 cos 2x, en −π ≤ x ≤ π;

7) x
𝑓𝑓(𝑥𝑥) = 3 cos � �, en −4π ≤ x ≤ 4π; (8) 𝑥𝑥
𝑔𝑔(𝑥𝑥) = 4 𝑠𝑠𝑠𝑠𝑠𝑠 � �, en −6π ≤ x ≤ 6π;
2 3

9) 𝑦𝑦 = 4 sen (πx), en −2 ≤ x ≤ 2; (10) 𝑦𝑦 = 3 cos(πx), en −2 ≤ x ≤ 2;

11) 𝑙𝑙(𝑥𝑥) = 3 sen (2πx), en −2 ≤ x ≤ 2; (12) 𝑓𝑓(𝑥𝑥) = 2 cos(2πx), en −2 ≤ x ≤ 2;


5 3

13) 𝑔𝑔(𝑥𝑥) = 2 sen �x + π�, en −π ≤ x ≤ 2π; (14) 𝑙𝑙(𝑥𝑥) = 2 sen �x − π�, en −π ≤ x ≤ 3π;
2 2

15) 𝑓𝑓(𝑥𝑥) = 4 cos �x − π�, en −π ≤ x ≤ 3π; (16) ℎ(𝑥𝑥) = 3 cos �x + π�, en −2π ≤ x ≤ 2π;
5 4 4

17) 𝑚𝑚(𝑥𝑥) = 3sen[π(x − 1)], en −2 ≤ x ≤ 3; (18) 𝑦𝑦 = 2 cos �2π �x − 1��, en −1 ≤ x ≤ 2;


2

19) 𝑔𝑔(𝑥𝑥) = 2 cos �πx + π�, en −2 ≤ x ≤ 2; (20) ℎ(𝑥𝑥) = 3 sen �πx − π�, en −1 ≤ x ≤ 3;
2 2

21) 𝑓𝑓(𝑥𝑥) = 4 tg 2x, en −π ≤ x ≤ π; (22) 𝑔𝑔(𝑥𝑥) = 2 ctg 4x, en 0 ≤ x ≤ π;


3 2

23) 𝑙𝑙(𝑥𝑥) = 5 tg (2x + π), en −π ≤ x ≤ π; (24) ℎ(𝑥𝑥) = −2ctg(πx − π), en −1 ≤ x ≤ 2;


6

CLEMENTE MORENO (MATERIAL DIDÁCTICO EN VALIDACIÓN) 41


Relaciones trigonométricas
(i) Coseno y seno del ángulo opuesto

Considere las rotaciones que transforman el vector �����⃗


OA en el vector �����⃗
OP, describiendo el
ángulo positivo «α» y en el vector �������⃗
OP ′ describiendo ángulo negativo « −𝛼𝛼», tal como muestra
la figura, en tales condiciones se observa que:


cos α = a y cos(−α) = a , así cos(−α) = cos α (4)
P(a, b)
Y sen α = b implica −sen α = −b «multiplicando ambos lados por (−1)»

𝛼𝛼 ℝ
0 −𝛼𝛼 Sí sen(−α) = −b y −sen α = −b entonces sen(−α) = −senα (∗)
A(1, 0)

P ′ (a, −b) Luego, sen(−α) = −senα (5) «(∗) es la igualdad entre el seno de α y −α»

(ii) Coseno y seno de la suma y diferencia de dos ángulos


�����⃗ en el vector �����⃗
De nuevo, considere la rotación que transforma el vector OA OP describiendo el
�����⃗ en el vector �����⃗
ángulo «𝛼𝛼» junto a la rotación que transforma el vector OA OQ describiendo el
ángulo «β», donde las componentes de �����⃗
OP y OQ�����⃗ son: OP
�����⃗ = (cosα, senα) y OQ
�����⃗ = (cosβ, senβ),
tal como indica la figura. En tales condiciones se tiene:

ℝ B(0,1) �����⃗
OP ∙ OQ�����⃗ = �OP
�����⃗� ∙ �OQ
�����⃗�cos(α − β) (∗) «definición de producto escalar»
P(senα, cos α)
Q(senβ, cos β) �����⃗� = 1 y �OQ
�����⃗� = 1
Pero �OP «el radio de la circunferencia unitaria es 1»
α β ℝ
0 A(1, 0) �����⃗ ∙ �����⃗
Así OP OQ = cos(α − β) (∗∗) �����⃗� y �OQ
«sustituyendo los valores de �OP �����⃗� en (∗)»

(cosα, senα) ∙ (cosβ, senβ) = cos(α − β) «sustituyendo �����⃗


OP y �����⃗
OQ en (∗∗)»

Luego, cos(α − β) = cos α cosβ + senα senβ (6) «definición de producto escalar según sus componentes»

Ahora al tener en cuenta que cos(α + β) = cos[α − (−β)] y aplicar (6) resulta:

cos(α + β) = cos[α − (−β)] «el producto de números con igual signo es positivo»

= cosα cos(−β) + senα sen(−β) (∗) «aplicando el 2do miembro de cos(α − β) dado en (3)»

Luego, cos(α + β) = cos α cos β − senα senβ (7) «aplicando (1) y (2) en (∗) anterior»

CLEMENTE MORENO (MATERIAL DIDÁCTICO EN VALIDACIÓN) 42


COMENTARIO

En párrafos precedente, se mostró la relación entre el coseno y el seno de los ángulos


π
complementarios, al respecto se indicó que α + β = tal como muestra la figura adjunta
2
π π
α+β= implica β = � − α� «despejando a β»
β 2 2
c a π a
cos � − α� = cos β = (∗) «definición de coseno en un triángulo rectángulo»
2 c
α
a
b sen α = (∗∗) «definición de seno en un triángulo rectángulo»
c
π
Luego, cos � − α� = senα (a) «igualando las identidades dadas en (∗) y (∗∗)»
2
b b π
Además sen β = y cosα = , luego senβ = cosα o bien sen � − α� = cosα (b)
c c 2

π
Con la identidad sen φ = cos � − φ� en mente y definiendo el ángulo «φ = α − β» se tiene:
2

π π
sen φ = cos � − φ� implica sen(α − β) = cos � − (α − β)� «sustituyendo φ en los lados de la igualdad»
2 2

𝜋𝜋
sen(α − β) = cos �� − α� + β� «reescribiendo el argumento de cosen en 2do miembro»
2

π π
= cos � − α� cos β − sen � − α� senβ (∗) «aplicando la identidad (3)»
2 2

Luego, sen(α − β) = senα cos β − cos α sen β (8) «sustituyendo (a) y (b) en (∗) anterior»

Ahora al tener en cuenta que sen(α + β) = sen[α − (−β)] y aplicar (8) resulta:

sen(α + β) = sen[α − (−β)] «el producto de números con igual signo es positivo»

= senα cos(−β) − cos α sen(−β) (∗) «aplicando el 2do miembro de cos(α − β) dado en (3)»

Luego, sen(α + β) = sen α cos β + cosα senβ (9) «aplicando (1) y (2) en (∗) anterior»

(iii) Tangente y cotangente de la suma y de la diferencia de dos ángulos


sen(α + β)
tg(α + β) = «aplicando definición de tangente»
cos(α + β)

CLEMENTE MORENO (MATERIAL DIDÁCTICO EN VALIDACIÓN) 43


senα cosβ + cosα senβ
= «aplicando la identidad (4) y (6)»
cosαcosβ − senα senβ

senα cosβ + cosαsenβ


cosα cosβ
= cosα cosβ − senα senβ «dividiendo numerador y denominador entre cos α cos β »
cosα cosβ

senα cosβ cosα senβ


+
cosα cosβ cosα cosβ
= cosα cosβ senα senβ «distribuyendo el denominador común para cada termino del numerador»

cosα cosβ cosα cosβ

tgα + tgβ
Luego, tg(α + β) = (10) «simplificando y aplicando definición de tangente»
1 − tgα tgβ

tgα − tgβ
De manera similar, se prueba que tg(α − β) = (11) «tangente de la diferencia de ángulos»
1 + tgα tgβ

cos(α + β)
ctg(α + β) = «aplicando definición de cotangente»
sen(α + β)

cos α cos β − sen α sen β


= «aplicando la identidad (4) y (6)»
senα cos β + cos α sen β

cos α cos β − sen α sen β


cos α cos β
= sen α cos β + cos α sen β «dividiendo numerador y denominador entre cos α cos β »
cos α cos β

cos α cos β sen α sen β


cos α cos β
− cos α cos β
= sen α cos β cos α sen β «distribuyendo el denominador común para cada termino del numerador»
cos α cos β
+ cos α cos β

1 − tg α tg β
Luego, ctg(α + β) = (12) «simplificando y aplicando definición de cotangente»
tg α + tgβ

1+ tgα tgβ
Similarmente, se prueba que ctg(α − β) = (13) «cotangente de la diferencia de ángulos»
tgα − tgβ

Ejercicio N° 6
Encuentre, mediante la aplicación de las identidades adecuadas y sin utilizar calculadora, los
valores exactos de las siguientes expresiones

1) 5
sen � π�; (2) sec 75°; (3) cos � 𝜋𝜋 �; (4) sen � 7 π�;
12 12 12

π π π 7 π π
«Sugerencia: =� − � y π = � + �»
12 4 6 12 3 4

5) sen22° cos 38° + cos 22°sen38°; (6) cos 74° cos 44° + sen74°sen44°;

CLEMENTE MORENO (MATERIAL DIDÁCTICO EN VALIDACIÓN) 44


7) tg 110° − tg 50° (8) tg 27°+tg 18°
; ;
1 + tg 110° ∙ tg 50° 1−tg 27°∙tg 18°

Aplicando las últimas diez identidades, más las tres igualdades deducidas en la página 9,
demuestre las siguientes identidades

9) π
cos � − α� = senα; (10) sen �π − α� = cosα; (11) tg �π − α� = ctgα;
2 2 2

12) π
cos � + α� = −senα; (13) sen �π + α� = cosα; (14) tg �π + α� = −ctgα;
2 2 2

15) cos(π − α) = −cosα; (16) sen(π − α) = senα; (17) tg(π − α) = −tgα;

18) cos(π + α) = −cosα; (19) sen(π + α) = −senα; (20) tg(π + α) = tgα;

21) 3
cos � π − α� = −senα; (22) sen �3π − α� = −cosα; (23) 3
tg � π − α� = ctgα;
2 2 2

24) 3
cos � π + α� = senα; (25) sen �3π + α� = −cosα (26) tg �3 π − α� = −ctgα;
2 2 2

27) cos(x + 2kπ) = cosx; (28) sen(x + 2kπ = senx; (29) tg(x + kπ) = tgπ;

Nota. En (19), (20) y (21), el número k ∈ ℤ y mueve el período de estas funciones en ℝ

Simplifique cada una de las siguientes expresiones trigonométricas


π π
30) sen(90° + α) ∙ sen � − α� − cos � + α� ∙ cos(180° − α);
2 2

π π − 2α
31) cos α ∙ sen � + α� + sen α ∙ cos(90° + α) − 2sen(π + α) cos � �
2 2

π−2α
32) sen(3π − α) ∙ cos � � − sen α ∙ sen(3π + α);
2
7π + 2α π−2α 5π + 2α
33) sen � � − sen � � − sen(α + 180°) + sen � �;
2 2 2
3π − 2α 3π 3π − 2α
34) tg � � tgα + cos � + α� csc � � tgα;
2 2 2
3π − 2α 3π+2α 3 3π − 2α
35) cos � � + sen � � + ctg � π + α� + ctg � �;
2 2 2 2

36) cos(−α) tg α + sen(2π − α) − tg(−α) cos(2π − α) + ctg(−α)senα;

Demuestre cada una de las identidades siguientes


π + 4x 3π + 4x 1
37) sen � � ∙ sen � � = (cos2 x − sen2 x);
4 4 2
7 π
38) cos � π + x� cos � + x� = sen x ∙ cos x − √3;
6 3

CLEMENTE MORENO (MATERIAL DIDÁCTICO EN VALIDACIÓN) 45


39) (senx − sen y)2 + (cosx + cosy)2 − 2cos(x + y) = 2;
ctg2 x − 1
40) ctg 2x =
2 ctg x
2 tgx
41) tg 2x =
1− tg2 x

sen (x − y)
34) tg x − tg y =
cosx ∙ cosy

Fórmulas de reducción de ángulos al primer cuadrante


Las demostraciones previstas en el ejercicio anterior «Ejercicio N° 6» proporcionan las
identidades que permiten llevar «reducir» al primer cuadrante aquellas funciones trigonometría
de ángulos mayores a 90°. Es decir, que para cualquier ángulo mayor a 90° se puede hallar un
ángulo agudo positivo cuyas funciones son iguales en valor absoluto a las del ángulo dado

(i) Reducción del segundo al primer cuadrante


Para llevar funciones con ángulos en el segundo al primer cuadrante, se coloca el ángulo en
función de su suplemento, seguido de la aplicación de las identidades

(a) cos(π − α) = −cosα; (b) sen(π − α) = senα; (c) tg(π − α) = −tgα;

2 5𝜋𝜋
Ejemplo. Al reducir las funciones cos 135°, sen � π� y tg � � se tiene:
3 6

a) 135° = 180° − 45° «como 180° − x = 135° implica 180° − 135° = x entonces x = 45° es el ángulo reducido al I cuadrante»

cos 135° = cos(180° − 45°) «aplicando la función coseno en ambos lados de la igualdad»

Y, cos(180° − 45°) = − cos 45° «aplicando la identidad dada en (a) o fórmula de reducción»

√2
Luego, cos 135° = − «sustitución de cos 45° por su valor»
2

2 π π
b) Como π=π− entonces x = π 2
« es la reducción π en el primer cuadrante»
3 3 3 3 3

2 π
Entonces sen � π� = sen �π − � «aplicando la función seno en ambos lados de la igualdad»
3 3

π π
Y, sen �π − � = sen � � «aplicando la identidad dada en (b) o fórmula de reducción»
3 3

2 √3
Luego, sen � π� = 2
«sustitución de sen � π� por su valor»
3 2 3

CLEMENTE MORENO (MATERIAL DIDÁCTICO EN VALIDACIÓN) 46


5 π 5 π
c) π=π− entonces sen � π� = sen �π − � «transformando el ángulo y tomando seno en ambos lados»
6 6 6 6

π π 5 1
Y, sen �π − � = sen � � luego sen � π� = «aplicando (b) o fórmula de reducción y evaluando»
6 6 6 2

5 π
Por su parte, cos � π� = cos �π − � «tomando seno en ambos lados de la transformación del ángulo»
6 6

π 𝜋𝜋 5 √3
Así, cos �π − � = − cos � � luego cos � π� = «aplicando (b) o fórmula de reducción y evaluando»
6 6 6 2
1
5𝜋𝜋 5𝜋𝜋 1 5𝜋𝜋 √3
Y, tg � � = 2
√3
así tg � � = o bien tg � � = «definición de tangente, operando y racionalizando»
6 6 √3 6 3
2

(ii) Reducción del tercero al primer cuadrante

Para llevar funciones con ángulos en el tercero al primer cuadrante, se agrega «π» al ángulo
dado y se aplican las identidades

(d) cos(π + α) = −cosα; (e) 𝑠𝑠en(π + α) = −senα; (f) tg(π + α) = tgα;

7
Ejemplo. Al reducir las funciones cos 210°, sen � π� y tg 210° al primer cuadrante, se tiene:
6

a) 210° = 180° + 30° implica cos 210° = cos(180° + 30°) «expresión de la función para reducción»

√3
Y, cos(180° + 30°) = −cos 30° o bien cos 210° = «aplicación de fórmula de reducción y evaluación»
2

7 π 7 π
b) π=π+ implica sen � π� = sen �π + � «expresión de la función para reducción»
6 6 6 6

π π 7 1
Y, sen �π + � = −sen � � o bien sen � π� = − «aplicación de fórmula de reducción y evaluación»
6 6 6 2

c) 210° = 180° + 30° implica tg 210° = tg(180° + 30°) «expresión de la función para reducción»

√3
Y, tg(180° + 30°) = tg30° o bien tg 210° = «aplicación de fórmula de reducción y evaluación»
3

(iii) Reducción del cuarto al primer cuadrante


Para llevar funciones con ángulos en el cuarto al primer cuadrante se resta al ángulo un vuelta
competa y se aplican las fórmulas

(g) cos(−α) = cos α; (h) sen(−α) = −sen α;

CLEMENTE MORENO (MATERIAL DIDÁCTICO EN VALIDACIÓN) 47


11
Ejemplo. Al reducir las funciones cos 330°, sen � π� y tg 330° al primer cuadrante, se tiene:
6

a) − 30° = 330° − 360° implica cos(−30°) = cos(330° − 360°) «expresión de la función para reducción»

√3
Y, cos(−30°) = cos(30°) entonces cos 330° = «aplicación de fórmula de reducción y evaluación»
2

π 11 π 11
b) − = π − 2π implica sen �− � = sen � π − 2π� «expresión de la función para reducción»
6 6 6 6

π 𝜋𝜋 11 1
Y, sen �− � = −sen � � entonces sen � π� = − «aplicación de fórmula de reducción y evaluación»
6 6 6 2

c) −30° = 330° − 360° implica t(−30°) = tg(330° − 360°) «expresión de la función para reducción»

√3
Y, t(−30°) = −tg(30°) entonces tg 330° = − «aplicación de fórmula de reducción y evaluación»
3

(iv) Reducción de ángulos superiores a un giro

Si el ángulo «α» está dado en grados, «α» se divide entre 360, se descarta su cociente debido
a que corresponde al número de vueltas completas y se toma el resto como valor equivalente
del ángulo «α» dado

Ejemplo. Reducir al primer cuadrante sen405°, cos3720° y tg530° se tiene:


√2
405 360 sen405° = sen45° así sen405° = «el resto se tomó como ángulo dado y se calculó»
1 2
45

cos 3720° = cos 120° «el resto se tomó como ángulo dado»
3720 360
120 10
Así, cos 120° = cos(180° − 60°) «expresión de la función para reducción»

1
Y, cos(180° − 60°) = −cos60° o bien cos 3720° = − «aplicación de fórmula de reducción y evaluación»
2

360
tg 530° = tg 170° «el resto se tomó como ángulo dado»
530
170 1
Así, tg170° = tg(180° − 10°) «expresión de la función para reducción»

Y, tg(180° − 10°) = −tg10° o bien tg 530° ≈ −0,176 «aplicación de fórmula de reducción y evaluación»

Si el ángulo «α» está «π» radianes, se divide el numerador entre el denominador pero de tal
forma que el cociente sea un número par «el mayor posible», y se toma como ángulo la
fracción de «π» cuyo numerador es el residuo y denominador el divisor

CLEMENTE MORENO (MATERIAL DIDÁCTICO EN VALIDACIÓN) 48


89π 79𝜋𝜋
Ejemplo. Al reducir al primer cuadrante sen � � y cos � � se tiene:
12 8
89π 17π
89 12 sen � � = sen � � «el cociente entre el resto y el divisor se tomó como ángulo dado»
12 12
17 6 17π 5π
sen � � = sen �π + � «expresión de la función para reducción»
12 12

5π 5π 89π
Y, sen �π + � = −sen � � o bien sen ≈ −0,9659 «aplicación de fórmula de reducción y evaluación»
12 12 12

79π 15π
8 cos � � = cos � � «el cociente entre el resto y el divisor se tomó como ángulo dado»
79 8 8
15 8 15π 15π
cos � � = cos � − 2π� «expresión de la función para reducción»
8 8

15π 𝜋𝜋 𝜋𝜋 79π
Y, cos � − 2π� = cos �− � = cos � � así cos � � ≈ 0,9239 «aplicación de fórmula de reducción y evaluación»
8 8 8 8

COMENTARIO

El cociente debe ser un número par para poder desechar las vueltas completas del
85π 13
ángulo, así, al transformar en 6π + π se puede desechar 6π, que corresponde a tres
12 12
85π π
vueltas completas «3 ∙ 2π»; en cambio, si se transforma en 7π + no se puede desechar
12 12

7π debido a que 7π corresponde a tres vueltas y media y, al desecharlas, las funciones del
ángulo cambian

Ejercicio N° 7
Reducir al primer cuadrante las siguientes expresiones y dar su valor en fracciones si se trata
de ángulos notables y en decimales en otro caso
1) 𝑠𝑠en 120°; (2) cos � π�;
2 (3) tg 150°; (4) 3
sec � π�;
3 4

5) sen139°54′ ; (6) cos 144°23′; (7) tg � π�;


7 (8) 9
csc � 𝜋𝜋�;
10 14

7 (10) csc 225°; (11) tg �4 π�; (12) cos �5 π�;


9) ctg � π�;
6 3 4

CLEMENTE MORENO (MATERIAL DIDÁCTICO EN VALIDACIÓN) 49


6 (14) csc �11 𝜋𝜋�; (15) ctg �7 π�; (16) tg �17 π�;
13) sec � π�;
5 6 4 12

17) tg 2880° (18) csc �23 π�; (19) sec �126π�; (20) ctg �29π�;
4 7 4

Sin utilizar calculadora, halle el valor de las siguientes expresiones trigonométricas


π π 2 2 π 1 π 3 3
21) 4sen2 � � + �2 cos � �� − tg 2 � � − ctg 3 � � + �3 csc � π�� ;
3 6 3 4 3 3 4
2
3 9π 2 17π 5 5π 7 1 7
22) sen2 � � + cos 2 � � + tg 3 � � − sec 2 (5π) + � ctg � �� ;
4 4 3 4 6 4 12 2 6
4
23) Sabiendo que tgα = y tgβ = 7, verifique que tg(α + β) = −1
3
±√3 �√5 + 1�
24) Sabiendo que ctgα = √3 y ctgβ = √15 verifique que sen(α + β) =
8

Identidades del ángulo doble y del ángulo medio


(i) Coseno, seno y tangente del ángulo doble
Si en la identidad de la adición para el coseno que es cos(α + β) = cos α cos β − sen α sen β,
se remplaza «β» por «α», esto es se hace α = β se tiene:
cos(α + α) = cos α cos α − sen α sen α o bien cos 2α = cos 2 α − sen2 α (a); además por la
identidad cos2 α + sen2 α = 1 resulta cos2 α = 1 − sen2 α y sen2 α = 1 − cos 2 α, luego
cos 2α = cos2 α − sen2 α implica cos 2α = 1 − sen2 α − sen2 α o bien cos 2α = 1 − 2sen2 α (b);
y cos 2α = cos 2 α − sen2 α implica cos 2α = cos2 α − (1 − cos 2 α) así cos 2α = 2cos2 α − 1 (c);
Luego, cos 2α = cos 2 α − sen2 α = 1 − 2sen2 α = 2cos2 α − 1 (14) «igualando (a), (b) y (c)»

Asimismo, en la identidad de la adición para el seno, sen(α + β) = sen α cos β + sen β cos α,
si se remplaza «β» por «α», esto es se hace α = β se tiene:
sen(α + α) = sen α cos α + sen α cos α o bien sen 2α = 2senα cosα (15)

tgα + tgβ
Del igual modo, en la identidad de la adición para la tangente, tg(α + β) = si se
1 − tgα tgβ

remplaza «β» por «α», esto es se hace α = β se tiene:


tgα + tgα 2 tgα 2senα cos α
tg(α + α) = o bien tg 2α = (a). También tg 2α = implica
1 − tgα tgα 1 − tg2 α cos2 α−sen2 α

CLEMENTE MORENO (MATERIAL DIDÁCTICO EN VALIDACIÓN) 50


2 sen α cos α 2 cos α
sen2 α 2 ctgα
tg 2α = cos2 α − sen2 α
así tg 2α = sen α
cos2 α sen2 α
o bien tg 2α = (b). Asimismo,
− ctg2 α − 1
sen2 α sen2 α sen2 α
2 sen α cos α
2senα cos α 2
tg 2α = implica tg 2α = sen α cos α
cos2 α − sen2 α
o bien tg 2α = (c).
cos2 α−sen2 α ctg α−tg α
sen α cos α

2 tg α 2 ctg α 2
Luego, tg 2α = = = (16) «igualando (a), (b) y (c)»
1 − tg2 α ctg2 α − 1 ctg α − tg α

3
Ejemplo. Al hallar sen 2α, sabiendo que cos 𝛼𝛼 = con α ∈ IV cuadrante, se tiene:
5

De sen2 α + cos2 α = 1 resulta sen α = √1 − cos 2 α «se aisló sen α de identidad fundamental»

3 2 4
Así, sen α = �1 − � � o bien sen α = − «se efectuaron operaciones y tomo signo de seno en el IV cuadrante»
5 5
3 4 24
Luego, sen 2α = 2 � � �− � o bien sen 2α = − «sustituyendo los valores en identidad (15) y operando»
5 5 25

(ii) Coseno, seno y tangente del ángulo mitad


Las identidades correspondientes a la mitad de un ángulo se derivan de las igualdades del
β
ángulo doble, cuando éste se divide a la mitad. Así, cuando α = se tiene que:
2
β β
cos 2α = 1 − 2sen2 α implica cos 2 � � = 1 − 2sen2 � � «sustituyendo α por »
β
2 2 2

β 1−cos β β 1−cos β
Así, sen2 � � = o bien sen � � = ± � (17) «despejando y tomando raíz cuadrada»
2 2 2 2

β β
Y, cos 2α = 2cos2 α − 1 implica cos 2 � � = 2 cos2 � � − 1 «sustituyendo α por »
β
2 2 2

β 1 + cos β β 1 + cos β
Así, cos2 � � = o bien cos � � = ±� (18) «despejando y tomando raíz cuadrada»
2 2 2 2

La identidad para la tangente del ángulo mitad deriva del cociente de las identidades (17) y
(18), cuya igualdad resultante puede escribirse en otras formas, tal como se indica

β 1 − cos β
β sen � 2 � β ±� 2 β �1−cos β
tg � � = β así tg � � = o bien tg � � = ± (a); «sustituyendo y operando»
2 cos� 2 � 2 1 + cos β 2 �1+cos β
±� 2

β (1−cos β) (1+cos β) β 1− cos2 β


Y, �tg � �� = ±� ∙ así �tg � �� = ±�(1+cos «el su radical se multiplico por 1 =
(1+cos 𝛽𝛽)
»
2 (1+cos β) (1+cos β) 2 β)2 (1+𝑐𝑐𝑐𝑐𝑐𝑐𝑐𝑐)

CLEMENTE MORENO (MATERIAL DIDÁCTICO EN VALIDACIÓN) 51


β sen β 2 β sen β
Luego �tg � �� = ±�� � o bien �tg � �� = ± (b); «transformando y tomando raíz cuadrada»
2 1+cos β 2 1 + cos β

𝛽𝛽 sen β 1−cos β
Además, tg � � = ± � �� �
2 1 + cos β 1−cos β

β sen β(1−cos β) β 1−cos β


Implica tg � � = ± o bien tg � � = (c); «operando y simplificando»
2 sen2 β 2 sen β

β �1−cos β sen β 1−cos β


Así, tg � � = ± = = (19) «igualando (a), (b) y (c)»
2 �1+cos β 1+cos β sen β

Ejemplo 1. Al hallar el valor exacto de tg 165° sin utilizar calculadora, se tiene:

330° 1−cos330° √3
sen � �=� y cos(360° − 330°) = cos(−30°) = «aplicando (17) y formula de reducción»
2 2 2

√3
1− �2 − √3
Así, sen 165° = � 2
o bien sen 165° = «sustituyendo y operando»
2 2

330° 1+cos 330° √3


Y, cos � � = −� con cos(360° − 330°) = cos(30°) = «aplicando (18) y formula de reducción»
2 2 2

√3
1+ �2+√3
Así, cos 165° = −� 2 o bien cos 165° = − «sustituyendo y operando»
2 2

�2 − √3
�2 − √3
Luego tg 165° = 2
�2 + √3
o bien tg 165° = − «aplicando (17) y operando»
− 2 �2 + √3

α+β 1
Ejemplo 2. Al hallar cos � � y sen (α + β), siendo α y β son ángulos agudos y sabiendo que
2 2
8 12
cosβ = y senα = se tiene:
17 13

12 2 5
cos α = �1 − � � o bien cosα = «despejando cos α en identidad (1), sustituyendo y operando»
13 13

8 2 15
sen β = �1 − � � o bien senβ = «despejando sen α en identidad (1), sustituyendo y operando»
17 17

α+β 1 + cos(α+β) α+β 1+(cos α∙cos β−sen α∙senβ)


Y, cos � �=� o bien cos � �=� «por identidades (7) y (18)»
2 2 2 2

5 8 12 15 221+40−180
α+β 1 + �13 ∙ 17 − 13 ∙ 17�
Así, cos � �=� =� 221
«sustituyendo y operando»
2 2 2

CLEMENTE MORENO (MATERIAL DIDÁCTICO EN VALIDACIÓN) 52


α+β 81 α+β 9√442
Luego, cos � �=� o bien cos � �= «operando y racionalizando»
2 442 2 442

1 1−cos(α + β) α+ β 1−(cos α∙cos β−senα∙senβ)


Y, sen2 (α + β) = o bien sen2 � �= «por identidades (7) y (17)»
2 2 2 2
5 8 12 15 140
1 1−�13 ∙ 17 − 13 ∙ 17� 1−( − 221)
Así, sen2 (α + β) = = «sustituyendo y operando»
2 2 2

α+β 19 2 α+β 19√442


Luego, sen2 � �=� � o bien sen2 � �= «operando y racionalizando»
2 √442 2 442

Identidades con productos


Cuando se suman miembro a miembro las identidades (7) y (6) «adición y sustracción»
del coseno se obtiene

cos(α + β) = cosα cosβ − senα senβ


1
cos(α − β) = cosα cosβ + senα senβ así, cos α cos β = [cos(α + β) + cos(α − β)]; (20)
2
cos(α + β) + cos(α − β) = 2cosα cosβ

Si en lugar de sumar se resta miembro a miembro de las identidades (7) y (6) se obtiene

cos(α + β) = cosα cosβ − senα senβ


1
−cos(α − β) = −cosα cosβ − senα senβ o bien sen α sen β = [cos(α − β) − cos(α + β)]; (21)
2
cos(α + β) − cos(α − β) = −2senα senβ

Si ahora se considera la suma miembro a miembro de las identidades (9) y (8) «adición y
sustracción» del seno se obtiene

sen(α + β) = senα cosβ + cosα senβ


1
sen(α − β) = senα cosβ − cosα senβ así, senα cos β = [sen(α + β) + sen(α − β)]; (22)
2
sen(α + β) + sen(α − β) = 2senα cosβ

Ahora, al considerar la resta miembro a miembro de las identidades (7) y (6) en lugar de
la suma, se obtiene

sen(α + β) = senα cosβ + cosα senβ


1
− sen(α − β) = −senα cosβ + cosα senβ o bien, cos α sen β = [sen(α + β) − sen(α − β)]; (23)
2
sen(α + β) − sen(α − β) = 2cosα senβ

CLEMENTE MORENO (MATERIAL DIDÁCTICO EN VALIDACIÓN) 53


Ejemplo 1. Al escribir el producto cos 4α ∙ sen 3α como suma o diferencia resulta:
1
cos 4α ∙ sen 3β = [sen(4α + 3α) − sen(4α − 3α)] «aplicando identidad (23)»
2
1 1
Luego, cos 4α ∙ sen 3β = sen 7α − sen α «sumando y restando los ángulos»
2 2

Ejemplo 2. Evaluar con exactitud el producto de sen 105° ∙ sen15°


1
sen 105° ∙ sen15° = [cos(105° − 15°) − cos(105° + 15°)] «aplicando identidad (21)»
2

1 1
= cos 90° − cos 120° «sumando y restando los ángulos»
2 2
1 1
= 0 − ∙ �− � «evaluando el coseno de los ángulos»
2 2

1
Luego, sen 105° ∙ sen15° = «valor exacto del producto solicitado»
4

Identidades con factores


Las identidades (20), (21), (22) y (23) se transforman en identidades con factores, qu
suelen ser útiles para expresar sumas y diferencias que contienen senos y cosenos en la forma
de producto de dichas expresiones. Para ello, se hace α + β = x y α − β = y identidades que
α+β=x x+y x+y x−y
forman el sistema � donde α = y β=x− o bien β = , luego
α−β=y 2 2 2

1 x+y x−y
cos α cos β = [cos(α + β) + cos(α − β)] implica cos y − cos x = 2 cos � � cos � � ; (24)
2 2 2
1 x+y x−y
sen α sen β = [cos(α − β) − cos(α + β)] implica cos x − cos y = 2sen � � sen � �; (25)
2 2 2
1 x + y x−y
senα cos β = [sen(α + β) + sen(α − β)] implica sen x + sen y = 2sen � � cos � �; (26)
2 2 2
1 x+y x−y
cos α sen β = [sen(α + β) − sen(α − β)] implica sen x − sen y = 2 cos � � sen � �; (28)
2 2 2

Ejemplo 1. Al escribir como producto la diferencia 𝑠𝑠en 7φ − sen 5φ se obtiene


7φ + 5φ 7φ − 5φ
sen 7φ − sen 5φ = 2 cos � � sen � � «aplicando la identidad (28)»
2 2

Luego, sen 7φ − sen 5φ = 2 cos 6φ ∙ senφ «efectuando operaciones entre ángulos»

Ejemplo 2. Al calcular el valor exacto de sen105° + sen15° se obtiene:

CLEMENTE MORENO (MATERIAL DIDÁCTICO EN VALIDACIÓN) 54


105° + 15° 105° − 15°
sen105° + sen15° = 2sen � � ∙ cos � � «aplicando la identidad (26)»
2 2

= 2 sen60° ∙ cos45° «efectuando operaciones entre ángulos»

√3 √2
=2∙ ∙ «evaluando el seno y coseno de los ángulos»
2 2

√6
Luego, sen105° + sen15° = «simplificando y operando»
2

COMENTARIO

Además de lo ilustrado en los ejemplos anteriores, las 28 identidades demostradas


hasta el momento, se aplican tanto en la justificación de simplificación de expresiones
trigonométricas como en la demostración de identidades o solución de ecuaciones en este
contexto. Por ello conviene tenerlas presentes «quizás en una ficha» cuando se realice alguna
de estas actividades

sen 4x
Ejemplo 1. Al simplificar la expresión resulta
4 cos 2x ∙ cos x

sen 4x sen 2(2x)


= «rescribiendo el numerador en términos del ángulo doble»
4 cos 2x ∙ cos x 4 cos 2x ∙cosx

2 sen 2x ∙ cos2x
= «desarrollando el seno del ángulo doble del numerador»
4 cos 2x ∙ cosx
sen 2x
= «se simplifica por cos 2x, dado que
cos 2x
= 1»
2 cos x cos 2x

2 senx ∙ cos x
= «desarrollando el seno del ángulo doble del numerador»
2 cos x

= sen x «se simplifica por cos 𝑥𝑥, debido a que


cos 𝑥𝑥
cos 𝑥𝑥
= 1»

sen 4x
Luego, = sen x «es la simplificación de la expresión»
4 cos 2x ∙ cos x

tg x
Ejemplo 2. Al simplificar la expresión resulta:
tg 2x − tgx

CLEMENTE MORENO (MATERIAL DIDÁCTICO EN VALIDACIÓN) 55


tg x tg x
= 2 «por (16), en el denominador se sustituyo tg 2x por
2
»
tg 2x − tgx − tgx ctgx − tgx
ctgx−tgx

tgx
= 2−tgx ∙ ctgx + tg2 x
«se efectuó la resta de fracciones en el denominador y ejecutó operaciones»
ctgx − tgx

tgx ∙ ctgx − tg2 x


= «se reescribió la fracción compleja y ejecutó operaciones»
2−1+tg2 x

1 − tg2 x
= «en el numerador tgx ∙ ctg x = 1 y ejecutó resta en el denominador»
1 + tg2 x

sen2 x
1−
cos2 x
= sen2 x
« tgx se reescribió en términos de seno y coseno»
1+
cos2 x

cos2 x − sen2 x
cos2 x
= cos2 x + sen2 x
«se efectuó la resta del numerador y la suma del denominador»
cos2 x

= cos 2 x − sen2 x «se reescribió la fracción compleja y ejecutó operaciones»

tg x
Luego, = cos 2x «aplicando (14), cos2 x − sen2 x se sustituyó por cos 2x»
tg 2x − tgx

cos x cos x
Ejemplo 3. Para demostrar que + = 2tg x, se tiene que:
csc x+1 csc x−1

cos x cos x cos x cos x


+ = 1 + 1 «csc x =
1
se sustituyó en el denominador»
csc x+1 csc x−1 +1 −1 senx
sen x sen x

cos x cos x
= 1 + sen x + 1 − sen x «se efectuó la suma y la resta del denominador»
senx sen x

sen x ∙ cos x sen x ∙ cos x


= + «se reescribió la fracción compleja y ejecutó operaciones»
1 + sen x 1 − senx

senx ∙ cos x (1 − sen x) + sen x ∙ cos x (1 + sen x)


= «se efectuó la suma de fracciones»
(1+sen x)(1−sen x)

sen x ∙ cos x (1 − senx + 1 + senx)


= «en el numerador se tomó a senx ∙ cos x como factor común»
1− sen2 x
sen x ∙ cos x ∙ 2
= «se efectuó la suma en el numerador»
cos2 x

cos x cos x
Luego, + = 2tg x «pues
cos 𝑥𝑥
= 1 para x ≠
π 3
y x ≠ π»
csc x+1 csc x−1 cos 𝑥𝑥 2 2

CLEMENTE MORENO (MATERIAL DIDÁCTICO EN VALIDACIÓN) 56


Ejercicio N° 8
A partir de los datos suministrados en cada caso, calcule el valor de las expresiones dadas en
los siguientes casos
5 4
1) cos2α, senα = y α ∈ I; (2) sen2α, cosα = − y 𝛼𝛼 ∈ II;
7 5

3) csc2φ, senφ =
√3
y φ ∈ I; (4) sec2θ, senθ =
√3
y θ ∈ III;
2 4

5) tg 2β, cos β = −
√2
y β ∈ II; (6) ctg 2ϑ, sen ϑ = −
√5
y ϑ ∈ III;
3 7

3 4 π
7) sen2x, cos x = y 0° < x < 90°; (8) cos 2x, sen x = y < x < π;
4 5 2

9) 5 3 (10) ctg 2x, 4 3


tg 2x, tg x = y π < x < π; cos x = y π < x < 2π;
7 2 5 2

11) cos � �,
φ
cosφ =
9
y φ ∈ IV; (12) sen �ϑ�, senϑ = −
7
y ϑ ∈ III;
2 13 2 12

13) x
cos � �, 𝑠𝑠en x =
√3
0° < x < 90° (14) x
tg � �, cos x = −
1
y π < x < π;
3
2 5 2 4 2

15) 𝑥𝑥
ctg � �; cosx = −
3
y 90° < x < 270° (16) csc � �;
x
senx =
7
y
π
< x < π;
2 5 2 25 2

17) 1 4 5 (18) φ−ϑ 3 7


sen (α + β), senα = y senβ = ; cos � �, cosφ = y cosϑ = ;
2 7 7 2 5 25

19) tg �
α+β
�, senα =
3
y senβ = ;
2 (20) ctg �δ − λ�, cos δ = 3 y cos λ = 7
;
2 5 5 2 5 25

Escriba cada producto en términos de suma o diferencia de senos y cosenos

21) cos(3A) ∙ cos �2 A�; (22) sen �2 B� ∙ cos �1 B�


3 3 3

23) cos 2 ϑ ∙ sen3ϑ (24) sen �2 A� ∙ sen �3 A�;


5 5

Escriba como producto cada suma o diferencia en términos de seno y coseno

25) sen 7ϑ + sen 5ϑ; (26) cos 3t + cos t;

27) sen 5m − cos 9m; (28) cos 7u − cos 3u;

CLEMENTE MORENO (MATERIAL DIDÁCTICO EN VALIDACIÓN) 57


Aplicando la identidad apropiada halle el valor exacto de cada una de las expresiones siguientes:

29) cos 75° ∙ sen 15°; (30) sen 195° ∙ cos 75°;

31) sen 105° ∙ sen 165°; (32) cos 15° ∙ cos 75°;

33) sen 105° + sen195°; (34) cos 195° + cos 285 °;

35) sen 75° − sen165°; (36) cos 15° − cos 105 °;

Simplifique al mínimo cada una de las expresiones siguientes

37) cos4 φ − sen4 φ − cos2φ (38) cos 𝜑𝜑 sen2φ − senφ(1 + cos2φ);


1 + ctg x csc x−sen x
39) ; (40) ;
sen x + cos x sec x−cos x

Demuestre cada una de las identidades siguientes

41) 2 cos 2φ + 1 3 − tg2 φ (42) 2 tg θ


= ; = sen 2θ;
2 cos 2φ − 1 1 − 3tg2 φ 1+ tg2 θ

cos x−cos 3x
43) = tg x; (44) cos x+cos y
= ctg �
x−y
�;
sen x+sen 3x sen x−sen y 2

Ecuaciones trigonométricas
En contraste con las identidades trigonométricas, igualdades que son verdaderas para
todo valor del ángulo, las ecuaciones trigonométricas son proposiciones que sólo se verifican

para ciertos valores del ángulo, dando pie a los valores que
satisfacen la ecuación. En la búsqueda de estas soluciones en el
intervalo [0, 2π] «o bien 0° ≤ 𝛼𝛼 ≤ 360°» se aplican las funciones π−α
x
α
trigonométricas inversas en el cálculo del primer ángulo y los π+α 2π − α

ángulos de las fórmulas de reducción al primer cuadrante en el


cálculo del segundo, tal como se indica en los ejemplos

Ejemplo 1. Al buscar el conjunto solución de la ecuación 2 sen x − 1 = 0, en [0, 2π] se tiene:


1
2 sen x − 1 = 0 implica 2 sen x = 1 o bien sen x = «se aisló la función sen x en un lado de la igualdad»
2

1 π
Así, sen−1 (sen x) = sen−1 � � implica x = 30° o bien x = π
«valor de α ∈ I para primera solución x = »
2 6 6

CLEMENTE MORENO (MATERIAL DIDÁCTICO EN VALIDACIÓN) 58


π 5
Y, π − implica x = π «es la 2da solución, pues sen x =
1
es positivo y sen x vuelve a ser positivo en II cuadrante»
6 6 2

π 5
Luego, S = � , π � «conjunto solución de la ecuación en el intervalo 0 ≤ x ≤ 2π»
6 6

Prueba:
π 1
Para x = resulta, 2 ∙ sen 30° − 1 = 2 � � − 1 = 0 «sustituyendo x por
π
= 30° y operando»
6 2 6

5 1
Para x = π resulta 2 ∙ sen 150° − 1 = 2 � � − 1 = 0 «sustituyendo x por
5
𝜋𝜋 = 150° y operando»
6 2 6

Ejemplo 2. Al buscar el conjunto solución de la ecuación 2 cos x + √2 = 0, en [0, 2π] se tiene:


√2
2 cos x + √2 = 0 implica 2 cos x = −√2 o bien cos x = − «se despejó la función cos x»
2

√2 3
Así, cos −1 (cos x) = cos −1 �− � o bien x = 135° o x = π «valor de α ∈ II para primera solución x = π»
3
2 4 4

3 π
Como, π − π = 3
«x = π está asociado un ángulo base de 45° o
π
en el I cuadrante»
4 4 4 4

π 5
Así, x = π + o bien x = π «valor de α ∈ III para segunda solución es x = π »
5
4 4 4

3 5
Luego, S = � π, π� «conjunto solución de la ecuación en el intervalo 0 ≤ x ≤ 2π»
4 4

Prueba:
√2
Para x = 135° resulta 2 cos 135° + √2 = 2 �− � + √2 = 0 «sustituyendo x por

= 135° y operando»
2 4

√2
Para 𝑥𝑥 = 225° resulta 2 cos 225° + √2 = 2 �− � + √2 = 0 «sustituyendo x por

= 225° y operando»
2 4

COMENTARIO

En atención al carácter periódico de las funciones seno, coseno y tangente, exhibido en


las gráficas dibujadas en párrafos anteriores, la solución general de:
sen−1 (a) + 2kπ con k ∈ ℤ (1)
(i) sen x = a para −1 ≤ a ≤ 1 es x = � −1
(2k + 1)π − sen (a) con k ∈ ℤ (2)
(ii) cos x = a para −1 ≤ a ≤ 1 es x = 2kπ ± cos −1 (a) con k ∈ ℤ
(iii) tg x = a para a ∈ ℝ es x = tg −1 a + kπ con k ∈ ℤ

CLEMENTE MORENO (MATERIAL DIDÁCTICO EN VALIDACIÓN) 59


1
Ejemplo 3. En el caso del ejemplo 1, luego de despejar sen x = , se tiene que:
2

1 π
x = sen−1 � � + 2 ∙ 0 ∙ π implica x = 30° o bien x = «aplicando (i) en rama (1), con k = 0»
2 6

1 π 5
Y, x = (2 ∙ 0 + 1)π − sen−1 � � implica x = π − o bien x = π «aplicando (i) en rama (2), con k = 0»
2 6 6

π 5
Luego, S = � , π � «conjunto solución de la ecuación dada»
6 6

√2
Ejemplo 4. En el caso del ejemplo 2, después de despejar cos x = − , se tiene
2

√2 3
x = cos −1 �− � implica x = 135° o bien x = π «primera solución en II cuadrante»
2 4

√2 3 5
Y, x = 2π − cos−1 �− � así x = 2π − π o bien x = π «aplicando (ii) a 2da solución»
2 4 4

3 5
Luego, S = � π, π� «conjunto solución de la ecuación dada»
4 4

Ejemplo 5. Al resolver la ecuación sen 2x = sen x para 0 ≤ x ≤ 2π, se tiene:

sen 2x = sen x implica 2senx cos x = sen x «aplicando identidad (15) en 1er lado de la igualdad»

2 sen x cos x − sen x = 0 «igualando a 0 la identidad dada»

sen x(2 cos x − 1) = 0 implica sen x = 0 o bien 2 cos x − 1 = 0 «los factores se igualan a 0»

sen−1 (0) + 2 ∙ 0 ∙ π
Así, sen x = 0 implica x = � o bien x = 0 y x = π «por (i), rama (1) y (2)»
(2 ∙ 0 + 1)π − sen−1 (0)
1
Y, 2 cos x − 1 = 0 implica cos x = «se aisló la función cos x en un lado de la igualdad»
2

1 π
Así, x = cos−1 � � implica x = 60° o bien x = «primera solución en I cuadrante»
2 3
1 π 5
Y, x = 2π − cos −1 � � así x = 2π − o bien x = π «aplicando (ii) a 2da solución»
2 3 3

π 5
Luego, S = �0, , π, π� «conjunto solución de la ecuación dada»
3 3

Ejemplo 6. Al resolver la ecuación cos 2x = 2(senx − 1) para 𝑥𝑥 ∈ ℝ y las inversas calculadas


con cuatro cifras significativas, se tiene:

cos 2x = 2(senx − 1) implica 1 − 2sen2 x = 2 senx − 2 «por (14) en 1er lado y distribución en 2do»

CLEMENTE MORENO (MATERIAL DIDÁCTICO EN VALIDACIÓN) 60


Así, 2 sen2 + 2 senx − 3 = 0 «igualando a 0 la identidad dada»

−2 ± �4 − 4(2)(−3) −2 ± √28
Y, senx = o bien senx = «se aplicó la resolvente, la ecuación es cuadrática»
2(2) 4

Así, sen x = −1,82288 y sen x = 0,82288 «cálculo de las raíces de la ecuación»

La raíz sen x = −1,82288 se descarta «−1,82288 ∉ [−1,1] que es el rango de la función»

2kπ + sen−1 (0,82288)


Y, sen x = 0,82288 implica x = � «por (i), rama (1) y (2)»
(2k + 1)π − sen1 (0,82288)
Luego, S = {2kπ + 0,9665; (2k + 1)π − 0,9665} «conjunto solución de la ecuación dada»

Resolución de triángulos oblicuángulos


En la parte central del capítulo se estudió la aplicación de las funciones trigonométricas
en la resolución de triángulos rectángulos. En lo que sigue, se aplica la trigonometría en la
resolución de triángulos oblicuángulos en general y para ello se estudia los teoremas del seno
y del coseno debido a que ellos permiten resolver cualquier tipo de triángulos

Teorema del seno


Considere el triángulo oblicuangulo ABC exhibido en la figura adjunta, y tracemos la altura

C «h» sobre el lado «c». De este modo el triangulo ∆ABC queda


γ dividido en los triangulos rectangulos ∆ACD y ∆BCD
b a
h h
En ∆BCD, sen β = así h = a ∙ sen β «definición de seno y despejando h»
a
α β
A B h
c D En ∆ACD, sen α = así h = b ∙ cos α «definición de coseno y despejando h»
b

h = a ∙ sen β y h = b ∙ cos α implica a ∙ senβ = b ∙ senα (∗) «antecedentes iguales, conseguentes iguales»

a ∙ senβ b ∙ senα a b
Y, = implica = (a) «ambos lados de (∗) se dividieron por sen α ∙ sen β»
senα ∙ senβ senα ∙ senβ sen α sen β

Si en lugar de haber trazado la altura sobre el lado «c», se traza sobre el lado «𝑏𝑏» y se
a c
desarrolla un proceso análogo, se obtiene = (b); y, al considerar (a) y (b), resulta:
sen α sen γ

a b c
= =
sen α sen β sen γ

CLEMENTE MORENO (MATERIAL DIDÁCTICO EN VALIDACIÓN) 61


Igualdades que expresan la ley de los senos e indican que en todo triángulo, los lados son
proporcionales a los senos de los ángulos opuestos

COMENTARIO

El teorema del seno se utiliza cuando se conoce un lado y dos de los ángulos del triángulo
o cuando se conoce el valor de dos lados y del ángulo opuesto a estos

Ejemplo 1. Al calcular la longitud de los lados «a» y «b» del triángulo de la figura, resulta:
38° + 69° + γ = 180° implica γ = 73° «los angulos internos suman 180°»
γ
b a a 53 53 sen 38°
= implica a = «ley de senos y despejando»
sen 38° sen 73° sen 73°
38° 69° 53 ∙ (0,61566)
Así, a = o bien a = 34,12cm «sustituyendo y operando»
c = 53cm 0,9563

b 53 53 ∙ sen 69°
= implica b = «aplicando ley de senos y despejando»
sen 69° sen 73° sen 73°
53 ∙ (0,93358)
Así, b = o bien b = 51,74cm «sustituyendo y operando»
0,9563

Ejemplo 2. Un aeroplano vuela exactamente sobre un camino recto que une dos pequeñas
ciudades que distan entre si 9,47km, los ángulos de depresión de ambas son 12,3° y 7,9°
respectivaente. Halle la distancia recta desde el aeroplano a cada una de las ciudades y la altura
a la que se encuentra el aeroplano
7,9° + 12,3° + γ = 180° así γ = 159,8° «los angulos internos suman 180°»
A
7,9° 12,3° b 9,47 9,47 ∙ sen 7,9°
= o bien b = «por ley de senos y despejando»
c sen 7,9° sen 159.8° sen 159,8°
b
h
9,47∙(0,1374)
7,9° 12,3° b= así b = 3,77 km «sustituyendo y operando»
0,3453
B 9,47 km C
c 9,47 9,47∙sen 12,3°
= asi b = «por ley de senos y despejando»
sen 12,3° sen 159,8° sen 159,8°

9,47 ∙ (0,213)
Luego b = o bien b = 5,84 km «sustituyendo y operando»
0,3453
h
Y, sen 12,3° = entonces h = b ∙ sen 12,3° «aplicando definción de seno y despejando»
b

Así, h = 5,84 ∙ (0,213) o bien h = 1,244 km «sustituyendo y operando»

CLEMENTE MORENO (MATERIAL DIDÁCTICO EN VALIDACIÓN) 62


Teorema del coseno
Considere el triángulo oblicuangulo ABC exhibido en la figura adjunta, y tracemos la altura
correspondiente al vértice «C», De este modo el triangulo ∆ABC queda dividido en los triangulos
rectángulos ∆CDB y ∆ADC sobre los cuales aplicaremos la igualdad pitagórica, es decir:
����)2 = (CD
(BC ����)2 + (DB
����)2 o bien a2 = h2 + (DB
����)2 (1) «por Pitagoras»
C
γ ����)2 = (CD
����)2 + (AD
����)2 o bien h2 = b2 − (AD
����)2
(AC «Pitagoras y despejando»
b a
h ����
AD + ���� DB = c − ����
DB = c implica ���� AD «suma de segmentos y despejando»
α β
A B Y, a2 = h2 + (DB ����)2 + (c − AD
����)2 implica a2 = b2 − (AD ����)2 «sustituy.»
c D

����)2 + c 2 − 2cAD
Así, a2 = b2 − (AD ���� + (AD
����)2 o bien a2 = b2 + c 2 − 2cAD
���� (2) «operando y simplificando»

�����
AD
En el ∆AD, cos α = o bien ����
AD = b ∙ cos α (3) «definición de coseno y despejando»
b

Luego, a2 = b2 + c 2 − 2bc ∙ cos α (∗) «sustituyendo (3) en (2) se llega a (∗) que corresponde a ley del coseno»

De manera similar, se prueba que b2 = a2 + c 2 − 2ac ∙ cos β y c 2 = a2 + b2 − 2ab ∙ cos γ,


indicando que en todo triángulo, el cuadrado de un lado es igual a la suma de los cuadrados de
los otros dos lados menos el doble producto de esos lados por el coseno del ángulo que se
forma entre esos lados

COMENTARIO

El enunciado del teorema indica que se utiliza cuando se conoce el valor de dos lados y
el ángulo que se forma entre ellos

Ejemplo 1. Al calcular la longitud del lado «c» del triángulo de la figura, resulta:

A α = 58°12′ implica α = 58,2° «transformación del ángulo»

c 2 = a2 + b2 − 2ab ∙ cos 58,2° «aplicando ley del coseno a lado requerido»


c
c 2 = (12,3)2 + (19,1)2 − 2(12,3)(19,1) ∙ (0,52696) «sustituyendo valores»
58°12′
B 12,3m C Luego, c 2 = 268,5m2 o bien c = 16,39m «operando»

CLEMENTE MORENO (MATERIAL DIDÁCTICO EN VALIDACIÓN) 63


Ejemplo 2. De la estación «A» dos lineas de ferrocarril parten formando un ángulo de 42°. Si
km km
desde «A» parten dos trenes con velocidades de 54 y 60 respectivamente, ¿a qué
h h

distancia estarán los trenes despúes de 30 minutos?


d
d1 v= implica d1 = t ∙ v1 y d2 = t ∙ v2 «distancia recorrida por trenes 1 y 2»
s t
km
A 15° d1 = (0,5h) ∙ 54 o bien d1 = 27km «distancia recorrida por el tren 1»
d2 h

km
d2 = (0,5h) ∙ 60 o bien d2 = 30 km «distancia recorrida por el tren 2»
h

s 2 = (27)2 + (30)2 − 2(27)(30) ∙ cos 15° o bien s = 8,01km «por ley de coseo,separación de trenes en 30m»

Ejercicio N° 9
Encuentre el conjunto solución de las ecuaciones trigonométricas en el intervalo indicado

1) 1 + cosx = 0, para 0 ≤ x ≤ 2π; (2) 1 + √2 senφ = 0 para 0° ≤ φ ≤ 360°;

3) 4 cos2 x − 3 = 0, para x ∈ [0, 2π]; (4) 1 − √2 cos ϑ = 0 para 0° ≤ ϑ ≤ 360°;

5) 1 − sen x = 0, para x ∈ [0, 2π]; (6) 2𝑠𝑠𝑠𝑠𝑠𝑠2 𝑥𝑥 − 1 = 0, para x ∈ [0, 2π];

7) sen2 α = senα, para 0° ≤ α ≤ 180°; (8) cos2 β = cos β, para β ∈ [0°, 180°];

9) 2senx cos x = cos x, para x ∈ [0, 2π]; (10) 2senx cos x = senx, para 0 ≤ x ≤ 2π;

11) 2sen2 x + 1 = 3 senx, para x ∈ [0, 2π]; (12) 2cos2 x = 1 − cos x, para 0 ≤ x ≤ 2π;

13) 4cos2 2x − 4 cos 2x + 1 = 0, x ∈ [0, π ]; (14) 2sen2 �x� − 3sen �x� + 1 = 0, x ∈ [0, 2π];
2 2

15) sen2 φ + 2 cos φ + 2 = 0, φ ∈ [0°, 360°]; (16) 2cos2 φ + 3senφ = 0, φ ∈ [0°, 360°];

Aplicando la ley de los senos y/o de los cosenos determine el valor los elementos desconocidos

17) γ
(18) γ
b a 10 a

60° β
42° 56°
20
32,3

19) γ
(20) γ
7,6 5,9 b 12

52°15′ β 42°55′ β
c 15

CLEMENTE MORENO (MATERIAL DIDÁCTICO EN VALIDACIÓN) 64


21) γ (22) γ
b a
0,7
69°32′ 56°28′ α 48°
1,17
0,9

23) Dado un triángulo ∆ABC, donde 𝛼𝛼 = 105°, b = √6 − √2 y c = √3 − 1, determine los otros


elementos: a, β y γ
24) En la parte suprior de un edificio que mide 34m de altura, está colocada una antena de
televisión. Desde un mismo punto de la calle se observa el punto superior de la antena con
un ángulo de elevación de 63,6° y su base con un ángulo de elevación de 58,3°. ¿Cuál es
la altura de la antena? Si se imaginan dos cuerdas inclinadas, una desde la base de la
antena al punto de observación, otra desde la cúspide de la antena al punto de observación,
¿cuál es la longitud de estas cuerdas?, ¿cuáles las medidas de los ángulos internos al
triángulo que se forma entre la vertical de la antena y las cuerdas al punto de observación?
25) Dos puntos de vigilancia, A y B dispuestos en línea recta y separados 12km a lo largo de
una costa para vigilar los embarques ilegales que arriben en un límite de 4km. Si el puesto
A reporta un barco S a un ángulo BAS = 38°27′ y el puesto B reporta el mismo barco a un
ángulo ABS = 19°48′, ¿a qué distancia estará el barco del punto A?, ¿a qué distancia del
punto B?
26) Dos vehiculos salen del mismo punto en el mismo instante y a la misma velocidad en
direcciones que forman un ángulo de 43°27′. Después de tres horas la distancia que los
separa es de 164,2 km. ¿Cuál es la velocidad con que se desplazan los carros?
27) Dos aviones vuelan uno detrás del otro, a la misma altura y separados por una distancia de
1,3 km. En un determinado instante se observa desde ellos exactamente hacia delante un
puente con ángulos de depresión de 42° y 44° respectivamente. Calcular la distancia que
separa el puente de cada uno de los aviones y la altura a la que vuelan éstos

CLEMENTE MORENO (MATERIAL DIDÁCTICO EN VALIDACIÓN) 65


República Bolivariana de Venezuela
Ministerio del Poder Popular para la Educación
Unidad Educativa: Colegio San Agustín – El Marqués
Material elaborado por el Prof. Clemente Moreno

NÚMEROS IMAGINARIOS

COMENTARIO

Es preciso recordar que en el conjunto de los números reales «ℝ», siempre fue posible
hallar solución a las ecuaciones lineales ax + b = 0, con «a» y «b» números reales y a ≠ 0,
pero no siempre se podía encontrar solución real a las ecuaciones del tipo ax 2 + bx + c = 0,
−b ± √b2 − 4ac
dado que en la resolvente x = si b2 − 4ac < 0 la raíz no era un número real.
2a

−2 ± �22 − 4(1)(2) −2 ± √−4


En particular, para x 2 + 2x + 2 = 0 se tiene x = o bien x = que
2∙1 2

evidentemente no es un número real. Esta deficiencia de «ℝ» conduce a la búsqueda de


un nuevo conjunto, donde sea posible encontrar la solución a este tipo de ecuaciones

La unidad imaginaria
La ecuación x 2 + 1 = 0 indica que x 2 = −1, pero el cuadrado de un número real es
siempre no negativo, por lo cual no existe un número real que satisfaga esta identidad. Para
remediar esta situación, los matemáticos incorporan la llamada unidad imaginaria, que
denotan con «i» y cuyo cuadrado es −1, es decir «i2 = −1 o bien i = √−1», dando origen a
los números imaginarios.
En los números imaginarios la ecuación x 2 + 1 = 0 tiene solución, debido a que:

x 2 = −1 implica x = √−1 entonces x = i «definición de unidad imaginaria»

Ejemplo 2. La ecuación 4x 2 + 25 = 0 también tiene solución en los números imaginarios


25
En efecto. 4x 2 + 25 = 0 implica x 2 = − «dejando a «x 2 » en el primer miembro de la igualdad»
4

25 25
Así, x = ±�− o bien x = ±� i2 «dejando a «x» en 1er miembro y sustituyendo −1 por i2 »
4 4

5
Luego, x = ± i «calculando la raíz cuadrada en el segundo miembro»
2

CLEMENTE MORENO (MATERIAL DIDÁCTICO EN VALIDADIÓN) 1


Ejemplo 3. En la ecuación x 2 + 2x + 2 = 0, dada en el comentario inicial, se tiene:

−2 ± �22 − 4(1)(2) −2 ± √−4


x= o bien x = «aplicando la resolvente y operando»
2∙1 2

−2 ± √4i2 −2 ± 2i
Así, x = o bien x = «sustituyendo −1 por i2 y calculando la raíz cuadrada»
2 2

Luego, x = −1 ± i o bien x1 = −1 + i y x2 = −1 − i «simplificando y separando soluciones»

Potencias de «𝐢𝐢»

i1 = i i5 = i4 ∙ i = 1. i = i i9 = i4 ∙ i4 ∙ i = 1 ∙ 1. i = i
i2 = −1 i6 = i4 ∙ i2 = 1 ∙ (−1) = −1 i10 = i4 ∙ i4 ∙ i2 = 1 ∙ 1 ∙ (−1) = −1
i3 = i2 ∙ i = (−1) ∙ i = −i i7 = i4 ∙ i3 = 1 ∙ (−i) = −i i11 = i4 ∙ i4 ∙ i3 = 1 ∙ 1 ∙ (−i) = −i
i4 = i2 ∙ i2 = (−1) ∙ (−1) = 1 i8 = i4 ∙ i4 = 1 ∙ 1 = 1 i12 = i4 ∙ i4 ∙ i4 = 1 ∙ 1 ∙ 1 = 1

La repetición de los valores de «i» de la primera a la cuarta potencia, indica que para
la potencia n > 4 de «i», basta dividir a «n» entre 4, el resto de esta división es el valor de la
potencia de «i».

Ejemplo 1. Al calcular el valor de i27 se tiene:

Dado que en 27 4 entonces i27 = i3 «el resto de la división es 3»


3 6
Entonces, i27 = i3 = −i «equivalencia entre potencias de «i»»

Ejemplo 2. Al calcular i235 + i29 resulta:

Como 235 ÷ 4 = 58 ∙ 4 + 3 entonces i235 = i3 «el resto de 235 ÷ 4 es 3»

Y, 29 ÷ 4 = 7 ∙ 4 + 1 entonces i29 = i1 «el resto de 29 ÷ 4 es 1»

Luego i235 + i29 = i3 + i1 «equivalencia entre potencias de «i»»

= −i + i «equivalencia entre potencias de «i»»

=0 «sumando términos semejantes»

En consecuencia, i235 + i29 = 0 «conclusión»

CLEMENTE MORENO (MATERIAL DIDÁCTICO EN VALIDADIÓN) 2


Ejemplo 3. ¿Qué número imaginario se corresponde con √−81 + 2√−36 − 3√−1 − 8i?

Como √−81 + 2√−36 − 3√−1 − 8i = √81i2 + 2√36i2 − 3√i2 − 8i «sustituyendo −1 por i2 »

= 9i + 12i − 3i − 8i «calculando raíz cuadrada y operando»

Luego √−81 + 2√−36 − 3√−1 − 8i = 10i «sumando términos semejantes»

NÚMEROS COMPLEJOS
Un número de la forma z = a + bi, donde «a» y «b» son números reales e «i» la unidad
imaginaria, se llama número complejo. A esta forma de expresar el número complejo se le
llama forma binomial o algebraica

COMENTARIO

Si z = a + bi es un número complejo, entonces:


a: corresponde a la parte real y se denota como Re(z)
b: corresponde a la parte imaginaria y se denota como Im(z)
Observación
• Si b = 0, «z» se denomina Complejo real o Real puro
• Si a = 0 y b ≠ 0, «z» se denomina Complejo imaginario puro

Ejemplo 1. En el número complejo z = 2 + 3i se tiene

Re(z) = 2 «parte real de z»

Im(z) = 3 «parte imaginaria de z»

Igualdad de números complejos


Dos números complejos son iguales, si son iguales sus partes reales y lo son sus partes
imaginarias. En símbolos, si z1 = a + bi y z2 = c + di, son números complejos, entonces
z1 = z2 implica que a = c y b = d

CLEMENTE MORENO (MATERIAL DIDÁCTICO EN VALIDADIÓN) 3


Ejemplo 1. Los números complejos z1 = x + 5i y z2 = (6 − x) + (y − x)i son iguales cuando

x=6−x y 5=y−x «igualdad entre dos números complejos»

Pero x = 6 − x implica 2x = 6 o bien x = 3 «resolviendo la primera ecuación»

Y, 5 = y − x implica 5 = y − 3 luego y = 8 «resolviendo la primera ecuación»

Luego z1 = 3 + 5i y z2 = (6 − 3) + (8 − 3)i o bien z2 = 3 + 5i «sustituyendo y operando»

Ejemplo 2. Los valores de «x» y «y» que verifican la igualdad 2x + y − 2i = 1 − (3x + y)i son:

Aquí, 2x + y = 1 y −2 = −(3x + y) «igualdad entre dos números complejos»

−2x − y =−1
2x + y = 1 �
Y, � o bien x3x+ +0yy == 21 «sistema que se forma con las ecuaciones y su equivalente»
3x + y = 2

Así, x = 1 y 2 ∙ 1 + y = 1 resulta y = −1 «sustituyendo el valor de «x», operando y despejando»

Luego, 2 ∙ 1 − 1 − 2i = 1 − (3 ∙ 1 − 1)i implica 1 − 2i = 1 − 2i «sustituyendo y operando»

Expresión Binomial y cartesiana de un número complejo


Todo número complejo a + bi, se puede expresar como un par ordenado (a, b) de
números reales, donde la segunda componente del par ordenado corresponde al coeficiente
de la unidad imaginaria «i», entonces:
La expresión cartesiana del número complejo z = a + bi corresponda a z = (a, b)

Ejemplo. El complejo z = 2 − √3i dado en forma canónica o binómica, tiene a z = �2, −√3�
como forma cartesiana

Representación gráfica de un número complejo


El complejo z = (a, b) puede ser representado en un gráfico
de Argand, mediante un vector anclado en el origen cuyo final
tiene coordenadas (a, b). Tal como se ilustra en la figura

CLEMENTE MORENO (MATERIAL DIDÁCTICO EN VALIDADIÓN) 4


Ejemplo. La gráfica de u = 1 − 2i; u = −2 + i; u = 2 + i; u = −1 + 2i y u = 1 + 2i es:

u=2+i u = −1 + 2i u = 1 + 2i
u = 1 − 2i u = −2 + i

Ejercicio N° 1

Calcule el valor de las siguientes raíces y simplifique al máximo sus resultados

1) √−144; 2) −√−0,09; 3) √−0,321;


1
4) �−
81
; 5) √23 − 81; 1
�−4√7056;
1024
3 6)
3 7

1 3 √−256
7)
1 2401
; 8) �− 3√216 + �√64i; 9) ;
7
√−196 − �− 256 2 3
√−343

Calcule cada una de las potencias de «i» y simplifique al máximo los resultados

10) i23 ; 11) i−37 ; 12) 3i14 + 12i17 ;


4
13) 5i50 − 7i52 ; i27 ∙142 ∙i−92
15)
−13i31 +14i67
;
14) i � � ; −i2012
4i6 ∙i7

16) i29 (−i1526 + i887 − i−928 ); 17) (5i63 − 4i41 )2 ; 18)


−19i67 ∙12i31
;
126((i5 )24 )3

Resuelva cada una de las siguientes ecuaciones

19) 6x 2 + 25 = 0; 20) x 2 + 5x + 7 = 0; 21) 5x 2 − 12x + 9 = 0;


23) x 2 − 10x + 34 = 0; 24) 2x 2 + 3x + 2 = 0; 25) 4x 2 + 24x + 37 = 0;
26) 2x 2 + x + 1 = 0; 27) x 2 − 10x + 34 = 0; 28) x 2 − 6x + 13 = 0;

Exprese de manera cartesiana «par ordenado» o forma binómica, según corresponda,


los números complejos siguientes

2 3
29) z = + 5i; 30) z = − i; 31) z = −2 − √2i
3 7

CLEMENTE MORENO (MATERIAL DIDÁCTICO EN VALIDADIÓN) 5


√5 3 4
32) z = �0, � �;
2 33) z = � , 0�; 34) 𝑧𝑧 = � , − �;
3 4 √5
5

35) z =
2
+
√2
i; 36) z = �√2 + 1, 1 − √3�; 37) z = 3 − �2 + √3�i;
√3 √3

Represente gráficamente los siguientes números complejos


3 3 2 3 3
38) z = −2 − i; 39) z = − i; 40) z = + i;
4 √3 √2 5 4

Operaciones con números complejos


Adición de números complejos
Si z1 = a + bi y z2 = c + di entonces z1 + z2 = (a + c) + (b + d)i, si los complejos
están en forma cartesiana z1 = (a, b) y z2 = (c, d) entonces z1 + z2 = (a + c, b + d)
2 1 7 2
Ejemplo. Si z1 = + i y z2 = − + i entonces el complejo que resulta de la suma es
3 3 9 15

2 1 7 2
z1 + z2 = � + i� + �− + i� «expresando la suma de complejos en forma binómica»
3 3 9 15

2 7 1 2
=� − �+� + �i «definición de suma de complejos en forma binómica»
3 9 3 15

1 7
Luego z1 + z2 = − + i «ejecutando la suma de complejos en forma binómica»
9 15

En forma cartesiana se tiene


2 1 7 2
z1 + z2 = � , � + �− , � «expresando la suma de complejos en forma cartesiana»
3 3 9 15

2 7 1 2
=� − , + � «definición de suma de complejos en forma cartesiana»
3 9 3 15

1 7
Luego z1 + z2 = �− , � «ejecutando la suma de complejos en forma cartesiana»
9 15

Sustracción de números complejos


Si z1 = a + bi y z2 = c + di entonces z1 − z2 = (a − c) + (b − d)i. Si los complejos
están en forma cartesiana z1 = (a, b) y z2 = (c, d) entonces z1 − z2 = (a − c, b − d)
3 3
Ejemplo. Si z1 = + 3i y z2 = 2 + i entonces el complejo que resulta de su diferencia es
4 5

CLEMENTE MORENO (MATERIAL DIDÁCTICO EN VALIDADIÓN) 6


3 3
z1 − z2 = � + 3i� − �2 + i� «expresando la diferencia de complejos en forma binómica»
4 5

3 3
= � − 2� + �3 − � i «definición de diferencia de complejos en forma binómica»
4 5

5 12
Luego, z1 − z2 = − + i «ejecutando la diferencia de complejos en forma binómica»
4 5

En forma cartesiana se tiene


3 3
z1 − z2 = � , 3� − �2, � «expresando la diferencia de complejos en forma de par ordenado»
4 5

3 3
= � − 2, 3 − � «definición de diferencia de complejos en forma de par ordenado»
4 5

5 12
Luego, z1 − z2 = �− , � «ejecutando la diferencia de complejos en forma de par ordenado»
4 5

Representación gráfica de la adición y sustracción de números complejos


Dados los números complejos z1 y z2 , entonces

La adición z1 + z2 queda representada en un plano de


argand por la diagonal de un paralelogramo cuyos lados son
los vectores z1 y z2 , es decir:
�����⃗
OP = z1
� entonces �����⃗
OR = z1 + z2
�����⃗ = z2
OQ

La sustracción «resta» z1 − z2 , queda representada por


la suma de z1 con el opuesto de z2 , así: z1 − z2 = z1 + (−z2 ),
es decir:
�����⃗ = z1
OP �����⃗ = z1 − z2
� entonces OT
����⃗
OS = −z2

COMENTARIO

• El neutro aditivo es el complejo (0,0) = 0 + 0i


• El inverso aditivo u opuesto de z es – z, así si z = a + bi, entonces −z = −a − bi

CLEMENTE MORENO (MATERIAL DIDÁCTICO EN VALIDADIÓN) 7


Ejemplo. Si z1 = 2 + 3i y z2 = −5 + 6i entonces las gráficas de z1 + z2 y z1 − z2 son:

𝑧𝑧1 + 𝑧𝑧2 z1 − z2

Módulo de un número complejo

Dado que los números complejos pueden representarse Im


z = (a, b)
como un vector, entonces su módulo es la medida de la longitud
del valor que este complejo representa en el plano de Argand.

Así, si z = a + bi entonces el módulo de |z|, es |z| = √a2 + b 2 a Re

Ejemplo. Para el modulo del número complejo z = 2 − 7i se tiene

|z| = �22 + (−7)2 «aplicando la definición de módulo de vector»

= √4 + 49 «resolviendo potencias»

Luego, |z| = √53 «operando»

COMENTARIO

Si el complejo es real puro o imaginario puro, entonces su representación gráfica se


ubica sobre el eje real e imaginario, respectivamente. Mientras que sus módulos, por ser
distancias unidimensionales, corresponden a la diferencia entre el extremo y el origen cero,
es decir, |Extremo − origen|, lo cual coincide con el número dado

Ejemplo. Para los complejos z1 = 2 y z2 = 3i se tiene

CLEMENTE MORENO (MATERIAL DIDÁCTICO EN VALIDADIÓN) 8


Como z1 = 2 es real puro «se ubica en el eje real»
Im
z1
Y, |𝑧𝑧1 | = |2 − 0| o bien |𝑧𝑧1 | = 2 «distancia unidimensional»
1 2 Re
−1
Como z2 = 3i es imaginario puro «se ubica en el eje imaginario»
−2
z2
Y, |𝑧𝑧2 | = |3 − 0| o bien |𝑧𝑧2 | = 3 «distancia unidimensional» −3

Conjugado de un número complejo

El conjugado del complejo z, denotado con z�, es el simétrico


de z con respecto al eje real. Algebraicamente, si z = a + bi
entonces z� = a − bi, luego el conjugado de un complejo es otro
complejo que, difiere del complejo dado solo en el signo de su
parte imaginaria
1 1
Ejemplo. Siendo z1 = 4 − 5i; z2 = −3i; z3 = −1 y z4 = + i entonces:
2 4

a) ������������ �����������������
(z1 + z2 ) = (4 − 5ı − 3ı) «aplicando conjugado a la suma de z1 y z2 »

����������
= (4 − 8ı) «operando el segundo miembro de la igualdad»

Luego, ������������
(z1 + z2 ) = 4 + 8i «resolviendo conjugada en segundo miembro de la igualdad»

1 1
b) |z�4 | = �2 − 4 i� «tomando conjugada al módulo de z4 »

1 2 1 2
= �� � + �− � «aplicando módulo al complejo del segundo miembro de la igualdad»
2 4

1 1
=� + «resolviendo potencias en el segundo miembro de la igualdad»
4 16

√5
Luego, |z�4 | = 4 «operando en el segundo miembro de la igualdad»

c) �3z�3 − z2 ���������
(2 + ı)� = �3(−1) ���������
������� − �−3i(2 + ı)�� «operación combinada con conjugados»

= |3 ∙ −1 + 3i(2 − i)| «operando en el segundo miembro de la igualdad»

= √02 + 62 «operando en el segundo miembro de la igualdad»

Luego, �3z�3 − z2 ���������


(2 + ı)� = 6

Notas. (1) El conjugado del conjugado de un complejo, es el mismo complejo z� = z


(2) Los módulos o valores absolutos de z y z�; −z y −z� son iguales

CLEMENTE MORENO (MATERIAL DIDÁCTICO EN VALIDADIÓN) 9


Multiplicación de números complejos
Si z1 = a + bi y z2 = c + di son dos números complejos, su producto se define por:

z1 ∙ z2 = (a + bi) ∙ (c + di) (1) «indicando el producto entre z1 y z2 »

= ac + adi + bci + dbi2 «por distributivita del producto respecto a la suma»

= ac + (ad + bc)i − bd «tomando a «i» como factor común y sustituyendo i2 por −1»

Luego, z1 ∙ z2 = ac − bd + (ad + bc)i (2) «agrupando términos semejantes»

O bien, z1 ∙ z2 = (a, b) ∙ (c, d) «transformado los complejos de la identidad (1) a forma cartesiana»

= (ac − bd, ad + bc) «transformado el complejo de la identidad (2) a forma cartesiana»

Entonces, z1 ∙ z2 = (ac − bd, bc + ad) «producto en forma cartesiana»

Ejemplo 1. Si z1 = 1 − 2i y z2 = −5 + 4i, entonces el producto en notación binomial es:

z1 ∙ z2 = (1 − 2i) ∙ (−5 + 4i) «indicando el producto entre z1 y z2 en forma binomial»

= −5 + 4i + 10i − 8i2 «efectuando el producto en forma binomial»

Luego, z1 ∙ z2 = 3 + 14i «sumando términos semejantes»

2 3
Ejemplo 2. Si 𝑧𝑧1 = � , 3� y 𝑧𝑧2 = �2, �, entonces el producto en notación cartesiana es:
3 4

2 3
𝑧𝑧1 ∙ 𝑧𝑧2 = � , 3� ∙ �2, � «indicando el producto entre z1 y z2 en forma cartesiana»
3 4

2 3 2 3
=� ∙2−3∙ , 3∙2+ ∙ � «efectuando el producto en forma cartesiana»
3 4 3 4

4 9 1
=� − , 6+ � «ejecutando operaciones»
3 4 2

11 13
Luego, 𝑧𝑧1 ∙ 𝑧𝑧2 = �− , � «efectuando operaciones»
12 2

División de números complejos


𝑧𝑧1
Si z1 = a + bi y z2 = c + di, con z2 ≠ 0, entonces el resultado de la división se
𝑧𝑧2

obtiene multiplicando el numerador y denominador de la fracción por el conjugado de 𝑧𝑧2 , es


decir:

CLEMENTE MORENO (MATERIAL DIDÁCTICO EN VALIDADIÓN) 10


z1 (a + bi) (c − di)
= ∙ «multiplicando numerador y denominador por conjugada del denominador»
z2 (c + di) (c − di)

ac − adi + bci − bdi2


= «efectuando los productos del numerador y denominador en forma binómica»
c2 −d2 i2

𝑧𝑧1 (ac + bd) + (bc − ad)i


Luego, = «agrupando términos semejantes»
𝑧𝑧2 c2 +d2

z1 ac + bd bc − ad
Mientras que, =� , � «corresponde a la división en forma cartesiana»
z2 c2 + d2 c2 + d2

z1
Ejemplo 1. Si z1 = 1 + i y z2 = 2 + i, entonces se corresponde con:
z2

z1 (1+i) (2−i)
= (2+i) ∙ «multiplicando numerador y denominador por « 2 − i», conjugada del denominador»
z2 (2−i)

2 − i + 2i − i2
= «efectuando los productos del numerador y denominador en forma binómica»
4 − i2
3 + i
= «agrupando términos semejantes»
5

z1 3 1
Luego, = + i
z2 5 5

COMENTARIOS

1 1 z�
• El reciproco z −1 del complejo z, se corresponde con z −1 = o bien z −1 = ∙
z z z�
a − bi a b
Pero z = a + bi implica z −1 = o z −1 = − i en forma binómica,
a2 + b2 a2 + b2 a2 + b2
a −b
y z −1 = � , � en forma cartesiana
a2 + b2 a2 +b 2

• El complejo 0 + 0i, no tiene inverso multiplicativo

Ejemplo 2. Si z = 7 − 4i ¿qué número representa a 𝑧𝑧 −1 ?


1 (7+4𝑖𝑖)
𝑧𝑧 −1 = ∙ 1 z�
«aplicando definición de reciproco z −1 = z ∙ �»
(7−4𝑖𝑖) (7+4𝑖𝑖) z

7 + 4i
= «efectuando los productos del numerador y denominador en forma binómica»
49 − 16i2

7 4
Luego, z −1 = + i
65 65

CLEMENTE MORENO (MATERIAL DIDÁCTICO EN VALIDADIÓN) 11


Raíz cuadrada de un número complejo en forma binómica

Siendo z = a + bi un complejo en forma binómica, su raíz √a + bi = x + yi (∗) es:

√a + +bi = x + yi implica a + bi = x 2 + 2xyi + y 2 i2 «elevando ambos miembros al cuadrado»

O bien a + bi = (x 2 − y 2 ) + 2xyi «cambiando i2 por −1, reuniendo real con real e imaginario con imaginario»

x2 − y2 = a (1)
Así, � «es el sistema que resulta de la igualdad»
2xy = b (2)
b b 2
Luego, y = y x2 − � � = a (3) «despejando a «y» en (2) y sustituyendo su valor en (1)»
2x 2x

b2
de x 2 − = a se tiene 4x 4 − 4ax 2 − b2 = 0 (4) «operando en (3) y despejando para llegar a (4)»
4x2

de (4) resulta 4u2 − 4au − b2 = 0 (5) «haciendo u = x 2 y sustituyendo se pasa de (4) a (5)»

4a ± �(4a)2 − 4∙4∙(−b2 ) 4a ± �16(a2 +b2 )


y, u = o bien u = (6) «aplicando la resolvente en (5) y operando»
2∙4 8

4�a ± √a2 +b2 � a ± √a2 + b2


de (6) se tiene u = o bien u = «tomando factor común 4 y simplificando»
8 2

a + √a2 + b2
Así, x 2 = (7) «devolviendo el cambio u = x 2 y tomando la raíz positiva»
2

a+r r+a
En (7) x 2 = o bien x = ±� (8) «siendo 𝐫𝐫 = √𝐚𝐚𝟐𝟐 + 𝐛𝐛 𝟐𝟐 y tomando raíz en ambos lados»
2 2

a+r a+r
Y, − y 2 = a o bien y 2 = −a (9) «sustituyendo (7) u (8) en (1) y luego operando»
2 2

r−a r−a
En (9) y 2 = o bien y = ±� (10) «operando (9) y luego tomando raíz en ambos lados»
2 2

r + a r−a
Luego, √a + bi = ±� ±� i «sustituyendo (8) y (10) en (∗)»
2 2

COMENTARIOS

• Si «b» es positivo, «x» y «y» toman igual signo, es decir


𝑟𝑟 + 𝑎𝑎 𝑟𝑟 − 𝑎𝑎
√𝑎𝑎 + 𝑏𝑏𝑏𝑏 = ±� 2
±�
2

• Si «b» es negativo, «x» y «y» toman signo diferente, es decir

CLEMENTE MORENO (MATERIAL DIDÁCTICO EN VALIDADIÓN) 12


𝑟𝑟 + 𝑎𝑎 𝑟𝑟 − 𝑎𝑎
√𝑎𝑎 + 𝑏𝑏𝑏𝑏 = ±� 2
∓�
2

Ejemplo 1. Siendo z = −5 − 12i, calcular √z, es decir √−5 − 12i

r +(−5) r −(−5)
Como √−5 − 12i = ±� ∓� i «aplicando fórmula para hallar la raíz cuadrada de un complejo»
2 2

Y, r = �(−5)2 + (−12)2 así r = √169 o bien r = 13 «calculando «r» aplicando r = √a2 + b 2 »

13−5 13+5
Así, √−5 − 12i = ±� ∓� i «sustituyendo e invirtiendo signos «± y ∓, pues b = −12» es negativo»
2 2

8 18
= ±� ∓ � i «ejecutando la suma algebraica en los numeradores de los radicandos»
2 2

= ±2 ∓ 3i «dividiendo y calculando la raíz cuadrada»

Luego, z1 = 2 − 3i y z2 = −2 + 3i son las raíces de z = √−5 − 12i

Ejemplo 2. Calcular la raíz de z = √8 + 15i

r+8 r−8
√8 + 15i = ±� ±� i «aplicando fórmula para hallar la raíz cuadrada de un complejo»
2 2

Y, r = √82 + 152 así r = √289 o bien r = 17 «calculando «r» aplicando r = √a2 + b 2 »

17 + 8 17 − 8
Así, √8 + 15i = ±� ±� i «conservando signos y sustituyendo el valor de r = 17»
2 2

25 9
= ±� ±� i «ejecutando la suma algebraica en los numeradores de los radicandos»
2 2

5√2 3√2
=± ± i
2 2

5√2 + 3√2i −5√2 − 3√2i


Luego, z1 = y z2 = «raíces requeridas»
2 2

CLEMENTE MORENO (MATERIAL DIDÁCTICO EN VALIDADIÓN) 13


Ejercicio N° 2
2 1 3 2 3
Dados los complejos z1 = 3 − 4i; z2 = + i; z3 = − + 2i y z4 = − i, hallar:
3 2 4 5 4

1) z1 + z2 ; 2) z2 − z3 ; 3) z3 − z4 ; 4) z2 − z1 ;

5) z1 − 3(z2 − z3 ); 6)
2 1
z3 + (z1 − z4 );
3
7) 2z3 − (z4 − z3 );
2
8) z2 − (z1 + z4 );
5 2 4 3

1 2 3 2
Dados los complejos u1 = 2 − 3i u2 = + i y u3 = − − i, determinar:
3 5 4 7

9) |u1 − u2 |; 10) |u2 − (u1 + u3 )|; 11) |3u2 + (u1 − u3 )|; 1


12) � u1 − u2 + u3 �;
2
2 3

13) ������������
(𝑢𝑢1 − 𝑢𝑢2 ); 14) ���������������������
[𝑢𝑢2 − (𝑢𝑢1 + 𝑢𝑢3 )]; 15) �����������������������
[3𝑢𝑢2 + (𝑢𝑢1 − 𝑢𝑢3 )]; ��������������������
1 2
16) u1 − u2 + u3 ;
2 3

1 2 2 3 3
Dados los complejos v1 = 2 − 3i; v2 = + i; v3 = − − i y v4 = 3 − i hallar:
2 3 5 4 5

17) v1 ∙ v2 ; 18) v2 ∙ ���;


v3 19) ���
v3 ∙ v4 ; 20) ��������;
v3 ∙ v4
v1 v2 ���3�
v ������
21) ; 22) ; 23) ; v
24) � 3 �;
v2 ���3�
v v4 v4

v1 ∙v2
25) ; 26) v3 ∙ v2−1 ; 27)
v−1
3
; 28)
���2�
v4 ∙v
;
v3 ����
v4 ���1�∙v3
v

Calcule la raíz cuadrada de los siguientes números complejos

29) z = √3 + 4i; 30) u = √−7 + 24i; 31) v = √9 − 40i; 32) z = √−11 − 60i;

33) z = √−5i; 34) w = √9 − 40i; 35) 𝑢𝑢 = √11 + 60i; 36) v = √−45 + 28i;

37) z = √12 − 35i; 38) u = √63 + 16i; 39) v = √−28 + 44i; 40) w = √36 + 77i;

CLEMENTE MORENO (MATERIAL DIDÁCTICO EN VALIDADIÓN) 14


Argumento de un número complejo

El argumento de un número complejo z = a + bi, denotado


Im
por Arg z , se corresponde con ángulo «𝛼𝛼» que forma el vector �����⃗
0N N
b
en representación de «z» con el semieje real positivo
El argumento de un complejo «z», toma una cantidad infinita α
0 a Re
de valores que se diferencian entre sí por el numero entero «k»

de vueltas que describe «𝛼𝛼», es decir


Arg z = α + 2kπ con k ∈ ℤ
El argumento principal está comprendido en el intervalo [0, 2𝜋𝜋)

Forma trigonométrica de un número complejo

Si |𝑧𝑧| es el modulo del complejo z = (a, b) o bien «r» designa


Im
el módulo de z = a + bi, como se ilustra en la figura, se tiene: N(a, b)
a
cos α = así a = rcos α y
r b
b α
sen α = así b = rsen α a
r 0 Re

Entonces z = a + bi puede escribirse z = rcos α + (rsenα)i

Así, z = r(cosα + isenα) «tomando factor común «r», se tiene «𝑧𝑧» en forma trigonométrica»

O bien z = rCis α «manera abreviada de escribir «𝑧𝑧» en forma trigonométrica»

COMENTARIO

Para pasar un complejo «z» de forma binómica y/o cartesiana a forma trigonométrica,
se requiere el módulo r = √a2 + b 2 y el ángulo, el cual calcula con las inversas de las
funciones seno, coseno y tangente, indicadas en las expresiones siguientes:
b a b
α = sen−1 � �, α = cos −1 � �; α = tg −1 � �
r r a

CLEMENTE MORENO (MATERIAL DIDÁCTICO EN VALIDADIÓN) 15


Ejemplo 1. Expresar el complejo z = −√2 + √2i en forma trigonométrica

2 2
r = ��−√2� + �√2� = √2 + 2 o bien r = 2 «cálculo de «r»» Im

√2
√2 π
α = tg −1 = tg −1 (−1) o bien α = − «cálculo de «α»»
−√2 4
α
π 3
Luego α = π − o bien z = π «z = (−√2, √2) es del segundo cuadrante» −√2 0 Re
4 4

3
Por ello, z = 2Cis � π� o bien z = 2Cis135° «expresión trigonométrica de «z»»
4

Ejemplo 2. Expresar en forma trigonométrica el complejo z = −3 − 4i

r = �(−3)2 + (−4)2 = √25 o bien r = 5 «cálculo de «r»» Im


−4 α
α = tg −1 � � = tg −1 �1, 3� � o bien α = 53,1301024 «cálculo de «𝛼𝛼»» −3 −2 −1 0 Re
−3 −1

o bien α = 53°7′48′′ y α = 237°7´48′′ «𝑧𝑧 = (−3, −4) es del tercer cuadrante» −2


−3
Por ello, z = 5Cis233°7′48′′ «expresión trigonométrica de «z»» −4

COMENTARIO

Si 𝑧𝑧 = 1 entonces z = Cis0° ««z» es un real puro de forma binómica a trigonométrica»

z = i entonces z = Cis90° ««z» es un imaginario puro de forma binómica a trigonométrica»

z = −1 entonces z = Cis180° ««z» es un real puro de forma binómica a trigonométrica»

z = −i entonces z = Cis270° ««z» es un imaginario puro de forma binómica a trigonométrica»

Ejemplo 3. Siendo 𝑧𝑧 = 5 se tiene z = 5 ∙ 1, entonces z = 5Cis0° «forma trigonométrica de «z»»

Ejemplo 4. Para z = 7i se tiene z = 7 ∙ (i), luego z = 7Cis90° «forma trigonométrica de «z»»

CLEMENTE MORENO (MATERIAL DIDÁCTICO EN VALIDADIÓN) 16


COMENTARIO

A la inversa para llevar un número complejo de forma trigonométrica a forma binómica


se extiende la notación rCisα = r(cosα + isenα), tal como ilustran los ejemplos siguientes:

Ejemplo 1. Llevar a forma binómica el complejo z = √3Cis150°

z = √3(cos150° + isen150°) «expansión de la forma trigonométrica de «z»»

√3
Y, cos150° = cos(180° − 30°) = −cos30° así cos150° = − «reducción al primer cuadrante»
2
1
Y, sen150° = sen(180° − 30°) = sen30° así sen150° = «reducción al primer cuadrante»
2

√3 1 3 √3
Luego, z = √3 �− + i� o bien z = − + i «sustituyendo y operando»
2 2 2 2

Ejemplo 2. Expresar en forma binómica el complejo u = 2√3Cis210°

u = 2√3(cos210° + isen210°) «expansión de la forma trigonométrica de «u»»

√3
Y, cos210° = cos(180° + 30°) = −cos30° así cos210° = − «reducción al primer cuadrante»
2
1
Y, sen210° = sen(180° + 30°) = −sen30° así sen210° = − «reducción al primer cuadrante»
2

√3 1
Luego, u = 2√3 �− − i� o bien u = −3 − √3i «sustituyendo y operando»
2 2

COMENTARIO

Dos números complejos 𝑧𝑧1 y 𝑧𝑧2 en forma trigonométrica son iguales, si verifican:
r1 = r2
z1 = z2 ⇒ �α = α + 2kπ con k ∈ ℤ
1 2

Ejemplo 3. Los complejos z1 = √5Cis45° y z2 = √5Cis1485° son iguales debido a que:

r1 = r2 = √5 y 1485° = 45° + 4 ∙ 360° entonces 45° = 45° + 2(4π)

CLEMENTE MORENO (MATERIAL DIDÁCTICO EN VALIDADIÓN) 17


Multiplicación de números complejos en forma trigonométrica
Si z1 = r1 Cis α y z2 = r2 Cisβ son complejos, entonces su producto se obtiene por:

z1 ∙ z2 = r1 Cis α ∙ r2 Cisβ «planteamiento del producto»

z1 ∙ z2 = r1 (cos α + isen α) ∙ r2 (cosβ + isenβ) «expansión de la forma trigonométrica»

z1 ∙ z2 = r1 ∙ r2 (cos α + isenα) ∙ (cosβ + isenβ) «replanteando el producto»

z1 ∙ z2 = r1 ∙ r2 (cosαcosβ + icosαsenβ + isenαcosβ + i2 senαsenβ) «ejecutado el producto»

z1 ∙ z2 = r1 ∙ r2 [cosαcosβ − senαsenβ + i(cosαsenβ + senαcosβ)] «operando y ordenando»

z1 ∙ z2 = r1 ∙ r2 [cos(α + β) + isen(α + β)] «definición del coseno y seno de la suma de dos ángulos»

z1 ∙ z2 = r1 ∙ r2 Cis(α + β) «forma abreviada de escribir el producto en forma trigonométrica»

√3
Ejemplo 1. El producto de z1 = √3cis25° y z2 = Cis 35° se corresponde con:
5

√3
z1 ∙ z2 = �√3Cis25°� ∙ � Cis35°� «planteamiento de la multiplicación»
5

√3
z1 ∙ z2 = √3 ∙ Cis(25° + 35°) «aplicando definición de producto en forma trigonométrica»
5
3
z1 ∙ z2 = Cis60° (∗) «ejecutando operaciones»
5

Si este producto se quiere llevar a forma binómica se tiene:


3
z1 ∙ z2 = (cos60° + isen60°) «expansión de la forma trigonométrica de «𝑧𝑧1 ∙ 𝑧𝑧2 » dada en (∗)»
5

3 1 √3 3 3√3
z1 ∙ z2 = � + i� o bien z1 ∙ z2 = + i «sustituyendo valores del seno y coseno de 60° y operando»
5 2 2 10 10

Potenciación de números complejos, fórmula de Moivre


La potencia enésima de número complejo en forma trigonométrica se determina por la
fórmula de Moivre, cuya expresión, se infiere como se indica a continuación:

z 2 = (rCisα)2 implica z 2 = (rCisα)(rCisα) o bien z 2 = r 2 Cis(2α) «ejecutando operaciones»

z 3 = (rCisα)3 así z 3 = (rCisα)(rCisα)(rCisα) o bien z 3 = r 3 Cis(3α) «ejecutando operaciones»

….
z n = (rCisα)n así z n = (rCisα)(rCisα) … (rCisα) o bien z n = r n Cis(nα) «fórmula de Moivre»

Si «n» es un entero negativo se tiene:

CLEMENTE MORENO (MATERIAL DIDÁCTICO EN VALIDADIÓN) 18


1 1
z −n = = «aplicando reciproco de «z» y sustituyendo denominador por fórmula de Moivre»
zn rn Cis (nα)

r−n Cis(−nα)
Y, z −n = ∙ «multiplicando numerador y denominador por «Cis(−nα)»»
Cis(nα) Cis(−nα)

r−n Cis(−nα) r−n Cis(−nα)


Así, z −n = = «ejecutando operaciones»
Cis(nα−nα) Cis0°

Pero Cis0° = cos0° + isen0° o bien Cis0° = 1 «pues, 𝑐𝑐𝑐𝑐𝑐𝑐0° = 1 y 𝑠𝑠𝑠𝑠𝑠𝑠0° = 1»

Luego, z −n = r −n Cis(−nr) «fórmula de Moivre para «−n»»

Ejemplo. Siendo z = √2Cis30°, hallar z 4 y 𝑧𝑧 −5


4 4
z 4 = �√2Cis30°� = �√2� Cis(4 ∙ 30°) o bien z 4 = 4Cis120° «por fórmula de Moivre y operaciones»

−5 √2
z −5 = (√2Cis30°)−5 = �√2� Cis(−5 ∙ 30) o bien z −5 = Cis(−150°) «operando fórmula de Moivre»
8

División de números complejos en forma trigonométrica


Si z1 = r1 Cis α y z2 = r2 Cisβ son complejos, entonces su cociente se obtiene por:
z1 r1 Cisα
= «planteamiento de la división»
z2 r2 Cisβ
z1 r1
= [Cisα ∙ (Cisβ)−1 ] «pues
Cisα 1 1
= Cisα ∙ Cisβ y (Cisβ)−1 = Cisβ »
z2 r2 Cisβ

z1 r1
= Cis α Cis(−β) «aplicando la fórmula de Moivre para «−n»»
z2 r2
z1 r1
Luego, = Cis(α − β) «aplicando multiplicación complejos de forma trigonométrica»
z2 r2

Ejemplo 1. Si z1 = 12Cis120° y z2 = 4Cis 75° entonces su cociente es:


z1 12Cis 120° z1 12 z1
= así = Cis(120° − 75°) o bien = 3Cis45° «aplicando división y operando»
z2 4Cis 75° z2 4 z2

Ejemplo 2. Si z1 = √3Cis30° y z2 = √2Cis75° entonces la división de z1 entre z2 es:


z1 √3 Cis30° 𝑧𝑧1 √3 z1 √6
= así = Cis(30° − 75°) o bien = Cis(−45°) «aplicando división y operando»
z2 √2 Cis75° 𝑧𝑧2 √2 z2 2

CLEMENTE MORENO (MATERIAL DIDÁCTICO EN VALIDADIÓN) 19


Raíz enésima de un número complejo en forma trigonométrica
n n
Como √a = b sí y sólo si bn = a, entonces la √rCisα = pCisβ al verificar esta definición,
es decir:
n
√rCisα = pCisβ implica (pCisβ)n = rCisα (1) «aplicando definición de raíz enésima»

Y, pn Cis(nβ) = rCisα «aplicando fórmula de Moivre al primer miembro de la igualdad»

pn = r (1)
Luego, � «aplicando igualdad de complejos»
nβ = α (2)
n α+2kπ
De (1), p = √r ; de (2), β = «donde «k» va a representar número de raíces»
n

n n α + 2kπ
Por ello, √rCisα = √rCis � � (3) «sustituyendo (2) en el segundo miembro de (1)»
n

Siendo k = 0, 1, 2, 3 … . , n − 1 «cuando «k» asume esos valores aparecen las «n» raíces del complejo»

n α
Así, z0 = √rCis � �
n

n α + 2π
z1 = √rCis � �
n

n α + 4π
z2 = √rCis � �
n

….
n α + 2(n−1)π
zn−1 = √rCis � �
n

4 4
Ejemplo 1. Siendo z = 625Cis45°, hallar √z o bien √625Cis45°
4 4 45°+2∙180°k
√625Cis45° = √625Cis � 4
� «aplicando la igualdad «(3)»»

45°+k∙360°
= 5Cis � � «ejecutando operaciones»
4

Dado que k = 0,1,2,3 asume estos valores, «z» tiene sólo cuatro raíces, a saber:
45°
z0 = 5Cis � � o bien z0 = 5Cis(11,25°)
4

45°+360° 405°
z1 = 5Cis � � así z1 = 5Cis � � o bien z1 = 5Cis(101,25°)
4 4

45°+2∙360° 765°
z2 = 5Cis � � así z2 = 5Cis � � o bien z2 = 5Cis(191,25°)
4 4

45°+3∙360° 1125°
z3 = 5Cis � � así z3 = 5Cis � � o bien z3 = 5Cis(281,25°)
4 4

CLEMENTE MORENO (MATERIAL DIDÁCTICO EN VALIDADIÓN) 20


3
Ejemplo 2. Si z = −√2 + √2i calcular √𝑧𝑧

Como «𝑧𝑧» está en forma binómica se debe pasar a forma trigonométrica, para ello se tiene:

2 2
r = ��−√2� + �√2� = √2 + 2 o bien r = 2 «cálculo del módulo de «z»»

√2
α = tg −1 � � así α = tg −1 (−1) o bien α = −45° «el argumento de «z» es del 2do cuadrante»
−√2

Así, α = 180° − 45° o bien α = 135° «ángulo recorrido hasta el 2do cuadrante»

Luego, z = 2Cis135° «forma trigonométrica de «z»»

3 3 135°+k∙360°
Y, √2Cis135° = √2Cis � � «aplicando fórmula de raíz enésima»
3

Aquí, k = 0, 1, 2 indica que el complejo «z» tiene tres raíces a saber:


3 135° 3
z0 = √2Cis � � o bien z0 = √2𝐶𝐶𝐶𝐶𝐶𝐶45°
3

3 135°+360° 3 495° 3
z1 = √2Cis � � así z1 = √2Cis � � o bien z1 = √2Cis165°
3 3

3 135+2∙360° 3 855° 3
z2 = √2Cis � � así z2 = √2Cis � � o bien z2 = √2Cis285°
3 3

Ejercicio N° 3
Los complejos siguientes están en forma binómica, expréselos en forma trigonométrica

1) z = 2 − 2i; 2) v = −3 + 4i; 3) u = 4 + 4i; 4) w = 3√2 − √3i;

5) w = −
1 √3
i; 6) u = 5 − 4i; 7) v =
√3
+ ;
i 8) z = −6i;
3 3 2 2

9) z = 11 + 9i; 10) v = √5 − √7i; 11) u = √3 + i; 12) w = √21 + √7i;

Los siguientes complejos están en forma trigonométrica, expréselos en forma binómica

13) z = √3Cis30°; 14) v = √2Cis45°; 15) u = 4Cis210°; 16) w =


√2
Cis405°;
2

17) w = 2Cis315°; 18) u =


√3
Cis420°; 19) v = Cis35°25′; 20) z = 4Cis38°15′;
2

2
21) z = Cis120°; 22) v = √3Cis240°; 23) u = 6Cis180°; 24) w = √5Cis150°;
3

CLEMENTE MORENO (MATERIAL DIDÁCTICO EN VALIDADIÓN) 21


2 √5
Dados los complejos z1 = 5Cis25°; z2 = √3Cis36°; z3 = Cis41° y z4 = Cis23°, hallar:
3 4

25) z1 ∙ z2 ; 26) z2 ∙ z3 ; 27) z3 ∙ z4 ; 28) z1 ∙ z3 ;

29) z1 ∙ z2 ∙ z3 ; 30) z1 ∙ z2 ∙ z4 ; 31) z2 ∙ z3 ∙ z4 ; 32) z1 ∙ z2 ∙ z3 ∙ z4 ;

33) (z1 )2 ; 34) (z2 )−3 ; 35) (z3 )4 ; 36) (z4 )−5 ;

Calcule cada una de las siguientes potencias

7
37) (3Cis23°)4 ; 38) �√2Cis15°� ;
5
√2 40) (3Cis40°)−3 ;
39) � Cis46°� ;
2

2 −4 6 −3 2 4
41) � Cis30°� ; 42) �
√6
Cis100°� ; 43) �2√3Cis(−10)� ; 44) � Cis24°� ;
3 3 √2

Dados los complejos 𝑧𝑧1 = 6𝐶𝐶𝐶𝐶𝐶𝐶90°; 𝑧𝑧2 = 3𝐶𝐶𝐶𝐶𝐶𝐶26°; 𝑧𝑧3 = 9𝐶𝐶𝐶𝐶𝐶𝐶30° y 𝑧𝑧4 = 12𝐶𝐶𝐶𝐶𝐶𝐶40°, hallar:

z1 z2 z3 z1
45) ; 46) ; 47) ; 48) ;
z2 z3 z4 z4

z z z z z z z1 z z z
49) � 2 ÷ 1 � 50) � 2 ÷ 1 � 51) � 1 ÷ 3 � ÷ 52) � 1 ÷ 2 � ÷ 3 ;
z4 z3 z3 z4 z2 z4 z3 z4 z3 z4

En cada caso, determine la raíz que se indica

3 4
53) √9Cis40°; 54) √27Cis32°; 55) √81Cis52°; 5 1
56) � Cis35° ;
32

4 3
57) √−1296i; 58) √2 − 2i; 4
59) �−1 − √3i;
5
60) �√12 + 2i;

CLEMENTE MORENO (MATERIAL DIDÁCTICO EN VALIDADIÓN) 22


República Bolivariana de Venezuela
Ministerio del Poder Popular para la Educación
Unidad Educativa: Colegio San Agustín – El Marqués
Material elaborado por el Prof. Clemente Moreno

Sucesiones

Una sucesión es un conjunto ordenado de elementos llamados términos, tales que cada
uno de ellos ocupa un lugar establecido, de modo que se puede distinguir cual es el primero
término, cual el segundo, cual el tercero y así sucesivamente, los términos obedecen a un
orden de formación, criterio de orden o fórmula de recurrencia

Ejemplo 1. La secuencia 1, 1, 2, 3, 5, 8, 13, …. conocida como sucesión de Fibonacci,


muestra los siete primeros términos t1 , t 2 , t 3 , t 4 , t 5 , t 6 , t 7 ,… aquí, el subíndice indica el lugar
que ocupa el término en la sucesión. Así t1 es el primer término, t 2 el segundo término y t n
un término cualquiera de la sucesión, tal como se indica

t1 t2 t3 t4 t5 t6 t7

1 1 2 3 5 8 13

COMENTARIO

Si nos fijamos en el tercer término «t 3 » se puede apreciar que «t 3 = 2» y «2» es la


suma de «t1 + t 2 », luego t 3 = t1 + t 2
De igual modo, el cuarto término «t 4 = 3» es la suma de «t 2 + t 3 », luego t 4 = t 2 + t 3
También, el quinto término «t 5 = 5» es la suma de «t 3 + t 4 », entonces t 5 = t 3 + t 4
En general, «t n = t n−2 + t n−1 » es la expresión que permite hallar cualquier término de
la sucesión de Fibonacci. Así, si «n = 7» se tiene t 7 = t 7−2 + t 7−1 o bien t 7 = t 5 + t 6
El problema asociado a la identificación de la expresión que define el orden de
formación de una sucesión, es un problema central en el estudio de las series, que tiene en
Fourier «matemático francés del siglo XIX» el exponente máximo de esta parte de la
matemática, cuyo estudio teórico es parte del currículo de las matemáticas superiores

CLEMENTE MORENO (MATERIAL DIDÁCTICO EN VALIDACIÓN) 1


Este curso, se aborda el estudio básico de dos tipos de sucesiones particulares, que
son las progresiones aritméticas y las progresiones geométricas

Progresiones aritméticas
Una progresión aritmética es una sucesión tal que cada término después del primero, se
obtiene añadiendo a éste una cantidad constante llamada razón

Ejemplo 1. La sucesión 3, 7, 11, 15, 19, … es una progresión aritmética «P. A.», ya que:

a1 = 3 «primer término de la P. A.»

a2 = 7 implica a2 = 3 + 4 o bien a2 = a1 + r «segundo término es igual anterior más la razón»

a3 = 11 implica a3 = 7 + 4 o bien a3 = a2 + r «tercer término es igual anterior más la razón»

a4 = 15 implica a4 = 11 + 4 o bien a4 = a3 + r «cuarto término es igual anterior más la razón»

a5 = 19 implica a5 = 15 + 4 o bien a5 = a4 + r «quinto término es igual anterior más la razón»

En general, si «an » es una «P. A.» que inicia en «a1 », con razón «r», se tiene

a2 = a1 + r
a3 = a2 + r
…………..
an = an−1 + r
……………

COMENTARIO

a2 = a1 + r implica r = a2 − a1 y en general an = an−1 + r implica r = an − an−1 , luego


una sucesión es aritmética si la diferencia entre un término y el anterior es una cantidad
constante

CLEMENTE MORENO (MATERIAL DIDÁCTICO EN VALIDACIÓN) 2


Ejemplo 2. Los múltiplos positivos de 7 conforman una «P. A.» de razón r = 7, cuyos primeros
términos son: 7, 14, 21, 28, 35, 42,… pues
a1 = 7 «primer término»

a2 = a1 + r implica a2 = 7 + 7 o bien a2 = 14 «segundo término»

a3 = a2 + r implica a3 = 14 + 7 o bien a3 = 21 «tercer término»

………
an = an−1 + 7 «término enésimo»

2
Ejemplo 3. Construir una «P. A.» de 5 términos que inicie en 3 y tenga − por razón
3
2
Aquí, a1 = 3 y r = − «identificación de datos»
3
2 7
a2 = 3 + �− � entonces a2 = «cálculo 2do término»
3 3
7 2 5
a3 = + �− � entonces a3 = «cálculo 3er término»
3 3 3
5 2 3
a4 = + �− � entonces a4 = o bien a4 = 1 «cálculo 4to término»
3 3 3
2 1
a5 = 1 + �− � entonces a5 = «cálculo 5to término»
3 3
1 2 1
a6 = + �− � entonces a6 = − «cálculo 6to término»
3 3 3
7 5 1 1
Luego 3, , , 1, , − «es la «P. A.» solicitada»
3 3 3 3

Ejemplo 4. La sucesión an = 3n − 2 es una progresión aritmética, pues


an = 3n − 2 «término enésimo»

an+1 = 3(n + 1) − 2 «término n + 1»

an+1 − an = 3(n + 1) − 2 − (3n − 2) «diferencia entre un término y el anterior»

= 3n + 3 − 2 − 3n + 2 «operando»

=3 «es una progresión aritmética de razón 3»

Luego 1, 4, 7, 10, 13, 16,… 3n − 2, … «son los términos de la «P. A. » solicitada»

CLEMENTE MORENO (MATERIAL DIDÁCTICO EN VALIDACIÓN) 3


Término general de una progresión aritmética

Dados a1 y la razón r «conocidos a1 y r»

a2 = a1 + r
a3 = a2 + r implica a3 = (a1 + r) + r o bien a3 = a1 + 2r «sustituyendo a2 = a1 + r en a2 de a3 »

a4 = a3 + r implica a4 = (a1 + 2r) + r o bien a4 = a1 + 3r «sustituyendo a3 = a1 + 2r en a3 de a4 »

a5 = a4 + r implica a5 = (a1 + 3r) + r o bien a5 = a1 + 4r «sustituyendo a4 = a1 + 3r en a4 de a5 »

…… …………. ……….
an = a1 + (n − 1)r (1) «fórmula general de una P. A.»

COMENTARIO

Si el desarrollo anterior se hubiese iniciado en un término cualquiera «ak », en lugar de


«a1 » la ecuación (1) se generaliza y se tendría
an = ak + (n − k)r (2)

Ejemplo 1. ¿Cuál es el milésimo término de una «P. A.» que inicia en −7 y tiene a 5 de razón?
Aquí, a1 = −7, r = 5, n = 1000 y a1000 =? «identificación de datos e incógnita»

Y, a1000 = −7 + (1000 − 1) ∙ 5 «aplicando (1), fórmula general»

Así, a1000 = −7 + 999 ∙ 5 o bien a1000 = 4988 «ejecutando operaciones»

Ejemplo 2. ¿Cuántos enteros divisibles entre 6 existen entre 32 y 395?


Como los enteros divisibles entre 6 «múltiplos de 6», forman una «P. A.» de razón 6, entonces
según el enunciado del problema: 36 es el primer entero mayor a 32 que es divisible entre 6 y
390 el último entero menor a 395 que es divisible entre 6, se tiene:
a1 = 36, an = 390, r = 6 y n =? «identificación de datos e incógnita»

Y, 390 = 36 + (n − 1) ∙ 6 «sustituyendo datos en fórmula general»

Así, 390 = 36 + 6n − 6 o bien 390 − 30 = 6n «operando y aislando a «n»»


360
Y, 360 = 6n o bien n = , así n = 60 «luego, entre 32 y 395 hay 60 múltiplos de 6»
6

CLEMENTE MORENO (MATERIAL DIDÁCTICO EN VALIDACIÓN) 4


1
Ejemplo 3. ¿Cuál es el décimo término de una «P. A.» de razón , cuyo quinto termino es 4?
2
1
Aquí, r = , a5 = 4, k = 5, a10 =?, n = 10 «identificación de datos e incógnita»
2
1
Y, a10 = 4 + (10 − 5) ∙ � � «sustituyendo datos en (2), ecuación generalizada»
2
5 13
Así, a10 = 4 + o bien a10 = «operando se tiene el término solicitado»
2 2

Ejemplo 4. En una «P. A.», la suma del segundo y del quinto término es 28, mientras que la
diferencia entre el séptimo y el primero es 24, determine la razón y los primeros ocho términos
de la progresión
a2 + a5 = 28
Aquí, � , r =?, a1 =? «identificación de datos e incógnitas»
a7 − a1 = 24
(a + r) + (a1 + 4r) = 28
Y, � 1 «sistema equivalente, en términos de «r» y «a1 »»
(a1 + 6r) − a1 = 24
2a + 5r = 28 (1)
Así, � 1 «operando el sistema anterior resulta este sistema»
6r = 24 (2)
24
De (2), r = o bien r = 4 «despejando y simplificando»
6

De (1), 2a1 + 5 ∙ 4 = 28 o bien 2a1 + 20 = 28 «sustituyendo el valor de «r» en (1) y operando»

Así, 2a1 = 28 − 20 es decir 2a1 = 8 o bien a1 = 4 «operando y despejando»

Luego 4, 8, 12, 16, 20, 24, 28, 32, 36, ….. «es la «P. A.» solicitada»

Interpolación de medias aritméticas

COMENTARIO

Interpolar una cantidad «k» de términos entre dos términos dados de una «P. A.», es
construir una «P. A.» de extremos los términos dados, es decir, «a1 » coincidente con el
extremo inferior, «an » coincidente con el extremo superior y «n = k + 2» el número de
términos de la progresión, en símbolos

an = a1 + [(k + 2) − 1] ∙ r o bien an − a1 = (k + 1) ∙ r «sustituyendo, operando y aislando a «r»»

an − a1
Así, r = «despejando a «r»»
k+1

CLEMENTE MORENO (MATERIAL DIDÁCTICO EN VALIDACIÓN) 5


Ejemplo 1. Interpolar 4 medias aritméticas entre 3 y 7
Aquí, a1 = 3, an = 7, k = 4 y r =? «identificación de datos e incógnita»
an − a1 7−3 4
r= implica r = o bien r = «sustituyendo y operando»
k+1 4+1 5
4 15 + 4 19
a2 = 3 + así a2 = o bien a2 = «cálculo 2do término»
5 5 5
19 4 23
a3 = + o bien a3 = «cálculo 3er término»
5 5 5
23 4 27
a4 = + o bien a4 = «cálculo 4to término»
5 5 5
27 4 31
a5 = + o bien a5 = «cálculo 5to término»
5 5 5
19 23 27 31
Luego 3, , , , , 7 «interpolación de las medias aritméticas requeridas»
5 5 5 5
31 4 35
Nota. a6 = + o bien a6 = =7 «como a6 = 7 coincide con an = 7 la interpolación es correcta»
5 5 5

Suma de los términos de una progresión aritmética


Sea a1 , a2 , a3 ,… an−2 , an−1 , an una «P. A.» de «n» términos y «Sn » la suma de estos,
Así, Sn = a1 + a2 + a3 + ⋯ + an−2 + an−1 + an «sumando los «n» términos de la «P. A.»»

Y, Sn = a1 + (a1 + r) + (a1 + 2r) + ⋯ + (an − 2r) + (an − r) + an (1) «suma en función de a1 y r»

Y, Sn = an + (an − r) + (an − 2r) + ⋯ + (a1 + 2r) + (a1 + r) + a1 (2) «invirtiendo orden en suma (1)»
Así, 2Sn = (a1 + an ) + (a1 + an ) + (a1 + an ) + ⋯ + (an + a1 ) + (an + a1 ) + (an + a1 ) «sumando miembro a miembro»

Luego 2Sn = n ∙ (a1 + an ) «el término (a1 + an ) se repite «n» veces»

n∙(a1 + an )
Por ello Sn = «fórmula de la suma de los «n» primeros términos de una «P. A.»»
2

Ejemplo 1. Determinar la suma de los términos de la progresión 3, 8, 13, …, 123


Aquí, a1 = 3, an = 123, r =?, n =?, Sn =? «identificación de datos e incógnitas»

r = a2 − a1 implica r = 8 − 3 o bien r = 5 «cálculo de «r»»

an = a1 + (n − 1) ∙ r implica 123 = 3 + (n − 1) ∙ 5 «sustituyendo datos en ecuación general»

123 = −2 + 5n es decir 5n = 125 o bien n = 25 «cálculo de «n»»


n∙(a1 + an ) 25 ∙ (3 + 123)
Sn = implica Sn = «sustituyendo datos en la fórmula»
2 2
25∙126
Sn = o bien Sn = 3.150 «suma de los «61» primeros términos de la «P. A.» dada»
2

CLEMENTE MORENO (MATERIAL DIDÁCTICO EN VALIDACIÓN) 6


Ejemplo 2. De la «P. A.» 5, 9, 13,… ¿cuántos términos deben tomarse para que sumen 1224?
Aquí, a1 = 5, Sn = 1224, r =?, an =?, n =? «identificación de datos e incógnitas»

r = a2 − a1 implica r = 9 − 5 o bien r = 4 «cálculo de «r»»

an = a1 + (n − 1) ∙ r implica an = 5 + (n − 1) ∙ 4 «sustituyendo datos en ecuación general»

an = 5 + 4n − 4 o bien an = 4n + 1 «cálculo de « an »»
n∙(a1 + an ) n∙[5+(4n+1)]
Sn = implica 1224 = «sustituyendo datos en la fórmula»
2 2

2448 = n ∙ (4n + 6) o bien 4n2 + 6n − 2448 = 0 «operando e igualando a cero»

O bien, 2n2 + 3n − 1224 = 0 «simplificando por «2»»


51
Así, n1 = − y n2 = 24 «resolviendo la ecuación de 2do grado»
2

Luego n = 24, el valor negativo carece de sentido ««24» es el número de términos a tomar»

Ejercicio 1
Con los datos indicados, construya la «P. A.» que se indica
1) a1 = −3; r = 4; n = 7; 2 3 √3
(2) a1 = ; r = ; n = 5; (3) a1 = −2√2; r = ; n = 5;
3 4 2
3 1 b b 1 2
4) 𝑎𝑎1 = ; 𝑟𝑟 = − ; n = 6; (5) a1 = ; r = ; n = 7; (6) a1 = ; r = − ; n = 6;
4 2 2 3 5 3

En cada caso encuentre los cinco términos siguientes, sabiendo que se trata de una «P. A.»
7) 3,12 (8) −1, 6, 13 (9) 4, 56 (10) 3, 0

Encuentre la razón de las siguientes «P. A.»


11) −1, −3, −5, −7…; (12) 1, 3, 2, 5… ; 31 27 23 (14) 1,5,9,13…;
(13) 7, , , …;
2 2 5 5 5

En cada caso determine los términos que se indican en el paréntesis


2 5 3 3 6 1 9 17
15) , , 1… ( a9 y an ) (16) − , , … ( a11 y an ) (17) − , − , − … (a9 y an )
3 6 5 10 5 4 4 4

18) 3, 2.7, 2.4… ( a13 y an ) (19) −7, −3.9, −0.8, 2.3… ( a15 y an )
20) a11 = 17; a12 = 25 ( a17 y an ) (21) a3 = −9; a10 = −
11
( a15 y an )
2

22) a3 = −9; a10 = −


11
( a1 ; r y an ) (23) a5 − a15 = 20; a7 + a11 = −18 ( a1 ; r)
2

CLEMENTE MORENO (MATERIAL DIDÁCTICO EN VALIDACIÓN) 7


Interpolar las medias aritméticas «m. a.» que se indican en cada caso
24) 3 «m. a.» entre 7 y 19; (25) 5 «m. a.» entre −5 y −17; (26) 4 «m. a.» entre 4 y −4;
3 1 3 1 1
27) 3 «m. a.» entre y 7; (28) 4 «m. a.» entre y ; (29) 2 «m. a.» entre y ;
2 2 16 2 3

Halle la suma de los términos de las «P. A.» siguientes


30) 1, 2, 3,… 200; (31) 2, 4, 6, …. 100; (32) −2, −5, −8, …, −56;
33) −80, −75, −70,…, 80; (34) 2 + i, 3, 4 − i,…, 28 − 25i; (36) 1,3,5,…., 2n − 1

Calcule la suma de
37) Los 100 primeros números pares; (38) Los 50 primeros números impares;
39) Los 35 primeros múltiplos de 5; (40) Los 42 primeros múltiplos de 7;
41) Los 15 primeros múltiplos de 41; (42) Los 20 primeros números que terminan en 7

43) El primer término de una «P. A.» es 13 y su 35-ésimo término es 235. Encuentre su
diferencia y los términos quito y 123-ésimo
1
44) ¿Cuántos términos hay en una «P. A.» que empieza en −2, tiene por razón y suma 21?
4

Progresiones geométricas
Una progresión geométrica es una sucesión tal que cada término después del primero, se
obtiene multiplicando a éste por una cantidad constante llamada razón

Ejemplo 1. La sucesión 2, 4, 8, 16, 32, … es una progresión geométrica «P. G.», ya que:

a1 = 2 «primer término de la P. G.»

a2 = 4 implica a2 = 2 ∙ 2 o bien a2 = a1 ∙ r «segundo término es igual anterior por la razón»

a3 = 8 implica a3 = 4 ∙ 2 o bien a3 = a2 ∙ r «tercer término es igual anterior por la razón»

a4 = 16 implica a4 = 8 ∙ 2 o bien a4 = a3 ∙ r «cuarto término es igual anterior por la razón»

a5 = 32 implica a5 = 16 ∙ 2 o bien a5 = a4 ∙ r «quinto término es igual anterior por la razón»

CLEMENTE MORENO (MATERIAL DIDÁCTICO EN VALIDACIÓN) 8


En general, si «an » es una «P. G.» que inicia en «a1 », con razón «r», se tiene

a2 = a1 ∙ r
a3 = a2 ∙ r
…………..
an = an−1 ∙ r
……………

COMENTARIO

a2 an
a2 = a1 ∙ r implica r = y en general an = an−1 ∙ r implica r = , luego una sucesión
a1 an−1

es geométrica si el cociente entre un término y el anterior es una cantidad constante

2
Ejemplo 3. Construir una «P. G.» de 5 términos que inicie en 3 y tenga por razón
3
2
Aquí, a1 = 3 y r = «identificación de datos»
3
2
a2 = 3 ∙ � � entonces a2 = 2 «cálculo 2do término»
3
2 4
a3 = 2 ∙ � � entonces a3 = «cálculo 3er término»
3 3
4 2 8
a4 = ∙ � � entonces a4 = «cálculo 4to término»
3 3 9
8 2 16
a5 = . � � entonces a5 = «cálculo 5to término»
9 3 27
16 2 32
a6 = ∙ � � entonces a6 = «cálculo 6to término»
27 3 81
4 8 16 32
Luego 2, , , , «es la «P. G.» solicitada»
3 9 27 81

2 8 32 128
Ejemplo 4. ¿Cuál es la razón de la «P. G.» , , , ,…?,
3 3 3 3
2 8 32 128
a1 = , a2 = , a3 = , a4 = «identificación de datos»
3 3 3 3
8
a2 8∙3
r= implica r = 3
2 o bien r = es decir r = 4 «cálculo de la razón»
a1 3∙2
3

CLEMENTE MORENO (MATERIAL DIDÁCTICO EN VALIDACIÓN) 9


2√2
Ejemplo 5. Insertar el término que falta en la «P. G.» 9, −3√2, ___, − ,…,
3

a1 = 9; a2 = −3√2 «identificación de datos»

a2 −3√2 √2
r= implica r = o bien r = − «cálculo de la razón»
a1 9 3

√2
a3 = a2 ∙ r implica a3 = �−3√2� ∙ �− � o bien a3 = 2 «cálculo 3er término»
3

2√2
Luego 9, −3√2, 2, − «inserción del 3er término»
3

Término general de una progresión geométrica

Dados a1 y la razón r «conocidos a1 y r»

a2 = a1 ∙ r
a3 = a2 ∙ r implica a3 = (a1 ∙ r) ∙ r o bien a3 = a1 ∙ r 2 «sustituyendo a2 = a1 ∙ r en a2 de a3 »

a4 = a3 ∙ r implica a4 = (a1 ∙ r 2 ) ∙ r o bien a4 = a1 ∙ r 3 «sustituyendo a3 = a1 ∙ r 2 en a3 de a4 »

a5 = a4 ∙ r implica a5 = (a1 ∙ r 3 ) ∙ r o bien a5 = a1 ∙ r 4 «sustituyendo a4 = a1 ∙ r 3 en a4 de a5 »

…… …………. ……….
an = a1 ∙ r n−1 (3) «fórmula general de una P. G.»

COMENTARIO

Si el desarrollo anterior se hubiese iniciado en un término cualquiera «ak », en lugar de


«a1 » la ecuación (3) se generaliza y se tendría
an = ak ∙ r n−k (4)

3
Ejemplo 1. ¿Cuál es el vigésimo término de la «P. G.» de razón 0,5 y tercer término ?
8
3 1
a3 = ; r = ; «identificación de datos»
8 2
a3
Y a3 = a1 ∙ r 2 implica a1 = «despejando a1 »
r2
3
3∙4 3
a1 = 8
1 o bien a1 = es decir a1 = «cálculo 1er término»
8∙1 2
4

CLEMENTE MORENO (MATERIAL DIDÁCTICO EN VALIDACIÓN) 10


3 1 19 1 20
Y, a20 = a1 ∙ r19 es decir a20 = ∙ � � o bien a20 = 3 ∙ � � «cálculo vigésimo término»
2 2 2

Ejemplo 2. El término general de una «P. G.» donde a1 − a2 = 8 y a3 − a4 = 72 es:


a1 − a2 = 8
Aquí, � ; a =?; r =? «identificación de datos e incógnitas»
a3 − a4 = 72 1
a1 − a1 ∙ r = 8 a (1 − r) = 8
Y, � 2 3 o bien � 12 «sistema equivalente, «(1 − r)» donde es factor común»
a1 ∙ r − a1 ∙ r = 72 a1 r (1 − r) = 72
a1 (1−r) 8 1 1
Y, = o bien = (∗) «dividiendo miembro a miembro y simplificando»
a1 r2 (1−r) 72 r2 9

De (∗) r 2 = 9 o bien r = ±3 «cálculo de la razón r»


8
Y, a1 (1 − r) = 8 implica a1 = «despeje de a1 »
1− r
8 8
Así, a1 = = −4 y a1 = =2 «cálculo 1er término»
1−3 1−(−3)

Luego, an = −4 ∙ (3)n−1 y an = 2 ∙ (−3)n−1 «término general de las «P. G.» que se forman»

Interpolación de medias geométricas

COMENTARIO

Interpolar una cantidad «k» de términos entre dos términos dados de una «P. G.», es
construir una «P. G.» de extremos los términos dados, es decir, «a1 » coincidente con el
extremo inferior, «an » coincidente con el extremo superior y «n = k + 2» el número de
términos de la progresión, en símbolos

an
an = a1 ∙ r [(k+2)−1] o bien = r k+1 «sustituyendo, operando y aislando a «r»»
a1

k+1 an
Así, r = �a «despejando a «r»»
1

Ejemplo 1. Interpolar 4 medias geométricas entre −5 y −160


a1 = −5; an = −160; k = 4; r =? «identificación de datos e incógnita»

k+1 a 4+1 −160 5


�a implica r = o bien r = √32 = 2
n
r= � «sustituyendo y operando»
1 −5

a2 = a1 ∙ r implica a2 = (−5) ∙ 2 o bien a2 = −10 «cálculo 2do término»

CLEMENTE MORENO (MATERIAL DIDÁCTICO EN VALIDACIÓN) 11


a3 = a1 ∙ r 2 implica a3 = (−5) ∙ 22 o bien a3 = −20 «cálculo 3er término»

a4 = a1 ∙ r 3 implica a3 = (−5) ∙ 23 o bien a3 = −40 «cálculo 4to término»

a5 = a1 ∙ r 4 implica a3 = (−5) ∙ 24 o bien a3 = −80 «cálculo 5to término»

Luego, −5, −10, −20, −40, −80, −120 «interpolación de las medias geométricas requeridas»

Suma de los n primeros términos de una «𝐏𝐏. 𝐆𝐆.»


Sea a1 , a2 , a3 ,… an−2 , an−1 , an una «P. G.» de «n» términos y «Sn » la suma de estos,
Así, Sn = a1 + a1 ∙ r + a1 ∙ r 2 + ⋯ + a1 ∙ r n−2 + a1 ∙ r n−1 (1) «sumando los «n» términos de la «P. G.»»

Y, −r ∙ Sn = −a1 ∙ r − a1 ∙ r 2 − a1 ∙ r 3 − ⋯ − a1 ∙ r n−1 − a1 ∙ r n (2) «multiplicando (1) por «−r»»

Así, Sn − r ∙ Sn = a1 − a1 ∙ r n «sumando miembro a miembro las ecuaciones (1) y (2)»

Luego Sn (1 − r) = a1 (1 − r n ) «tomando a Sn y a1 como factor común»

a1 (1 − rn )
Entonces Sn = «fórmula de la suma de los «n» primeros términos de una «P. G.»»
1−r

Ejemplo 1. Calcular la suma de los términos de la «P. G.» 6, −12, 24,…,1536


a1 = 6; an = 1536; r =?; n =? Sn =? «identificación de datos e incógnitas»
a2 −12
r= implica r = o bien r = −2 «cálculo de la razón r»
a1 6

an = a1 ∙ r n−1 implica 1536 = 6 ∙ (−2)n−1 «sustituyendo datos en la fórmula general»

(−2)n
Así, 256 = (−2)n−1 o bien 256 = «aplicando división de potencias de igual base»
(−2)

Y, −512 = (−2)n o bien (−2)9 = (−2)n así, n = 9 «número de términos de la «P. G.»»

a1 (1 − rn ) 6 ∙ (1−�−2)9 �
Luego Sn = implica S9 = «sustituyendo datos en la fórmula»
1−r 1−(−2)
6∙(1+ 512)
Así, S9 = o bien S9 = 1026 «suma requerida»
3

Producto de los n primeros términos de una «𝐏𝐏. 𝐆𝐆.»


Si a1 , a2 , a3 ,… an−2 , an−1 , an es una «P. G.» de «n» términos, entonces
Pn = a1 ∙ a1 r ∙ a1 r 2 ∙∙∙∙ an r −2 ∙ an r −1 ∙ an (1) «producto de los «n» primeros términos»

Pn = an ∙ an r −1 ∙ an r −2 ∙∙∙∙ a1 r 2 ∙ a1 r ∙ a1 (2) «producto invertido de los «n» primeros términos»

Pn2 = (a1 an ) ∙ (a1 an ) ∙ (a1 an ) ∙∙∙ (an a1 ) ∙ (an a1 )(an a1 ) «multiplicando miembro a miembro»

Pn = ±�(a1 an )n «fórmula del producto de los «n» primeros términos»

CLEMENTE MORENO (MATERIAL DIDÁCTICO EN VALIDACIÓN) 12


COMENTARIO

En la determinación del signo de Pn se tiene en cuenta lo siguiente:


(1) Si «P. G.» es de términos positivos, el producto de ellos es un número positivo
(2) Si «P. G.» es de términos negativos, el producto de ellos será positivo o negativo según
sea par o impar el número de términos
n = par ⟶ Pn > 0
Si 𝑎𝑎 < 0, 𝑟𝑟 > 0 entonces �
n = impar ⟶ Pn < 0
(3) Si «P. G.» es de términos oscilantes, es necesario considerar cuantos son los términos
positivos y cuantos los términos negativos
En particular, el signo de P14 en la «P. G.» 1, −2, 4… es negativo, debido a que habrán
7 términos negativos; mientras que en P25 para la «P. G.» 1,−3, 9 será positivo, porque
hay 12 términos negativos

135 45 15 40
Ejemplo 1. Determinar el producto de los términos de la «P. G.» ,− , , …,
8 4 2 27
135 40
a1 = ; an = ; r =?; n =? Pn =? «identificación de datos e incógnitas»
8 27
45
−4 45 ∙ 8 2
r= 135 o bien r = − entonces r = − «cálculo de la razón r»
4 ∙ 135 3
8

40 135 2 n−1
an = a1 ∙ r n−1 implica = ∙ �− � «sustituyendo datos en la fórmula general»
27 8 3
2 n
40 8 �−3� 2 2 6 2 n
Y, ∙ = 2 o bien �− � ∙ � � = �− � «despejando, operando y simplificando»
27 135 �−3� 3 3 3

2 7 2 n
Así, �− � = �− � entonces n = 7 «número de términos de la «P. G»»
3 3

135 40 7
Pn = ±�(a1 an )n implica P7 = ±�� ∙ � «sustituyendo datos en la fórmula»
8 27

P7 = ±�(52 )7 o bien P7 = ±57 «operando y simplificando»

Luego, P7 = −57 «hay tres términos negativos, pues la sucesión es oscilante y el 1er termino es positivo»

CLEMENTE MORENO (MATERIAL DIDÁCTICO EN VALIDACIÓN) 13


Ejercicio N° 2
Con los datos indicados, construya la «P. G.» que se indica
1) a1 = −3; r = 4; n = 9; 2
2) a1 = ; r = ; n = 8;
3
3) a1 = −2√2; r =
√2
; n = 5;
3 4 2
3 1 b b 1 2
4) 𝑎𝑎1 = ; 𝑟𝑟 = − ; n = 6; 5) a1 = ; r = ; n = 7; 6) a1 = ; r = − ; n = 6;
4 2 2 3 5 3

En cada caso encuentre los cinco términos siguientes, sabiendo que se trata de una «P. G.»
7) 3,12 8) −1, 6, −18 9) ,
2 8
10) 12, 10
3 15

Encuentre la razón de las siguientes «P. G.»


11) 2, − , ,…;
3 9
12) −4096, 1024, −256,…; 13) −2, − 4, − 8,…; 14)
125
,
25√15
,…;
2 8 3 9 9 9

En cada caso determine los términos que se indican en el paréntesis


1 1 1 4 8
15) 1, − , , − … ( a9 y an ) 16) √2, 2, √8, 4 ( a11 y an ) 17) −2, − , − ,… ( a9 y an )
2 4 8 3 9
1 1
18) a2 = 16 y a9 = ; ( a13 y an ) 19) a2 = 8192; a9 = 32; an = ( a15 y an )
8 2

( a17 y an ) 21) 𝑎𝑎1 + 𝑎𝑎3 = 8; 𝑎𝑎6 + 𝑎𝑎7 = 128 ( a15 y an )


5
20) a4 = − ; a7 = −15; an = −135
9

22) 𝑎𝑎5 − 𝑎𝑎1 = 6; 𝑎𝑎8 − 𝑎𝑎4 = 12√2 ( a1 ; r y an ) 23) 𝑎𝑎1 + 𝑎𝑎5 = 30; 𝑎𝑎3 + 𝑎𝑎7 = 10 ( a1 ; r)

Interpolar las medias geométricas «m. g.» que se indican en cada caso

; 25) 4 «m. g» entre −5 y −160;


25 √6
24) 4 «m. g» entre 75 y 26) 5 «m. g» entre 9√6 y
9 3
27) 2 «m.g» entre
√15
y
√5
; 28) 4 «m.g» entre −
128
y 196; 29) 3 «m. g» entre i y 16i;
45 5 343

Halle la suma de los términos de las «P. G.» siguientes


30) −3, −6,−12,…. −384; 31) −4, 20, −100,… 12500;
3 9 27 729 81 27 9 8
33) − , , − ,…. , 34) , , ,…
2 8 32 2048 32 16 8 81

Halle el producto de los términos de las «P. G.» siguientes


1 1 1 1 1 1
35) , , ,…, 64; 36) − , − , − ,….−243;
32 16 8 81 27 9
3 3 3 16 8 4 9
37) − , , − ,…, 1875; 38) , − , ,….,
125 25 5 81 27 9 4

CLEMENTE MORENO (MATERIAL DIDÁCTICO EN VALIDACIÓN) 14

También podría gustarte